Business Law - PDFCOFFEE.COM (2024)

Law for Business and Personal Use 18e JOHN E. ADAMSON

Australia • Brazil • Japan • Korea • Mexico • Singapore • Spain • United Kingdom • United States

Law for Business and Personal Use, 18th Edition

© 2009, 2006 South-Western, a part of Cengage Learning

John E. Adamson

ALL RIGHTS RESERVED. No part of this work covered by the copyright herein may be reproduced, transmitted, stored or used in any form or by any means graphic, electronic, or mechanical, including but not limited to photocopying, recording, scanning, digitizing, taping, Web distribution, information networks, or information storage and retrieval systems, except as permitted under Section 107 or 108 of the 1976 United States Copyright Act, without the prior written permission of the publisher.

Vice President of Editorial, Business: Jack W. Calhoun Vice President/Editor-in-Chief: Karen Schmohe Executive Editor: Eve Lewis Senior Developmental Editor: Enid Nagel Marketing Manager: Valerie Lauer

For product information and technology assistance, contact us at Cengage Learning Academic Resource Center, 1-800-423-0563

Marketing Coordinator: Kelley Gilreath Content Project Manager: Jennifer A. Ziegler Production Manager: Patricia Matthews Boies Senior Technology Project Editor: Sally Nieman Website Project Manager: Edward Stubenrauch Manufacturing Coordinator: Kevin Kluck Editorial Assistant: Virginia Wilson

For permission to use material from this text or product, submit all requests online at www.cengage.com/permissions Further permissions questions can be emailed to [emailprotected] ExamView ® and ExamView Pro® are registered trademarks of FSCreations, Inc. Windows is a registered trademark of the Microsoft Corporation used herein under license. Macintosh and Power Macintosh are registered trademarks of Apple Computer, Inc. used herein under license. The Career Clusters icons are being used with permission of the:

Production Service: ICC Macmillan Inc. Senior Art Director: Tippy McIntosh Internal Designer: Trish Knapke, Ke Design States’ Career Clusters Initiative, 2007, www.careerclusters.org Cover Designer: Trish Knapke, Ke Design © 2008 Cengage Learning. All Rights Reserved. Cover Image: Kelly Redinger/Design Pics/Corbis ISBN-13: 978-0-538-44588-7 ISBN-10: 0-538-44588-2 South-Western Cengage Learning 5191 Natorp Boulevard Mason, OH 45040 USA Cengage Learning products are represented in Canada by Nelson Education, Ltd. For your course and learning solutions, visit school.cengage.com

Printed in the United States of America 1 2 3 4 5 6 7 11 10 09 08

Reviewers Brenda Albright-Barnhart

Linda Hughes

Tammy Savage

Digital Academy Career Tech Department Bolton High School Alexandria, Louisiana

Business Educator Rapides High School Lecompte, Louisiana

Business Department Chair Spain Park High School Hoover, Alabama

Kristine Labbus

Janice B. Shelton

Judith Kay Binns

Business Education Instructor Neenah High School Neenah, Wisconsin

Business Teacher Mills E. Godwin High School Richmond, Virginia

Jallane Link

Lynn M. Taillon

Business Education Teacher Puxico R-8 School Puxico, Missouri

Business Office Technology Instructor Canadian Valley Technology Center Chickasha, Oklahoma

Business Educator Cheshire High School Cheshire, Connecticut

Leo Dunn

Catherine McDonald

Business Teacher Waynesboro Area High School Waynesboro, Pennsylvania

Teacher, Business Department Bryant High School Bryant, Arkansas

Dawna Carter

Instructor, Business and Technology Greater Lowell Technical High School Tyngsboro, Massachusetts

Senior Teacher, Business Ridgefield High School Ridgefield, Connecticut

Nancy Everson

Business Teacher Beloit Memorial High School Beloit, Wisconsin

Chair, Business Education Sun Prairie High School Sun Prairie, Wisconsin

Christine Ferreira Business Teacher Algonquin Regional High School Northboro, Massachusetts

Richard T. Gordon Lead Teacher, Business & Applied Tech Department Bloomington High School Bloomington, Illinois

Vicki Noss

Jeanne Robinson Teacher, Business Department South Technical High School Sunset Hills, Maryland

Thomasine Montoya Teacher/Coordinator Cibola High School Albuquerque, New Mexico

Caroline Janovec Tassone

Tamara M. Tucker Teacher, Business Department Columbia High School Columbia, South Carolina

Grant Pierce Williams Business Instructor/FBLA Adviser Harrison High School Harrison, Arkansas

Kevin W. Willson Business Education Chair York Suburban Senior High School York, Pennsylvania

About the Author John E. Adamson is Emeritus Professor of Business and Law in the Department of Finance and General Business at Southwest Missouri State University. Adamson received a B.S. from the U.S. Military Academy at West Point, New York; an M.A. from

Georgetown University; and an M.B.A. and J.D. from the University of Virginia at Charlottesville. A decorated, disabled veteran and past mayor and school board member of Miller, Missouri, Adamson is author of numerous business law publications.

Mock Trial Consultant Melissa A. Marcin Attorney at Law Milford, Ohio Reviewers

iii

Contents Unit 1 Law, Justice, and You 2 Planning a Career in Law: Trial Lawyer 3 Entrepreneurs and the Law: Law, Justice, and You 102 Community Action: Get Out the Vote 103

Winning Edge: FBLA Emerging Business Issues 103 Mock Trial Prep: Concepts of Advocacy 104

Features in Unit 1 A Question of Ethics 17, 29, 57, 65, 87 Cyberlaw 8, 86 Economic Impact 66 Global Issues 6, 32, 71 Hot Debate 4, 24, 48, 64, 80 In This Case 11, 16, 42, 54, 70, 73, 83, 85, 93

Law Brief 9, 29, 58, 74, 94 Legal Research 10, 38, 50, 69, 91 Net Bookmark 19, 32, 53, 72, 95 Online Research 97 What’s Your Verdict? 5, 6, 10, 12, 13, 16, 25, 28, 31, 33, 36, 39, 49, 52, 55, 57, 65, 67, 68, 71, 72, 74, 81, 82, 83, 85, 89, 91, 93, 94, 96

Chapter 1 Laws and Their Ethical Foundation

Chapter 4 Criminal Law and Procedure

1-1 1-2 1-3

Laws and Legal Systems Types of Laws Ethical Bases for Laws Chapter 1 Assessment Sports & Entertainment Law

Chapter 2 Constitutional Rights 2-1 2-2 2-3

Foundations of the U.S. Constitution Division and Balance of Governmental Powers The Internet and Constitutional Rights Chapter 2 Assessment Sports & Entertainment Law

Chapter 3 Court Systems 3-1 3-2 3-3

iv

Forms of Dispute Resolution The Federal Court System State Court Systems Chapter 3 Assessment Case For Legal Thinking

Contents

4 5 10 16 20 23

24 25 31 36 44 47

48 49 52 55 60 63

4-1 4-2

Criminal Law Criminal Procedure Chapter 4 Assessment Sports & Entertainment Law

Chapter 5 Civil Law and Procedure 5-1 5-2 5-3

64 65 71 76 79

80

Private Injuries vs. Public Offenses 81 Intentional Torts, Negligence, and Strict Liability 85 Civil Procedure 93 Chapter 5 Assessment 98 Sports & Entertainment Law 101

Unit 2 Contract Law 106 Planning a Career in Law: Franchisee 107 Entrepreneurs and the Law: Contract Law 222 Community Action: Positive Peer Pressure 223

Winning Edge: FBLA Multimedia Presentation 223 Mock Trial Prep: Case Scenario 224

Features in Unit 2 A Question of Ethics 112, 133, 148, 159, 183, 193, 216 Cyberlaw 134, 161 Economic Impact 115, 171 Global Issues 116, 132, 143, 157, 180, 207 Hot Debate 108, 126, 140, 156, 170, 190, 206 In This Case 109, 115, 119, 127, 130, 142, 147, 159, 163, 173, 177, 181, 182, 193, 199, 208

Law Brief 120, 132, 150, 158, 173, 193, 215 Legal Research 110, 134, 150, 164, 178, 200, 210 Net Bookmark 111, 133, 145, 158, 175, 198, 209 Online Research 217 What’s Your Verdict? 109, 110, 114, 116, 118, 127, 128, 130, 131, 133, 141, 145, 147, 149, 157, 160, 162, 163, 164, 171, 174, 177, 179, 181, 191, 192, 197, 199, 207, 208, 210, 213, 214

Chapter 6 Offer and Acceptance

Chapter 10 Legal Purpose and Proper Form

6-1 6-2 6-3

Creation of Offers Termination of Offers Acceptances Chapter 6 Assessment Sports & Entertainment Law

Chapter 7 Genuineness of Assent 7-1 7-2

109 114 118 122 125

126

Duress and Undue Influence 127 Mistake, Misrepresentation, and Fraud 130 Chapter 7 Assessment 136 Case For Legal Thinking 139

Chapter 8 Consideration 8-1 8-2 8-3

108

140

Types of Consideration Questionable Consideration When Consideration Is Not Required Chapter 8 Assessment Sports & Entertainment Law

141 145 149 152 155

Chapter 9 Legal Capacity to Contract 156 9-1 9-2

170

10-1 Illegal Agreements 10-2 The Statute of Frauds Chapter 10 Assessment Sports & Entertainment Law

171 177 186 189

Chapter 11 Contractual Obligations and their Enforcement

190

11-1 Transfer and Discharge of Obligations 11-2 Remedies for Breach of Contract Chapter 11 Assessment Sports & Entertainment Law

Chapter 12 Contractual Aspects of Marriage and Divorce

191 197 202 205

206

12-1 Marriage and the Law of Contracts 12-2 Divorce and the Law of Contracts Chapter 12 Assessment Case For Legal Thinking

207 213 218 221

Contents

v

Contractual Capacity of Individuals and Organizations 157 Limits on the Rights of Those Without Capacity 162 Chapter 9 Assessment 166 Sports & Entertainment Law 169

Unit 3 The Law of Sales 226 Planning a Career in Law: Border Patrol Agent 227 Entrepreneurs and the Law: The Law of Sales 278

Community Action: Become a Consumer Activist 279 Winning Edge: FBLA Public Speaking I 279 Mock Trial Prep: Case Law: Search and Seizure 280

Features in Unit 3 A Question of Ethics 230, 250, 270 Cyberlaw 251, 264, 271 Economic Impact 252, 268 Global Issues 231, 244 Hot Debate 228, 242, 258 In This Case 232, 236, 244, 248, 270

Chapter 13 Sales Contracts 13-1 Sales 13-2 Special Rules for Sales Contracts Chapter 13 Assessment Case For Legal Thinking

Chapter 14 Ownership and Risk of Loss in Sales 14-1 Transfer of Ownership 14-2 Risk of Loss and Insurable Interest in Sales Chapter 14 Assessment Case For Legal Thinking

Law Brief 232, 249, 261 Legal Research 230, 251, 260 Net Bookmark 233, 251, 261 Online Research 273 What’s Your Verdict? 229, 232, 234, 235, 243, 245, 247, 249, 259, 260, 263, 266, 268

228 229 234 238 241

242

Chapter 15 Consumer Protection

258

15-1 Protection through Governmental Action 15-2 Protection through Action by the Consumer Chapter 15 Assessment Case For Legal Thinking

243 247 254 257

Unit 4 Property Law 282 Planning a Career in Law: Title Examiner 283 Entrepreneurs and the Law: Property Law 372 Community Action: Protect Your Environment 373

Winning Edge: BPA Presentation Management 373 Mock Trial Prep: Opening Statements

374

Features in Unit 4 A Question of Ethics 291, 307, 326, 349, 365 Cyberlaw 286, 320, 349 Economic Impact 346 Global Issues 290, 306, 325 Hot Debate 284, 300, 314, 334, 356 In This Case 286, 290, 301, 307, 319, 324, 338, 344, 348, 359, 365

vi

Contents

Law Brief 287, 306, 318, 345, 361 Legal Research 293, 305, 317, 348, 366 Net Bookmark 290, 303, 321, 343, 364 Online Research 367 What’s Your Verdict? 285, 289, 292, 301, 302, 305, 307, 315, 319, 323, 325, 335, 336, 338, 340, 343, 347, 348, 357, 360, 364, 366

259 266 274 277

Chapter 16 Property and Its Acquisition 284 16-1 Types of Property 16-2 How Property Is Acquired and Held Chapter 16 Assessment Case For Legal Thinking

Chapter 17 Bailments 17-1 Bailments 17-2 Bailor and Bailee Duties Chapter 17 Assessment Case For Legal Thinking

Chapter 18 Ownership and Leasing of Real Property 18-1 Ownership and Transfer of Real Property 18-2 Leasing Real Property Chapter 18 Assessment Case For Legal Thinking

285 289 296 299

300 301 305 310 313

Chapter 19 Insurance Law

334

19-1 Insurance Fundamentals 19-2 Property and Casualty Insurance 19-3 Life and Social Insurance Chapter 19 Assessment Case For Legal Thinking

335 340 347 352 355

Chapter 20 Wills, Trusts, and Estates 356 20-1 Property Distribution Upon Death 20-2 Trusts Chapter 20 Assessment Case For Legal Thinking

357 364 368 371

314 315 323 330 333

Unit 5 Agency and Employment Law 376 Planning a Career in Law: Pension Investigator 377 Entrepreneurs and the Law: Agency and Employment Law 458 Community Action: Homeland Security at School 459

Winning Edge: BPA Human Resources Management 459 Mock Trial Prep: Direct and Cross Examination 460

Features in Unit 5 A Question of Ethics 385, 404, 414, 436, 450 Cyberlaw 429, 446 Economic Impact 436 Global Issues 388, 402, 420 Hot Debate 378, 394, 410, 426, 442 In This Case 381, 387, 398, 404, 414, 417, 428, 433, 444, 449

Law Brief 384, 403, 412, 428, 452 Legal Research 380, 402, 416, 430, 447 Net Bookmark 381, 396, 411, 434, 451 Online Research 437 What’s Your Verdict? 379, 380, 384, 386, 387, 395, 396, 401, 404, 411, 413, 416, 417, 427, 429, 432, 434, 435, 443, 447, 449

Chapter 21 Agency Law

378

Chapter 22 Employment Law 394

379 384 390 393

22-1 Making and Terminating Employment Contracts 395 22-2 Duties of Employers and Employees 401 Chapter 22 Assessment 406 Sports & Entertainment Law 409

21-1 Creation and Operation of Agencies 21-2 Agency Duties Chapter 21 Assessment Case For Legal Thinking

Contents

vii

Chapter 23 Unions and the Employment Relationship

410

23-1 Establishment of Unions 23-2 Employment Relations in a Unionized Workplace Chapter 23 Assessment Sports & Entertainment Law

411 416 422 425

Chapter 25 EmploymentRelated Injuries

442

25-1 Safety on the Job 25-2 Employer’s Liability for Work-Related Injuries Chapter 25 Assessment Sports & Entertainment Law

443 447 454 457

Chapter 24 Discrimination in Employment 426 24-1 Legal versus Illegal Discrimination 24-2 Proving Illegal Discrimination Chapter 24 Assessment Sports & Entertainment Law

427 432 438 441

Unit 6 Legal Forms of Business Organization 462 Planning a Career in Law: Corporate Attorney 463 Entrepreneurs and the Law: Legal Forms of Business Organization 524

Community Action: Ethical Business Conduct 525 Winning Edge: FBLA Business Plan 525 Mock Trial Prep: Evidentiary Questions: Objections 526

Features in Unit 6 A Question of Ethics 467, 498, 512 Cyberlaw 494, 516 Economic Impact 488, 514 Global Issues 473, 490 Hot Debate 464, 486, 508 In This Case 467, 472, 477, 487, 495, 502, 509, 516

Law Brief 480, 498, 518 Legal Research 471, 501, 518 Net Bookmark 466, 494, 517 Online Research 503 What’s Your Verdict? 465, 469, 473, 476, 479, 487, 490, 493, 496, 500, 501, 509, 511, 514, 517

Chapter 26 Forms of Business Organization 464

Chapter 28 Organizational Forms for Small Business

26-1 Main Forms of Business Organization 465 26-2 Creating and Terminating Partnerships 469 26-3 Operating Partnerships 476 Chapter 26 Assessment 482 Case For Legal Thinking 485

28-1 Traditional Small Business Forms 28-2 New and Evolving Small Business Forms Chapter 28 Assessment Sports & Entertainment Law

Chapter 27 The Law of Corporations 27-1 Founding a Corporation 27-2 Shareholders, Directors, and Officers 27-3 Corporate Powers and Termination Chapter 27 Assessment Sports & Entertainment Law viii

Contents

486 487 493 500 504 507

508 509 514 520 523

Unit 7 Borrowing Money and Paying Bills 528 Planning a Career in Law: Forensic Accountant 529 Entrepreneurs and the Law: Borrowing Money and Paying Bills 614 Community Action: Protect Against Identity Theft 615

Winning Edge: BPA Extemporaneous Speaking 615 Mock Trial Prep: Closing Arguments 616

Features in Unit 7 A Question of Ethics 534, 554, 563, 578, 602 Cyberlaw 539, 570 Economic Impact 581, 604 Global Issues 538, 548 Hot Debate 530, 544, 560, 576, 592 In This Case 533, 546, 553, 561, 569, 578, 584, 594, 601

Law Brief 531, 547, 568, 577, 605 Legal Research 538, 551, 570, 584, 606 Net Bookmark 534, 548, 564, 582, 599 Online Research 609 What’s Your Verdict? 531, 532, 537, 545, 546, 551, 552, 561, 562, 564, 568, 577, 578, 581, 586, 593, 597, 601, 604, 606

Chapter 29 Commercial Paper

Chapter 32 Secured and Unsecured Credit Transactions

530

29-1 Basic Types of Commercial Paper 29-2 Specialized Types of Commercial Paper Chapter 29 Assessment Case For Legal Thinking

531 537 540 543

Chapter 30 Negotiability and Negotiation of Commercial Paper 544 30-1 Requirements of Negotiability 30-2 Proper Indorsem*nt and Negotiation Chapter 30 Assessment Case For Legal Thinking

Chapter 31 Discharge of Commercial Paper and Electronic Fund Transfers

545 551 556 559

560

31-1 Discharge of Commercial Paper 31-2 The Law of Electronic Fund Transfers Chapter 31 Assessment Case For Legal Thinking

561 568 572 575

576

32-1 What Is a Secured Credit Transaction? 32-2 How Are Security Interests Perfected and Terminated? Chapter 32 Assessment Case For Legal Thinking

Chapter 33 Creditors, Debtors, and Bankruptcy

577 581 588 591

592

33-1 Legal Protection of Creditors 33-2 Legal Protection of Debtors and Credit Card Users 33-3 Bankruptcy Chapter 33 Assessment Case For Legal Thinking

593 597 604 610 613

Appendix A Constitution of the United States

618

Appendix B The Declaration of Independence 630 Glossary of Legal Terms

633

Case Index

649

Index

652

Contents

ix

To the Student

Y

ou will soon find business law is one of your most valuable subjects. You will study true situations that show how business and personal law impacts not only business, but the lives of young people and adults as well. The learning package will help you achieve an understanding of legal principles you will use throughout your life. Use the following plan to help you effectively study Law for Business and Personal Use.

How to Study the Textbook

• •

• • •

x

Each chapter opens with a Hot Debate. Read the Hot Debate scenario and then discuss with your class the questions that go along with it. Each chapter is divided into two or three lessons. Each lesson has a list of Goals at the beginning. Read the Goals and then scan the lesson to see where each goal is addressed. Read each lesson slowly and carefully. Make notes of important points. In the Student Activities and Study Guide, there is an outline of every lesson to help you take notes. Each main topic heading is followed by a What’s Your Verdict? scenario. Try to answer the question. Then read the following paragraphs to learn the law that applies to What’s Your Verdict? before you go on to the next topic. As you read, apply what you learn to your own experiences or those of your family and friends. Think about situations within your own experience to which the law applies. At the end of each main topic section, ask yourself the Checkpoint question to review the material. After you have carefully studied the lesson, complete the exercises in Think About Legal Concepts and Think Critically About Evidence. After you complete all the lessons in a chapter, read the Concepts in Brief to refresh your memory. Then complete Your Legal Vocabulary, Review Legal Concepts, Write About Legal Concepts, Make Academic Connections, and Analyze Real Cases. To the Student

The exercises in Think Critically About Evidence help you apply what you learned in the chapter. Analyze Real Cases allows you to work with the facts from real lawsuits related to the topics in each chapter.

Special Features

• • • • • • • • •

• • •

A Question of Ethics presents ethical issues that arise within the legal framework. Cyberlaw discusses emerging legal aspects of the Internet and e-commerce. Economic Impact discusses a situation in which the law is affected by a certain economic circ*mstance. Global Issues illustrates legal procedures and issues in other nations. Hot Debate promotes thoughtful discussion of important legal issues. In This Case demonstrates legal principles and concepts through hypothetical situations. Law Brief provides unusual or interesting facts or ideas related to the law. Legal Research helps you investigate issues relating to federal, state, and local laws using Internet and library resources. Net Bookmark encourages you to use the Internet for research. Find the URLs you need to complete the research on the Xtra! web site at school.cengage.com/blaw/lawxtra. Planning a Career in Law introduces you to seven law-related careers within various career clusters. Online Research gives you a scenario and then instructs you to use the Findlaw web site to answer questions about it. What’s Your Verdict? motivates you to learn new legal concepts.

Mock Trial Competitions

Mock Trial Prep features in each unit cover court and trial procedures to prepare you for participation in mock trial competition.

Real-World Cases

• •

Case for Legal Thinking presents important real-world cases for analysis and discussion. Sports & Entertainment Law presents cases involving sports and entertainment figures.

To analyze the dilemmas using fundamental ethical rules, • “Universalize” the action in question. • Determine whether the action is irrational, illogical, or self-defeating.

Projects

How to Read Case Citations

Law cases are referenced in a way that makes them easy to find. There are three parts to a citation. For example, 28 A2d 309 identifies (1) a series of law books, (2) one volume in that series, and (3) the page where the case begins. In the example, A2d identifies the series of books that report the decisions of certain courts. The A stands for Atlantic Reporter, a series that reports the cases of appellate courts in the North Atlantic Region. The 2d indicates that the case appears in the second series of the Atlantic Reporter. The 28 in this example citation refers to Volume 28 in the series. The case begins on page 309.

• •

Entrepreneurs and the Law allows you to apply the law to a start-up business situation. Community Action provides ideas for getting involved and making a difference in your local, national, and global communities. Winning Edge helps you prepare for business law-related FBLA and BPA competitive events.

How to Analyze Legal Situations Following each lesson and each chapter you will find a number of legal situations in Think Critically About Evidence. You also will find actual cases that have been decided by courts in Analyze Real Cases, Case for Legal Thinking, and Sports & Entertainment Law. You may use the same method to analyze all of these exercises. To answer the question raised, first read it carefully. Be sure you understand the question and the facts involved. Then determine the rule of law involved and reach a decision. You will find it helpful to answer these questions.

• • • • •

What are the facts? What is the disputed point? What rule of law is involved? How does this rule apply to the facts? What is the answer or decision?

Legal Advice Consult a lawyer if you have any doubts about your rights or duties when your property, life, or liberty is endangered or if significant changes occur in your circ*mstances.

• • •

How to Analyze Ethical Situations An ethical dilemma is posed in A Question of Ethics. You can use the ethical frameworks described in Lesson 1-3 to analyze these dilemmas. To analyze the dilemmas using ethical reasoning based on consequences, • Describe alternative actions that would improve the situation. • Forecast consequences that would flow from each alternative described. • Evaluate the consequences for each alternative by selecting a standard for judging right or wrong consequences.

Familiarize yourself with local, state, and federal laws to help avoid violations. Ignorance of the law normally is no excuse. Remember that as a minor you generally are liable for crimes and torts and bound by contracts. If you are involved in a legal dispute, try to learn the other person’s version and honestly seek a friendly solution out of court. In every court action at least one person loses and often both find the costs burdensome. If someone injures you or your property, do not rush to sign a statement releasing the person from liability in exchange for some payment of money. The damages may be greater than they appear at first. Consult your attorney immediately. Although oral agreements can be legally binding, you should write out all contracts that involve significant time, money, or detail and have both parties sign and receive copies.

To the Student

xi

Unit 1 Law, Justice, and You Chapters 1 Laws and Their Ethical Foundation 2 Constitutional Rights 3 The Court System 4 Criminal Law and Procedure 5 Civil Law and Procedure

2

Planning a Career in Law Trial Lawyer

EMPLOYMENT OUTLOOK

Employment opportunities for trial lawyers are expected to grow steadily through 2014, primarily as a result of growth in the population and in the general level of business activities. Competition for job openings should continue to be keen because of the large number of students graduating from law school each year. Graduates with superior academic records from highly regarded law schools will have the best job opportunities. The willingness to relocate may be an advantage in getting a job, but to be licensed in another state, a lawyer may have to take an additional state bar examination.

NEEDED SKILLS AND EDUCATION

• •

Prospective lawyers should develop proficiency in writing and speaking, reading, researching, analyzing, and thinking logically. A multidisciplinary background is recommended. Students interested in a particular aspect of law may find related courses helpful. Courses in English, foreign languages, public speaking, government, philosophy, history, economics, mathematics, and computer science, among others, are useful. The required college and law school education usually takes seven years of full-time study

after high school—four years of undergraduate study, followed by three years of law school. Law school applicants must have a bachelor’s degree to qualify for admission.

How you’ll spend your day As a new trial lawyer you’ll put in long hours assisting senior lawyers. The work involves fact gathering and legal research. The amount of information you’ll need to organize and sort before trial can be daunting, but the work is excellent training. After awhile you’ll sit in on trials, participate in conferences with judges, and prepare arguments. Eventually you will represent your own clients in court. In a criminal case you may represent the prosecution or the accused. In a civil case you’ll take the side of your client in a private dispute. Your job will be to persuade a jury of the facts in the case. You’ll present the facts in a way that best supports your client’s position, using evidence to prove your case. When not in court you’ll review files, contact witnesses, take depositions, and talk to your client. Preparing for a trial can take weeks or months. Many cases settle before they ever reach trial. In these instances you’ll work for the best settlement for your client. On court days you’ll meet with judges, select jurors, and argue your case.

What about you? Does the job of trial attorney interest you? Why or why not? Which aspects of the job would you most enjoy? Which aspects would you least enjoy?

© RUBBERBALL

A

re you good at arguing and debate? Do you like to defend others? Are you passionate about justice being served for the good of society? If so, you might consider a career as a trial lawyer. Trial lawyers are needed in both criminal and civil law courts. Trial lawyers must be able to think quickly and speak with ease and authority. Knowledge of courtroom rules and strategy is particularly important in trial work. Trial lawyers spend most of their time outside the courtroom, conducting research, interviewing clients and witnesses, and handling other details in preparation for a trial. Criminal trial attorneys work as defense attorneys or for the local, state, or federal government as a prosecutor.

Chapter 1 Laws and Their Ethical Foundation

With her stereo system blasting, Clarisse began her drive home from school in her small pickup. She was soon pulled over by a police officer. The officer cited her for violation of the city’s noise ordinance. As he did so, his voice was partially drowned out by the sound of jack hammers from a nearby construction site. Clarisse asked the officer why he had not cited the construction company as well. He replied that, even though the company was in violation, the noise ordinance had been put on the books to stop people from playing loud music in public, not to stop honest work.

WHERE DO

YOU STAND?

1. Is the ordinance fair? 2. Has the ordinance been fairly applied? 3. Can you suggest some changes in the ordinance that might make it a better law?

4

Chapter 1 > Laws and Their Ethical Foundation

© GETTY IMAGES/PHOTODISC

1-1 Laws and Legal Systems 1-2 Types of Laws 1-3 Ethical Bases for Laws

1-1 Laws and Legal Systems GOALS

KEY TERMS

■ Explain the stages in the growth of law

laws code common law

■ Describe the differences between common law

and positive law ■ Identify the origin of the U.S. legal system

What Is Law? Two archeologists, Professor DiPalermo and Professor Shuster, were deciphering some stone law tablets from the Middle East dating back before 2000 B.C.E. Looking up from his work, Professor DiPalermo commented that, while the amount of knowledge held by humanity had increased dramatically, its basic nature had not changed. Professor Shuster agreed, saying, “Just compare the laws they had way back then with the ones we have now. We’re still making the same mistakes and still need the same protections from the conduct of others.”

What's Your Verdict?

positive law jurisdiction equity

Stages In the Growth of Law Most societies go through four distinct stages in forming their legal systems: 1. Individuals are free to take revenge for wrongs done to them 2. A leader acquires enough power to be able to force revenge-minded individuals to accept an award of goods or money instead 3. The leader gives this power to a system of courts 4. The leader or central authority acts to prevent and punish wrongs that provoke individuals to seek revenge

The laws, or enforceable rules of conduct in a society, reflect the culture and circ*mstances that create them. Laws may be grouped into an organized form referred to as a code. When you compare one civilization’s code with the codes of other civilizations, you see many similarities. For example, the law code set down about four thousand years ago by Hammurabi, King of Babylon, had sections on criminal law, property law, business law, family law, personal injury law, labor law, and others. Such coverage is similar to that found in the U.S. law codes today. The professors in What’s Your Verdict? were correct. The need for law has not changed much over recorded history. People still make the same mistakes and still need the same protections from the conduct of others.

© NORTH WIND PICTURE ARCHIVES

Do you think their observations are correct?

The law code set down by King Hammurabi of Babylon contained sections or categories similar to those found in modern U.S. law codes. How do you think the specific content of Hammurabi’s codes compares to that of U.S. law codes today?

1-1 Laws and Legal Systems

5

In the first stage, injuries inflicted on one human being by another are matters for personal revenge. Those who are wronged feel that justice can be done only through personally punishing the wrongdoers. Gang wars in the inner cities often result from this type of attitude. Whether they occur in a big city or in a developing society, such events usually disrupt the normal productive routine of the people and result in harm to innocent bystanders. The situation often leads to one individual taking power and exerting control to bring peace to the society. To achieve this peace, the powerful leader or authority, often called the sovereign, hears and resolves disputes between the people. The sovereign then forces the injured parties to accept awards of money or goods as a substitute for their taking revenge. This is the second stage in the growth of law. In many civilizations, the sovereign ultimately becomes faced with more cases than one person can handle. As a consequence, the sovereign sets up a system of courts and gives them powers to decide certain types of disputes. Elders or priests generally preside over these courts. However, the sovereign still presides over the most important matters. In the fourth and final stage, the sovereign takes a step beyond being merely a passive authority working only to resolve disputes after they occur. Instead the sovereign tries to prevent breaches of the peace before they can occur. The sovereign does this through enforcing a set of laws and matching punishments.

some laws are set down by a sovereign or other central authority to prevent disputes and wrongs from occurring in the first place. Law dictated from above in this fashion is called positive law. CHECKPOINt How does common law

differ from positive law?

What Is the Origin of the U.S. Legal System? LaBonne, from Louisiana, was visiting his cousin in St. Louis, Missouri. While they were talking about the differences in traffic laws in the two states, LaBonne told his cousin that Louisiana’s legal system was different from that used in Missouri and all the other states.

What's Your Verdict?

Is LaBonne correct?

The world’s two great systems of law are the English common law and the Roman civil law. Countries with systems patterned after the Roman civil law have adopted written, well organized, comprehensive sets of statutes in code form. The laws in these codes are typically only changed by the central government, not by the

Common Law versus Positive Law Laws reflect the wisdom—or lack thereof—of their creators. In any society laws should be both predictable and flexible. A system of laws that is not predictable will not produce a stable society. Chaos, unrest, and the replacement of the system by one that can exercise control and restore peace will follow. A legal system that is too controlling and too rigid to change with the wants and needs of the people also will be overthrown. The best system of laws always evolves slowly towards a form that is most appropriate to the current standards of the people. Law based on the current standards or customs of the people is called common law. Common law usually is formed from the rules used by judges to settle people’s disputes. However, as noted earlier, 6

Napoleonic Code The territory that is now Louisiana was claimed for France by the explorer Lasalle in 1682. French settlers brought many French influences to the state of Louisiana—customs, food, language, and law. Louisiana’s legal system was developed using the French Code Napoléon (or Napoleonic Code) as a foundation. However, regardless of the legal system it uses, over the years Louisiana has modified its laws to correspond with those of the other 49 states. The Napoleonic Code was derived from Germanic customs and Roman law. The Napoleonic Code also was the model for the civil law codes of Italy, the Netherlands, Spain, and the Canadian Province of Quebec.

Chapter 1 > Laws and Their Ethical Foundation Copyright 2009 Cengage Learning, Inc. All Rights Reserved. May not be copied, scanned, or duplicated, in whole or in part.

English Common Law Before the English common law system developed, feudal barons acted as judges within their territories. Disputes were settled on the basis of local customs and enforced by the barons’ power. Because of this, the laws of England differed from region to region. Such differences were difficult for people to follow. They also made it hard for a central government to maintain control. KING’S BENCH Around 1150 King Henry II

decided to improve the situation. He appointed a number of judges from a group of trusted nobles. King Henry gave these judges the power to order that wrongdoers pay with money or goods the parties they injured. In good-weather months the judges would “ride circuit” into the countryside, holding court in the villages. During bad-weather months, the judges stayed in London and sat together as a court to hear cases on appeal that might have been decided unwisely on circuit. This court so formed by the circuit-riding judges came to be called King’s Bench—or Queen’s Bench if the regent was female. The baron’s courts, which heard local cases before the King’s courts were created, kept the power to decide some of the minor cases. However, the King’s courts always had jurisdiction—the power to decide a case—over the most important cases. JURY King Henry recognized that it was important to decide the court cases in harmony with the customs of the people. To do otherwise would cause unrest, if not revolution. The judges were instructed to choose citizens from each region to help interpret that region’s customs for the court. This panel of citizens evolved into what we know today as the jury. The jury is an institution unique to the English common law system. AN EXAMPLE The early system of English

common law worked something like this. Imagine that a farmer named William is on his way to

© NORTH WIND/NORTH WIND PICTURE ARCHIVES

judges who administer them. Only one state in the United States—Louisiana—has a civil law system. The legal system used in the other 49 states is based on the English common law. Therefore, in What’s Your Verdict?, LaBonne was correct. Colonists from England brought the common law system to this continent. To understand how this system works, you must look back to England to see how it was developed.

Because he could not resolve all the disputes in the nation by himself, King Henry appointed a group of judges to decide them for him. What personal qualities do you think he looked for in these judges?

market one morning in his ox cart. He is traveling through an unfamiliar region. As he approaches an intersection, he sees another person in a similar ox cart coming into the intersection from his right. In William’s home region the right of way at an intersection goes to the person on the left. So he continues on, expecting the other person to rein in. The other party, a local resident named Gwen, does not yield and a collision results. Both William and Gwen are injured. Their oxen are gored. Their carts and other property are destroyed. As a consequence, the next time the circuit-riding judge comes into Gwen’s region, both people appear in court and request damages (a monetary award) for their losses. The judge needs to know who is at fault in the case in order to decide who must pay. To find out, the judge chooses a jury of 12 residents. These people must decide which person, according to their customs, acted improperly. The jury determines that, because the right of way customarily goes to the person on the right in their region, William is at fault and must pay Gwen damages. The judge accepts the decision and orders William to pay. The decision, however, upsets William. He knows that throughout most of his travels the right of way is given to the person on the left. Therefore, he decides to take his case to the higher, or appellate, court—King’s Bench in London—on appeal. That court will not be in session for several 1-1 Laws and Legal Systems

7

months yet, so William uses the time to collect information on the law used in other courts in England that have ruled on the issue of right of way. Finally, the time comes to appear before King’s Bench. The judges listen to William’s appeal and Gwen’s defense of the lower court’s decision. The appellate court judges review the information presented to them by both parties, including the laws used by other courts to settle like issues. They decide that it would be wisest to reverse the holding in the lower court (that the right of way should go to the person on the right). Instead, says King’s Bench, the right of way will be given to the person on the left. They send the case back down to the lower court with instructions to enter a judgment for damages in William’s favor. From that point on, anyone in the kingdom will need to give the right of way to the person on the left. If any lower court, including the one in Gwen’s region, decides a case using a different rule, the result can be appealed to King’s Bench, where it will be reversed. ADVANTAGES OF ENGLISH COMMON LAW The judicial process described in the example was repeated over and over in England throughout the centuries. As a result, a uniform web of custombased common law developed across the whole of England. The process used to achieve this end is called the English common law system. The English common law system achieves uniformity while maintaining an ability to adapt to changes in society. It has been a model for legal systems worldwide, including that in the United States.

A

h, the joys of e-mail—instant communication of thoughts. You feel it. You say it. You send it. But, if your message is perceived as a threat, you could be prosecuted and convicted for it. In the first conviction of an online hate crime, a 21-year-old Los Angeles man was found guilty in federal court. He sent death threats by e-mail to more than 50 Asian students. The case set a precedent, as it put Internet communications on equal legal ground with telephone calls and postal mail. It also addressed civil rights violations committed online—hate crimes in this case. The defense

8

Chapter 1 > Laws and Their Ethical Foundation

Equity:An Alternative to Common Law The common law courts carefully follow precedent. This means the courts use prior cases as a guide for deciding similar new cases as would be done with the decision in William v. Gwen above. Following precedent helps to provide stability in the law. However, if overdone it can have the disadvantage, as came to be the situation in old England, of requiring a rigid adherence to proper form. For example, a misplaced period or misspelled word could nullify, or void, the effect of a document. Another disadvantage of the early common law system was that its courts could only grant the remedy of damages. This meant that common law courts had to wait until the harm actually occurred before they could take action. For example, if a farmer decided to dam up the stream that watered the neighbor’s crops and animals, the courts of law had to wait until the harm had occurred and then award the neighbor damages for what the farmer did. The courts of law could not order the farmer to stop building the dam. This inability to stop a wrong before it inflicted actual harm often resulted in a waste of resources from the perspective of the country as a whole. However, if the neighbor were a noble, he might be able to get around the courts and directly petition the king for help. The king would refer the matter to his chancellor, who was usually a high clergyman respected for his equity, or fairness.

team argued this was a “stupid prank” and that so-called flames or abusive messages are commonplace with Internet culture and discussion groups. The jury thought otherwise and took the threats seriously. At first the Net was considered to be a fantasy land where users could be anonymous. This case shows that the legal system will not treat the Net differently from other forms of communication. THINK ABOUT IT Do you think people should be held accountable for threats they make online? Why or why not?

The chancellor would conduct a hearing under rules different from those of a common law court. There would be no jury, for example, and the remedies the chancellor could impose in the king’s name were different from those available to the law courts. The chancellor might issue an order to compel that something be done. Or, he might issue an injunction, which stops something from being done. For example, the chancellor could issue an injunction to stop the dam from being built. However, if the neighbor were not a noble, he would not have been able to petition the king. The harm would be allowed to occur. Eventually the king sensed a need for access to equitable remedies for all citizens. He created a system of equity courts and placed them under the chancellor’s control. These courts were given the power to issue injunctions or to compel

T

hree states in the United States administer law and equity separately. In Delaware and Mississippi equity is administered in chancery courts. In Tennessee equity is administered in law-equity courts.

specific actions. In the United States today, law courts and equity courts generally are merged. Consequently, most American courts can award damages or issue orders or both. CHECKPOINt On which early legal system

is the U.S. legal system based?

1-1 Assessment school.cengage.com/blaw/lawxtra

THINK ABOUT LEGAL CONCEPTS 1. Substitution of damages for revenge is the first stage in the evolution of law. True or False? 2. The two systems of law in use today are the English common law and the (a) American Constitution (b) French legal code (c) Roman common law (d) none of the above. 3. A remedy of the English Courts of Equity was the (a) injunction (b) court order (c) disputation (d) none of the above.

4. Louisiana is the only one of the 50 states whose legal system was not originally based on the English common law system. True or False? 5. Most American law courts can use either damages or an injunction or both as remedies in civil cases. True or False?

THINK CRITICALLY ABOUT EVIDENCE Study the situation, answer the questions, and then prepare arguments to support your answers. 6. You are on your daily jog when a car negligently pulls out in front of you. Unable to stop, you run into it and injure yourself. Should you be able to recover damages for the harm done to you? 7. The driver of the car in exercise 6 becomes abusive towards you after your recovery. The driver follows you on your jogs and yells threats at you. He has recently taken to driving very close to you as you jog. What can you do legally to make him stay away from you? 8. Cracked Mirror, a local rock group, contracts to play for your high school prom. A week before

the dance, the group cancels its appearance. A teacher finds out that the band booked a concert that will pay them $800 more. The class president’s mother is an attorney and offers her services to the school. If you sue the band for damages, what would be an appropriate amount and why? 9. For the situation in exercise 8, fashion an equitable remedy that might encourage Cracked Mirror to decide to keep its commitment to play at your prom.

1-1 Laws and Legal Systems

9

1-2 Types of Laws GOALS

KEY TERMS

■ Identify the four sources of law

constitution statutes ordinances case law stare decisis

■ Discuss how conflicts between laws are

resolved ■ Compare and contrast criminal and civil law, and substantive and procedural law

What Are the Sources of Law? What's Your Verdict?

The federal Constitution guarantees the citizens of the United States many rights. These include freedom of speech, of press, of assembly, to petition, to bear arms, against unreasonable searches and seizures, and more. What part of the U.S. Constitution contains most of these guarantees?

Laws in this country are created at all three levels of government—federal, state, and local. The forms that these laws can take include constitutions, statutes, case law, and administrative law.

Constitutions A constitution is a document that sets forth the framework of a government and its relationship to the people it governs. When constitutions are adopted or amended, or when courts interpret constitutions, constitutional law is made. You are governed by both the Constitution of the United States and the constitution of your state. The Supreme Court of the United States is the final interpreter of the federal Constitution. Constitutions are the highest sources of law, and the federal Constitution is “the supreme law of the land” (U.S. Constitution, Article VI). This means that any federal, state, or local law is not valid if it conflicts with the federal Constitution. Similarly, within each state the state constitution is supreme over all other state laws. Federal and state constitutions are concerned primarily with defining and allocating certain 10

Chapter 1 > Laws and Their Ethical Foundation

administrative agencies civil law criminal law procedural law substantive law

powers in our society. Constitutions allocate powers (1) between the people and their governments, (2) between state governments and the federal government, and (3) among the branches of the government. ALLOCATION OF POWER BETWEEN PEOPLE AND THEIR GOVERNMENTS The federal Constitution

is the main instrument for allocating powers between people and their governments. It does this primarily with its first ten amendments, called the Bill of Rights. The Bill of Rights protects people from actions of their governments. This is the section of the federal Constitution referred to in What’s Your Verdict? The Bill of Rights will be discussed in detail in Chapter 2.

The Fraudulent Online Identity Sanctions Act (H.R.3754) was signed into law in December 2004. It increased prison sentences by up to seven years for someone who provided “material and misleading false contact information to a domain name registrar, domain name registry, or other domain name registration authority.” It was intended to prevent online scams and also make it much easier for the recording studios and movie producers to track down copyright violators and obtain the $150,000 per violation damages. However, opponents of the law saw it as a danger to the exercise of several Constitutional rights. They also claimed it would be ineffective in achieving its intended results. Research and assess the accuracy of these criticisms. Does the Act achieve its intended goals?

© GETTY IMAGES/PHOTODISC

the people. Acting for their citizens, these legislatures enact laws called statutes. In addition, all states delegate some legislative authority to local governments. Thus, towns, cities, and counties can legislate on matters over which the state has given them authority. These pieces of legislation created by a town or city council or by a county board or commission are typically referred to as ordinances. They are only effective within the boundary of the local governments that enacted them.

Case Law

Railroads often are used to transport goods over long distances. What type of commerce, interstate or intrastate, does this represent, and which level of government most likely regulates it?

ALLOCATION OF POWER BETWEEN FEDERAL AND STATE GOVERNMENTS The federal Constitution

also allocates powers between the federal and state governments. For example, many governmental powers over business are divided between state governments and the federal government on the basis of commerce. In general, the Constitution gives the federal government the power to regulate both foreign and interstate commerce. Interstate commerce occurs between two or more states. The power to regulate intrastate commerce, which occurs within one state, is left with that state. ALLOCATION OF POWER AMONG THE BRANCHES OF GOVERNMENT State and federal constitutions

also allocate governmental powers among the three branches of government: executive, legislative, and judicial. Traditionally these powers are distributed so as to create a system of checks and balances between the branches of each government. This ensures that no branch of government becomes too powerful. For example, the U.S. Constitution gives the federal courts the authority to determine which laws passed by Congress are constitutional. However, in turn it gives Congress the power to ordain and establish the courts in the first place.

Statutes The federal Constitution created the Congress of the United States. State constitutions created the state legislatures. These state and federal legislatures are composed of elected representatives of

The judicial branch of governments creates case law. Case law usually is made after a trial has ended and one of the parties has appealed the result to a higher court. This appeal will be based on legal issues arising from rulings made by the lower court in deciding the case. When the appellate court publishes its opinion on a case, that opinion may state new or more appropriate rules to be used in deciding the case and others like it. These rules are referred to as case law at either the federal or state level. The effectiveness of case law arises out of the doctrine of stare decisis. This is Latin for “let the decision stand.” This doctrine requires that lower courts must follow established case law in deciding similar cases. Generally, case law doctrines are carefully established and seldom revoked. However, the doctrine of stare decisis generally does not bind supreme courts. Carol borrowed her stepfather’s car without his express permission. The police stopped her and discovered the car was not registered in her name. They then phoned her stepfather. When he said he did not know where his car was, Carol was arrested. At her trial, Carol and her stepfather testified that she had his permission to use the car without asking each time. The trial judge nevertheless found Carol guilty of auto theft, which, the judge stated, occurs when one person takes the car of another without expressly asking permission. Carol appealed to the state supreme court. That appellate court issued an opinion stating implied permission is enough, and, therefore, Carol was innocent. This rule then became state case law to be applied in similar cases.

In THIS CASE

1-2 Types of Laws

11

Administrative Law Federal, state, and local legislatures all create administrative agencies. Administrative agencies are governmental bodies formed to carry out particular laws. The federal Social Security Administration, your state’s division of motor vehicles, and your county’s zoning commission are examples of administrative agencies. Although created by legislatures, administrative agencies usually are controlled by the executive branch of government. Thus, the president, governor, or mayor will supervise the agency’s activities. Legislatures sometimes give administrative agencies legislative powers and limited judicial powers. Legislative power means the agency is authorized to create administrative laws, also called rules and regulations. For example, the federal Social Security Administration might establish rules for determining when a student is depending on support from a widow or widower and thereby qualified to receive Social Security payments. If an agency has judicial power, it can hold hearings, make determinations of fact, and apply the law to particular cases. In the above example, the Social Security Administration might hold a hearing to judge whether a particular student is in fact a dependent. Once this determination is made, the Social Security Administration would use its executive powers to carry out the rules on payments to dependents of widows or widowers. CHECKPOINt What are the four sources

of law?

What Happens When Laws Conflict? What's Your Verdict?

When adopted, the U.S. Constitution provided that there could be no income tax. So when Congress levied a 2 percent income tax in 1894, the U.S. Supreme Court declared it unconstitutional. Many people wanted the federal government to raise money by taxing incomes because the burden imposed would be based on one’s current ability to pay. What did the people do to override the Supreme Court decision?

12

Chapter 1 > Laws and Their Ethical Foundation

Sometimes laws created by different levels of government conflict. For example, a city ordinance setting a speed limit of 35 mph on a state highway near an elementary school may conflict with a state statute setting 45 as a minimum speed limit on state roads. Different types of laws created by the same level of government also may conflict. A federal administrative regulation requiring the phase-out of coal-fired electric-generating plants may conflict with a federal court decision holding that such regulations are unconstitutional. In these situations, legal rules are used that determine which statement of the law is superior to the other and should therefore be enforced. Generally, these supremacy rules hold that federal law prevails over state law and state law prevails over local (city and county) law. Within the state and federal systems, constitutional law prevails over statutory law and statutory law over administrative law. Finally, within the state and federal court systems, a higher court’s decision prevails over a lower court’s. CONSTITUTIONS AND VALIDITY Constitutions are the highest sources of law, and the federal Constitution is “the supreme law of the land” (U.S. Constitution, Article VI). This means that any federal, state, or local statute, case law or administrative decision is not valid if it conflicts with the federal Constitution. Within each state’s legal jurisdiction, the state constitution is supreme to all other state laws. When any type of law is declared invalid by a state or federal court because it conflicts with a constitution, it is said to be unconstitutional. Such a determination may be appealed to the highest court within the state and federal systems. Within the federal system, the U.S. Supreme Court has the final say. The highest state court wins out on state issues. Of course, even when interpreting constitutions, courts are not the ultimate authority. The people have the power to amend constitutions if they disagree with the courts’ interpretations. In answer to What’s Your Verdict? the people’s response to the Supreme Court decision was the Sixteenth Amendment to the U.S. Constitution. Adopted in 1913, it gave Congress the power to lay and collect an income tax. This in effect nullified the U.S. Supreme Court decision. STATUTES AND VALIDITY As discussed, statutes or ordinances must be constitutional to be valid. In addition, when needed in a particular case, courts also examine the statutes and ordinances involved to

see whether or not the law’s enactment exceeded the scope of the powers of the body that authored it.

individuals to seek legal remedies for wrongs done to them. (Civil law in this sense does not refer to the comprehensive system of law mentioned in Lesson 1-1.) For example when a tenant fails to pay the rent as promised, the landlord has the right to sue the tenant. (The police do not take action in civil matters.) If a defendant loses a civil case, that defendant is liable. This means that she or he must pay compensation to the plaintiff for his or her loss. This is typically done by the payment of money in civil matters. In addition to enforcing legal promises, civil law also applies whenever one person is injured by another. Such private wrongs (civil offenses) against people or organizations are referred to as torts. A crime is an offense against society rather than individuals. It disrupts the stable environment that we all depend upon to make civilization work. So, when the citizens’ right to live in peace is violated by such activity, the offense is governed by criminal law. Acting in the name of all the people, the government investigates an alleged wrongdoing. If a crime has been committed and the person responsible can be found, the government will prosecute. Conviction of a crime can result in a fine, imprisonment, and in some states, execution. Usually when a crime occurs, private injuries may be inflicted as well. A result may be both a crime and a civil offense. Thus, the civil law may also apply, and the victim of the crime may sue the wrongdoer. In What’s Your Verdict? Bolyston committed both a crime and a civil offense. Exceeding the

ADMINISTRATIVE REGULATIONS AND VALIDITY Administrative regulations also can be

reviewed by courts to determine whether they are constitutional. The courts also may invalidate a rule or regulation if it is outside the scope of powers delegated to the agency by the legislature that created it. CASE LAW AND VALIDITY A case law decision by a

court holding a statute invalid is not always the end of the issue. A legislative body has the power to nullify a court’s interpretation of a statute or ordinance by rewriting the statute. Administrative agencies also can revise their regulations when challenged. CHECKPOINt Which source of law in the

United States is the highest authority?

What Are the Main Types of Laws? Bolyston was steering her 18 wheeler down a steep hill in the outside lane of westbound I–44. Gradually her speed built up to over 85 mph. On the right-angled curve at the bottom of the hill, the truck’s speed carried it into the inside lane where it side-swiped JJ’s car and forced it into a guard rail. JJ lost control and his car veered across both eastbound lanes and into a concrete barrier. Thanks to a seat belt, an air bag, and quick avoidance by other traffic, JJ’s injuries were minor, but his car was demolished.

What's Your Verdict?

Laws may be classified in various ways. Common classifications include civil law, criminal law, procedural law, substantive law, and business law.

Civil and Criminal Laws When the private legal rights of an individual are violated, the matter is governed by civil law. The use of the term civil law within the common law system refers to the group of laws that allows

© GETTY IMAGES/PHOTODISC

Did Bolyston violate civil law or criminal law or both?

Is shoplifting a crime or a tort? Explain your answer.

1-2 Types of Laws

13

speed limit was a crime for which Bolyston could be arrested, convicted in a criminal trial, and fined. In addition, Bolyston committed a civil offense when she carelessly allowed her truck to smash into the side of JJ’s car. JJ could probably win a civil lawsuit against Bolyston for compensation for his injuries and the value of his car.

Procedural and Substantive Laws responsibilities can be legally exercised and enforced through the legal system. Procedural laws, for example, determine what remedies are available in a lawsuit and how those remedies are to be secured. They determine whether equitable remedies, such as an injunction, are available. The doctrine of stare decisis is a procedural law. Rules for determining the supremacy of conflicting laws are procedural laws. In contrast, substantive law defines rights and duties. It is concerned with all rules of conduct except those involved in enforcement. Substantive laws define offenses, such as murder, theft, breach of contract, and negligence. There are two types of procedural law: civil procedure and criminal procedure. Criminal procedure defines the process for enforcing the law when someone is charged with a crime. Civil procedure is used when a civil law has been violated. Civil law is concerned only with private offenses. When a civil law is violated the injured party is entitled to protect his or her rights. Police and public prosecutors generally do not involve themselves in the dispute.

Business Law Business law covers rules that apply to business situations and transactions. This book’s table of contents shows that the scope of business law is broad. Business law is important for all students—not just those planning careers in business or law. Most business transactions involve a merchant and a consumer. As you study business law, you will gain legal knowledge that will make you a better consumer. Business law mainly is concerned with civil law, especially contracts. The area of the law pertaining to commercial torts is another category of business law. Such torts are distinct from breaches of contracts. For example, torts may occur when manufacturers make defective products that injure users. Business activities are at times also governed 14

Chapter 1 > Laws and Their Ethical Foundation

© GETTY IMAGES/PHOTODISC

Procedural law sets forth how rights and

Assume that the woman in this photo is a public prosecutor giving her closing argument in a case to a jury. Is this most likely a civil trial or a criminal trial? Justify your answer.

by criminal law. For example, criminal law would punish a firm that conspires with competitors to fix prices or an employee who steals company tools. UNIFORM BUSINESS LAWS Laws of our various

states do not have to be alike as long as they are constitutionally valid. However, with the growth of interstate commerce and large business firms, more uniformity among state laws governing business and commercial transactions is important. Committees of legal experts have written model laws covering such areas as sales, certain credit transactions, and business forms. Sets of these model laws were then offered to the states for adoption in place of their current statutes covering the areas. The result has been more uniformity in state commercial laws. The Uniform Commercial Code (UCC) is a widely adopted uniform business law. It governs such areas as sales of goods, certain aspects of banking, and leases of goods. You will learn more about the UCC throughout this book. CHECKPOINt Compare and contrast

criminal and civil law and substantive and procedural law.

TYPES OF LAW Constitutional law

Based on constitutions

Statutory law

Enacted by legislative bodies

Administrative law

Rule-making by administrative agencies

Civil law

Addresses wrongs done to individuals

Criminal law

Addresses wrongs done to society

Procedural law

Deals with methods of enforcing legal rights and duties

Substantive law

Defines legal rights and duties

Business law

Rules that apply to business transactions

1-2 Assessment school.cengage.com/blaw/lawxtra

THINK ABOUT LEGAL CONCEPTS 1. The first ten amendments to the U.S. Constitution are known as the _ ?_. 2. Which of the following would not be considered civil law? (a) contract law (b) tort law (c) felonies (d) All of the above would be considered civil law. 3. Legislative enactments by a city government are called _ ?_. 4. Stare decisis is the doctrine that requires lower courts to adhere to existing case law in their decisions. True or False?

5. Torts are private wrongs committed against individuals or organizations. True or False? 6. Business activities are at times governed by criminal law. True or False? 7. Businesses cannot commit torts. True or False? 8. The statutory definition of murder is a (a) substantive law (b) procedural law (c) prohibitive law (d) none of the above

THINK CRITICALLY ABOUT EVIDENCE Study the situation, answer the questions, and then prepare arguments to support your answers. 9. Suppose the principal of your public school required all students to recite a prayer at the start of each school day. What level of government (federal, state, or local) would most likely determine whether or not the principal’s action was constitutional? 10. In exercise 9, what governmental body would make the determination? 11. Sonoma County passed a law making it legal to drive 65 mph on freeways inside the county. A state law limited all vehicles anywhere in the state to 55 mph. What is the valid speed limit on freeways inside this county? 12. In 1896, the U.S. Supreme Court held in Plessy v. Ferguson, 163 U.S. 537, that equal treatment of different races is provided when public and semipublic facilities, even though separate,

are substantially equal in quality. For years, railroad cars, buses, schools, and other facilities had separate and supposedly equal facilities for blacks. In 1954 black plaintiffs in Delaware, Kansas, South Carolina, and Virginia sought admission for their children to public schools on an integrated basis. Does the doctrine of stare decisis bar the U.S. Supreme Court from changing the law declared in Plessy v. Ferguson? (Brown v. Board of Education, 347 U.S. 483) 13. On a two-week vacation in a neighboring state, you buy several large firecrackers and take them home. A police officer notices them in your car on a routine traffic stop. She cites you for possession of an illegal explosive device, which is a felony in your state. Possession of fireworks in the neighboring state is not a crime. Can this be a defense for you?

1-2 Types of Laws

15

1-3 Ethical Bases for Laws GOALS

KEY TERMS

■ Define ethics

ethics civil disobedience

■ Compare and contrast consequences-based

ethics with rule-based ethics ■ Discuss ways in which ethics are reflected in laws

Ethics and the Law What's Your Verdict?

In the early 1960s, Dr. Martin Luther King, Jr., wanted to lead a march into Birmingham, Alabama, to protest racial segregation in that city. When he applied for a parade permit, his request was denied. Dr. King, knowing that his conduct was illegal, led the nonviolent march anyway. He was at the front of the line and allowed himself to be arrested, although he could have escaped easily. He went to jail. Community leaders were highly critical of Dr. King because he had violated the law. Was there sufficient ethical justification for Dr. King’s violation of the law?

Ethics is a practice of deciding what is right

or wrong in a reasoned, impartial manner. To involve ethics, a decision must affect you or others in a significant way. An ethical decision is one that is reasoned out typically by referring to an established authority that provides consistency. The law is such an authority. So are religious texts such as the Torah, the Bible, and the Koran. For example, a person might reason, “I believe that God is the source of the Bible, and the Bible tells me not to lie. Therefore, it would be wrong, or unethical, for me to lie.” To make ethical decisions, we usually must base our decisions on reason, not on emotion. In addition to being reasonable, ethical decisions should be impartial. Impartiality is the idea that the same ethical standards are applied to everyone. The reason you are learning about ethics in general is to prepare you to apply ethical concepts to business decision making. Business ethics are the ethical principles used in making business decisions. 16

Chapter 1 > Laws and Their Ethical Foundation

All too often, however, ethics are not considered when business decisions are made. The reason can be summarized in two words: profit maximization. The idea of profit maximization is supported by those who need a justification for actions that would hurt the general welfare. These actions might include moving factories offshore, having skilled jobs done overseas instead of by American workers, and buying solid, conservatively run companies only to close them down and sell the company’s assets piecemeal. To move toward a more ethically motivated economy, the profit maximization justification for such actions will need to be replaced by more humane ethical standards.

Basic Forms of Ethical Reasoning Ethical reasoning about right and wrong takes two basic forms. One form is based on consequences. In this style of ethical reasoning, rightness or wrongness is based only on the results of the action. Particular acts have no ethical, or moral, character. As Gabriella was pulling out of her parking spot at the mall, her tires slipped on the coating of ice and snow left by a recent winter storm. Her car slid sideways, causing her bumper to crunch into the back fender of the car parked next to her. Gabriella’s car was undamaged by the impact, but the bumper left a large dent and deep scratches on the other car. Gabriella realized she could just drive off without anyone knowing. Then she considered that the damage would probably cost more than a thousand dollars to repair. Consequently, she flagged down a mall security car and had the other owner paged so as to take appropriate responsibility.

In THIS CASE

An act that produces good consequences is good. An act that produces bad consequences is bad. The other form of moral reasoning is based on ethical rules. In this style of reasoning, acts are either right or wrong. For example, telling the truth is always right, and lying is always wrong. In rule-based ethics, good consequences do not justify wrong or bad acts. For example, in rulebased ethics, you cannot justify lying by showing that it produces good consequences. For almost all ethical decisions, these two forms of reasoning reach the same conclusion. In the decision of whether to lie or to tell the truth, for example, both forms usually conclude that one should not lie. CONSEQUENCES-BASED ETHICAL REASONING

Consequences-based reasoning first looks for alternative ways to alter the current situation. Then it attempts to forecast the consequences that will arise from each alternative. Finally, it evaluates those possible consequences to select the alternative that will generate the greatest good. This last criterion often is very subjective as to how one defines the good—financial reward, pleasure, love, justice. Who will be receiving “the good” often is a major consideration in this reasoning process. RULE-BASED ETHICAL REASONING With fun-

damental ethical rules, the acts themselves are judged as right or wrong. The standard for judging usually comes from one of two sources—a recognized authority or human reasoning. The authority, frequently a religious source, often clearly prescribes what is correct by timetested rules such as those found in the Ten Commandments. Human reasoning also can

T

he term culture refers to a society’s shared values, beliefs, and behaviors. Values reflect the goals a society considers important. Values also express the culture’s ideas of how people should act, as well as ideas about what is good, right, and desirable. Culture expresses a society’s ethics. Different cultures may have very different ethical systems. What is considered ethical behavior in one culture may not be acceptable behavior in another. For example, in Islamic societies, women’s activities outside the home are greatly restricted. Compare this to the role of women in U.S. society today. Every society has a unique culture, which evolves and changes over time. Each culture develops a set of laws based on the ethical values expressed at the time. Laws evolve as the culture’s sense of ethics evolves.

show what is basically wrong or right. A test, called universalizing, has been developed to help in this effort. In universalizing, you picture everyone doing the action and then ask yourself, “Would the result be irrational, illogical, or demeaning?” If so, the action is considered ethically wrong.

Ethics Reflected in Laws Under the U.S. system of democracy, representatives must vote for laws that are acceptable to the

© PHOTODISC/GETTY IMAGES

Universalize cheating on tests. Is it unethical? Why or why not?

1-3 Ethical Bases for Laws

17

18

Chapter 1 > Laws and Their Ethical Foundation

In contrast to the self-serving behavior of scofflaws, some people care passionately about ethical behavior, human rights, and justice. Their concern for justice sometimes compels them to violate what they consider to be an unjust law—a law they believe to be in conflict with ethical reasoning. They violate the law by engaging in acts of civil disobedience. Civil disobedience is an open, peaceful, violation of a law to protest its alleged, or supposed, injustice. The goal of those who engage in civil disobedience is not to advance their self-interest but rather to make the legal system more just. The participants may be willing, or even eager, to be arrested in order to test the validity of the law in court. In What’s Your Verdict? Dr. Martin Luther King, Jr., engaged in civil disobedience. Dr. King believed that civil disobedience is justified only in very limited circ*mstances. He and others conclude that civil disobedience is ethical when ■

a written law is in conflict with ethical reasoning ■ no effective political methods are available to change the law

© FLIP SCHULKE/COBBS

majority of people if the representatives expect to be reelected. Because this system is grounded on majority rule, it uses many of the features of consequences-based ethics. In this system, laws are judged to be right or good when they affect the majority of the people positively. Laws are judged to be wrong when they affect the majority negatively. Thus the government, empowered by the U.S. Constitution, seeks to ensure that the federal lawmaking system provides the greatest good for the greatest number. On the other hand, the Constitution also seeks to protect the well-being of minorities that might be taken advantage of by the wrong actions of the majority. This is done primarily through the rights preserved in the first ten amendments to the Constitution. Such protection of basic human dignity afforded by the Bill of Rights and other civil rights laws reflect ethics based on rules. Both ethics based on consequences and ethics based on rules conclude that we are obligated to obey the law. According to consequences-based reasoning, when the law is violated, many more people are injured than are benefited. With rule-based reasoning, if we say that we have agreed to obey the law but violate it, we are breaking our promise. If we universalize promise breaking by imagining that everyone always breaks promises, there would be no point to promising. In this universalized state, promise breaking is illogical or pointless and thus wrong. Even if unethical, the ease with which laws can be broken, especially by those with superior abilities or knowledge, such as a computer programmer, makes such behavior tempting. Aware of this, employers often purchase fidelity bonds for persons who handle large sums of money, such as cashiers, managers of movie theaters, or supervisors of restaurants. A fidelity bond is an insurance policy that pays the employer money in the case of theft by employees. Unethical, lawbreaking behavior does not always occur on a large scale, however. Often, individuals do not obey even minor laws. These individuals are termed scofflaws. These are persons who do not respect the law at all. They simply assess the risk of being caught and punished against the benefits they obtain by breaking the law. They lack personal integrity, or the capacity to do what is right in the face of temptation or pressure to do otherwise. They think they are smart because they frequently violate valid laws without being caught. A scofflaw is never ethically justified in violating the law.

the civil disobedience is nonviolent ■ the civil disobedience does not advance a person’s immediate self-interest ■ the civil disobedience is public and one willingly accepts the punishment for violating the law As a result of Dr. King’s efforts, many human rights were extended for the first time to several minority groups in this country. CHECKPOINt In the U.S. system of

Access school.cengage.com/blaw/lawxtra and click on the link for Chapter 1. Read the excerpt from Dr. Martin Luther King’s autobiography. How did Henry David Thoreau’s essay on “Civil Disobedience” influence Dr. King’s development as a civil rights activist?

democracy, how are ethics reflected in laws?

school.cengage.com/blaw/lawxtra

1-3 Assessment school.cengage.com/blaw/lawxtra

THINK ABOUT LEGAL CONCEPTS 1. Ethics is a practice of deciding what is right or wrong in a reasoned, _?_ manner. 2. The two forms of ethical reasoning almost never reach the same conclusions. True or False? 3. Which of the following best describes the elements involved in civil disobedience? (a) violating

the law (b) violating the law openly (c) violating the law openly and peacefully (d) violating the law openly, peacefully, and accepting punishment for the violation 4. Majority rule usually involves which of the following types of ethical reasoning? (a) reasoning based on consequences (b) rule-based reasoning

THINK CRITICALLY ABOUT EVIDENCE Study the situation, answer the questions, and then prepare arguments to support your answers. 5. Conner walked past the candy section in the grocery store and quickly stuffed a handful of candy bars into her purse. A store security guard saw her do it, and she was arrested. Her parents came to the police station after her arrest to take her home. Did Conner’s shoplifting significantly affect anyone? If so, who? Was Conner basing her conduct on emotion or reason? Was she treating herself and the other customers and stockholders of the store equally? 6. Sharon knows about tax laws and how the Internal Revenue Service (IRS) audits tax returns. She knows a way to cheat on her tax return that would save her almost $2,000. She thinks her chance of being caught is about one in 100. Can this cheating be justified by reasoning based on ethical rules? Can it be justified by ethics based on consequences?

7. An ordinance of Walker County provided that all automobiles must pass a smog emissions test once a year. Ross was ticketed because his car had not been inspected and approved at a testing center. Ross claimed to be a skilled mechanic who kept his car well tuned and cleaner than the law required. According to Ross, the law violated his natural rights. Explain why you agree or disagree with Ross. 8. If a legislature enacted a law that made it illegal to shout “fire” in a movie theater, would the dominant ethical character of the law reflect consequences-based or rule-based reasoning? 9. Assume a state legislature enacted legislation which budgeted more money to educating rich children than to educating poor children. Also assume that the majority of children are poor. Would complying with the law reflect more of a rule-based or a consequences-based ethic?

1-3 Ethical Bases for Laws

19

Chapter 1 Assessment

school.cengage.com/blaw/lawxtra

CONCEPTS IN BRIEF 1-1 Laws and Legal Systems 1. Laws are the enforceable rules of conduct in a society. 2. The two great systems of law in the world are the English common law and the Roman civil law. The legal systems of all the states except Louisiana are based on the English common law. 3. Sources of law include constitutions, statutes, cases decided by appellate courts, and regulations and rulings of administrative agencies. 1-2 Types of Laws 4. Civil law is concerned with the private legal rights of individuals and governs relations between individuals. Criminal law deals with crimes (offenses against society) and governs the behavior of individuals in relation to the laws of a society.

5. Procedural law deals with methods of enforcing legal rights and duties. Substantive law defines those rights and duties.

1-3 Ethical Bases for Laws 6. Business law is concerned with the rules that apply to business situations and transactions. 7. Ethical reasoning should be applied to reach reasonable and impartial decisions that affect people. 8. Ethics are either based on consequences or on fundamental rules. 9. Civil disobedience is the open, peaceful violation of a law to protest its alleged injustice or unfairness. It is only justified in rare and extraordinary circ*mstances.

YOUR LEGAL VOCABULARY Match each statement with the term that it best defines. Some terms may not be used. 1. Doctrine requiring lower courts to adhere to existing case law in making decisions a. administrative agency 2. Group of laws that set forth how rights and responsibilities can b. case law be legally exercised and enforced through the legal system c. civil disobedience 3. Enforceable rules of conduct in a society d. civil law 4. Law based on the customs of a group of people e. code 5. Legislative enactment by a city f. common law 6. Group of laws defining and setting punishments for offenses g. constitution against society h. criminal law 7. An open, peaceful, violation of a law to protest its alleged injustice i. equity 8. Power of a court to decide a case j. ethics 9. Law enacted by state legislatures or federal legislatures k. jurisdiction 10. Law made when an appellate court endorses a rule to be used l. laws in deciding court cases m. ordinance 11. The practice of deciding what is right or wrong in a reasoned, impartial manner n. positive law 12. Document that sets forth the framework of a government and o. procedural law its relationship to the people it governs p. stare decisis 13. Group of laws that allows individuals to seek legal remedies for q. statute wrongs done to them r. substantive law 14. Laws grouped into an organized form 15. Fairness 16. Law dictated from above

20

Chapter 1 > Laws and Their Ethical Foundation

REVIEW LEGAL CONCEPTS 17. Louisiana is the only one of our 50 states whose law was not originally based on the English common law system. What system was its law based on? How did this system come to be used? 18. Why should a constitution be considered the highest law of a nation or a state? 19. What does the phrase “interstate commerce” mean to you? Would you consider growing vegetables in your garden for your own consumption interstate commerce? Why or why not?

20. Why does the doctrine of stare decisis not bind supreme courts? 21. Why are most fines the courts assess for criminal behavior paid to the government and not to the victim of the crime? 22. Explain how both rule-based ethical reasoning and consequences-based ethical reasoning conclude that people are obligated to obey the law. 23. Identify a situation in your life where someone used rule-based ethical reasoning. 24. Describe the three steps involved in making a decision using consequences-based ethics.

WRITE ABOUT LEGAL CONCEPTS 25. Would you rather be under the control of a system of common law or positive law? Write a paragraph to explain your answer. 26. Why is it important to have uniform state laws governing business? Write a paragraph to list and discuss at least four reasons. 27. Use one to three words to identify a current event that others will quickly recognize. Next, write a paragraph evaluating someone’s conduct in the current event using the ethics of reasoning based on consequences. 28. Use one to three words to identify a current event that others will quickly recognize. Next, write a paragraph evaluating someone’s

conduct in the current event using the ethics of rule-based reasoning. 29. Create and write a scenario which raises an ethical issue. Try to create a scenario where something is wrong in rule-based reasoning but right in reasoning based on consequences. 30. HOT DEBATE Clarisse is convicted of a violation of the noise ordinance. Her appeal is dismissed. As a consequence, she organizes a group of friends with cars to drive through the neighborhoods of the city with their stereos going at full volume as an act of civil disobedience. Is she ethically justified in so doing? Why or why not?

THINK CRITICALLY ABOUT EVIDENCE Study the situation, answer the questions, and then prepare arguments to support your answers. 34. Staub, Conly, and Winfield were employees of 31. You are in France and are accused of a crime. You the Prime Time Restaurant. They were aware that ask for a trial by jury. Why is one not available? the owner never checked the totals on the sales 32. Your friend Bill has shared with you his dream checks against the cash in the register. Therefore, of getting rid of the income tax. He says that it would be very easy to steal from the cash regif the people wanted to, they could pass a ister. However, they did not steal. When asked constitutional amendment that would elimiwhy, they gave the following reasons. Staub nate the tax. Is he correct? said he did not take the money because he was 33. Juana was trying to decide whom to vote for in afraid of being caught. Conly said she did not an upcoming election. After reviewing the cantake the money because she felt obligated to didates, she said, “I’ve decided to vote for Gary obey the law. Winfield said he did not take the because I just feel better about him.” Is Juana’s money because of his religious beliefs. To which decision based on ethics? If not, why? person do you best relate? Why?

Chapter 1 Assessment

21

MAKE ACADEMIC CONNECTIONS 35. MEDICINE As medical science moves forward, major ethical issues are developing that are likely to be encountered in the courts and legislative bodies. Identify such issues in medical fields such as cloning, cryogenic preservation,

organ marketing and farms, etc. Choose one of these issues, and apply both consequencesbased and rule-based ethical reasoning to determine whether it is ethical or not. Write a paragraph summarizing your findings.

ANALYZE REAL CASES 36. Alaska enacted a statute known as “Alaska Hire.” It required employers in the state to hire qualified Alaskan residents in preference to nonresidents. Hicklin, a nonresident, sued Orbeck, the state official charged with enforcing the statute. After the Supreme Court of Alaska found the statute constitutional, Hicklin appealed to the U.S. Supreme Court, which found the statute to be in conflict with the U.S. Constitution. Which supreme court is the final authority in this case? (Hicklin v. Orbeck, 437 U.S. 518) 37. The city of Chicago sued to stop the operation of the Commonwealth Edison Company’s coal-burning, electricity-generating plant in nearby Hammond, Indiana. Chicago claimed that the plant emitted too much smoke, sulfur dioxide, and other harmful substances. The city also claimed that the plant was a common-law public nuisance because it caused “an unreasonable interference with a right common to the general public” to clean air. Edison argued that it had spent much money to reduce harmful emissions and that the emissions were now well below the levels prescribed by federal clean air regulations and by the city of Hammond. Edison also pointed out that “unpleasant odors, smoke, and film” already characterized the area in which the plant was located. The trial court refused to issue an injunction. Therefore, the city of Chicago appealed to a higher court, which affirmed (upheld) the trial court. How can this legal action be ethically justified? (City of Chicago v. Commonwealth Edison Company, 321 N.E.2d 412, Ill.) 38. On March 30, 1981, as then President Ronald Reagan left the Washington Hilton Hotel after giving a speech, he was wounded by one of a series of bullets allegedly fired by John

22

Chapter 1 > Laws and Their Ethical Foundation

Hinckley. Three other men also were wounded, including James Brady, the President’s press secretary, who received a severe wound to the head. President Reagan underwent surgery shortly thereafter at the hospital. Hinckley was arrested at the scene literally with the smoking gun. Soon after the criminal process was begun against him, he underwent two psychiatric examinations by order of the magistrate and then the chief judge involved in the case. Both examinations found him to be competent. On August 28, after being indicted by a federal grand jury, Hinckley pled not guilty to a battery of charges, including the attempted assassination of the President of the United States. Hinckley’s lawyers then began their work. In two very important rulings they were able to suppress the use of evidence that would have been very damning to the defendant. They first had thrown out all answers he had given to questions prior to having his attorney present. This was done even though he had been read his Miranda rights three times before agreeing to answer those questions. Then Hinckley’s attorneys were able to suppress allegedly incriminating material in the form of a diary gathered by the jailers when they checked Hinckley’s cell for contraband and instruments useful in a potential suicide attempt. These rulings by the court were based on the constitutional rights to counsel and to protection against improper searches and seizures. Do you consider their use in this case a proper application of such rights? Is the exclusion of the evidence the only possible remedy, or could you come up with another one? If so, what is it, and why would it be an improvement over the current policy? (United States v. John W. Hinckley, Jr., 525 F. Supp. 1342)

Sports & Entertainment Law 2 Live Crew Raps Roy Orbison’s “Oh, Pretty Woman” A CUFF -R OSE M USIC I NC. V. C AMPBELL 972 F. 2D 1429 BACKGROUND The hit song for singer Roy Orbison in 1964, “Oh, Pretty Woman” also was the theme song for “Pretty Woman,” a hit movie three decades later. Orbison became famous for his rendition of the song. After eight more top-ten hits, he was inducted into the Rock and Roll Hall of Fame in 1987. Orbison died a short time later at age 52.

FACTS When “Pretty Woman” was recorded in parody form by 2 Live Crew in the early 1990s, the holder of the copyright, Acuff-Rose Music Inc., brought suit for infringement. 2 Live Crew had faced similar conflicts before. Manager-performer Luke, through his company Luke Records, previously had paid George Lucas $300,000 for using the name of Lucas’ Star Wars character Luke Skywalker. The U.S. Supreme Court, which already had overturned a Florida court’s declaration of another 2 Live Crew recording as obscene, was called upon to rule in the “Oh, Pretty Woman” case. Here are some comparative first-stanza lyrics:

© RUNN WOLFSON/TIME LIFE PICTURES/GETTY.IMAGES

Roy Orbison’s version Pretty Woman, walking down the street Pretty woman, the kind I like to meet Pretty woman

I don’t believe you, you’re not the truth No one could look as good as you 2 Live Crew’s version (Luke rapping, entire group’s words in parentheses) (Pretty woman) Ha haaa, walkin’ down the street (Pretty woman) Gir, girl, you look so sweet (Pretty woman) You, you bring me to the knees (Pretty Woman) You make me wanna beg please (O-o-o-o-oh, pretty woman)

THE LAW Section 107 of the Copyright Act allows the “fair use” without permission of copyrighted material for criticism, comment, news reporting, teaching (including classroom use in the form of multiple copies), scholarship, and research without being considered infringement.

THE ISSUE Does the publication of a parody for commercial gain fall within the protection afforded by Section 107?

HOLDING The Court held that 2 Live Crew’s version of the song was a ridiculing commentary on Orbison’s work and deserving of protection against the infringement charges. As long as the work was intended as a parody and not for advertising for the group or its products, the fact that it was a commercial success for 2 Live Crew does not bar the Crew from using the Section 107 defense.

PRACTICE JUDGING 1. Assess the Court’s holding from a consequences-based ethical standpoint. 2. Assess the Court’s holding from a rule-based ethical standpoint.

Sports & Entertainment Law

23

Chapter 2 Constitutional Rights

The original draft of the Declaration of Independence condemned the slave trade and criticized the English people. Both passages were edited out by the delegates of the Continental Congress before the Declaration was approved.

WHERE DO

YOU STAND?

1. Why do you think the passages were omitted? 2. Was omitting them a good or bad idea? Why?

24

Chapter 2 > Constitutional Rights

© GETTY IMAGES/PHOTODISC © GETTY IMAGES/PHOTODISC

2-1 Foundations of the U.S. Constitution 2-2 Division and Balance of Governmental Powers 2-3 The Internet and Constitutional Rights

2-1 Foundations of the U.S. Constitution GOALS

KEY TERMS

■ Identify the documents written in the course of

Declaration of Independence Articles of Confederation U.S. Constitution

the nation’s founding ■ Explain how the U.S. Constitution addresses and protects citizens’ civil rights

The Documents That Formed a Nation When the Constitutional Convention sent the proposed Constitution to the states for ratification, it met with strong opposition. Patrick Henry, James Monroe, and John Hanco*ck, for example, all opposed it. When the Constitution was put into effect in 1789, two colonies had yet to ratify it. The ratification did not become unanimous until 1790.

What's Your Verdict?

Declaration of Independence On July 4, 1776, delegates from the 13 original American colonies to the second Continental Congress, meeting in Philadelphia, formally adopted the Declaration of Independence. Thomas Jefferson of Virginia drafted the Declaration in less than a month. The document asserted the rights the colonists desired. It charged King George III of England with “a history of repeated injuries and usurpation” for the purpose of establishing an “absolute tyranny. . . .” It declared that “all men are created equal, that they are endowed by their Creator with certain unalienable rights, that among these are life, liberty, and the pursuit of happiness.” (See the full text of the Declaration of Independence in Appendix B on page 630.)

© NORTH WIND/NORTH WIND PICTURE ARCHIVES

What caused the opposition to the Constitution? What action resolved the dispute?

Bill of Rights civil rights due process of law

This image is a reproduction of a famous painting by John Trumbull entitled “Declaration of Independence.” The painting has been criticized for showing a scene that never took place. That is, according to historians, all 48 men shown in this picture were never gathered in the same place at the same time. Despite this criticism, do you think the picture is historically valuable? Why or why not?

2-1 Foundations of the U.S. Constitution

25

The authors of the Declaration of Independence believed that to secure these rights, they would need to have their own government. This new government would draw its power from the people. At the time the Declaration was adopted, fighting between British troops and colonists had already begun. The American War of Independence that followed lasted more than eight years, with the colonies claiming ultimate victory.

Articles of Confederation The 13 former colonies, each with its own independent form of government, united in 1781 under a charter called the Articles of Confederation. The Articles of Confederation contained the following provisions: ■ the government was to be conducted by a one-house legislature (the “United States in Congress”) with two to seven representatives from each state; each state, however, had only one vote on issues before the Congress ■ strict term limits (no person could serve more than three years in a 6-year period) placed on members of Congress, who were subject to recall by their states

ORDER OF RATIFICATION OF THE U.S. CONSTITUTION 1. Delaware

December 7, 1787

2. Pennsylvania

December 12, 1787

3. New Jersey

December 18, 1787

4. Georgia

January 2, 1788

5. Connecticut

January 9, 1788

6. Massachusetts

February 6, 1788

7. Maryland

April 28, 1788

8. South Carolina

May 23, 1788

9. New Hampshire

June 21, 1788

10. Virginia

June 26, 1788

11. New York

July 26, 1788

12. North Carolina

November 21, 1789

13. Rhode Island

May 29, 1790

26

the legislature had only the power to declare war, make peace, enter into treaties and alliances, manage relations with Indian nations, coin money, settle differences between states, establish a postal system, and appoint a Commander in Chief; all other powers were reserved to the states ■ paying for the common defense or general welfare came from a common treasury funded by the states in proportion to the value of all land within each state ■ paupers, vagabonds, fugitives from justice, and slaves were not entitled to the privileges and immunities of free citizens of the states ■ major legislation, including bills relating to finance, would require a two-thirds vote for passage ■ amendments to the Articles would require a unanimous vote of the states Many people felt a need for a stronger central government than the one the Articles provided. Disagreement among the states regarding the Articles ultimately led to the calling of a special convention of delegates from the original 13 states.

THE ORIGINAL 13 COLONIES

DE PA NJ GA CT MA MD SC NH VA NY NC RI

Chapter 2 > Constitutional Rights Copyright 2009 Cengage Learning, Inc. All Rights Reserved. May not be copied, scanned, or duplicated, in whole or in part.

U.S. Constitution The special convention, held in Philadelphia in the summer of 1787, drafted the initial U.S. Constitution. The seven articles of the U.S. Constitution provided a workable framework for a federal government. By June 21, 1788, the Constitution had been ratified by delegates to special conventions held in nine of the original 13 colonies. (See the full text of the U.S. Constitution in Appendix A on page 618.) As specified in Article VII of the Constitution, ratification by at least nine prospective states was enough to put the Constitution in force. Consequently, March 4, 1789, was selected as the date the government of the United States of America was to begin operation under the new Constitution. Ratification took so long to be approved in all the states because critics felt the wording of the Constitution failed to provide adequate protection of the human rights proclaimed in the Declaration of Independence. In What’s Your Verdict? North Carolina and Rhode Island took longer to ratify because they felt this fundamental problem with the Constitution— the failure to protect human rights—had to be corrected. The situation was resolved by the adoption of the Bill of Rights.

The Bill of Rights The first ten amendments to the Constitution, known as the Bill of Rights, were adopted to ensure that U.S. citizens would enjoy the human rights proclaimed in the Declaration of Independence. Three documents that helped secure the rights of individuals in England contributed greatly to our own Bill of Rights. These were: The Magna Carta, the Petition of Right of 1628, and the Bill of Rights. The amendments in the U.S Bill of Rights shown below are in the words of the founding fathers. AMENDMENT I Congress shall make no law

respecting an establishment of religion, or prohibiting the free exercise thereof; or abridging the freedom of speech, or of the press, or the right of the people peaceably to assemble, and to petition the Government for a redress of grievances. AMENDMENT II A well regulated Militia, being necessary to the security of a free State, the right of the people to keep and bear Arms, shall not be infringed.

AMENDMENT III No Soldier shall, in time of peace be quartered in any house, without the consent of the Owner, nor in time of war, but in a manner to be prescribed by law. AMENDMENT IV The right of the people to be

secure in their persons, houses, papers, and effects, against unreasonable searches and seizures, shall not be violated, and no warrants shall issue, but upon probable cause, supported by oath or affirmation, and particularly describing the place to be searched, and the persons or things to be seized. AMENDMENT V No person shall be held to answer

for a capital, or otherwise infamous crime, unless on a presentment or indictment of a Grand Jury, except in cases arising in the land or naval forces, or in the Militia, when in actual service in time of War or public danger; nor shall any person be subject for the same offense to be twice put in jeopardy of life or limb, nor shall be compelled in any criminal case to be a witness against himself, nor be deprived of life, liberty, or property, without due process of law; nor shall private property be taken for public use without just compensation. AMENDMENT VI In all criminal prosecutions, the accused shall enjoy the right to a speedy and public trial, by an impartial jury of the State and district wherein the crime shall have been committed, which district shall have been previously ascertained by law, and to be informed of the nature and cause of the accusation; to be confronted with the witnesses against him; to have compulsory process for obtaining witnesses in his favor, and to have the assistance of counsel for his defense. AMENDMENT VII In Suits at common law, where the value in controversy shall exceed twenty dollars, the right of trial by jury shall be preserved, and no fact tried by a jury shall be otherwise re-examined in any Court of the United States, than according to the rules of the common law. AMENDMENT VIII Excessive bail shall not be

required, nor excessive fines imposed, nor cruel and unusual punishments inflicted. AMENDMENT IX The enumeration in the

Constitution of certain rights shall not be construed to deny or disparage others retained by the people. AMENDMENT X The powers not delegated to the

United States by the Constitution, nor prohibited by it to the States, are reserved to the States respectively, or to the people. 2-1 Foundations of the U.S. Constitution

Copyright 2009 Cengage Learning, Inc. All Rights Reserved. May not be copied, scanned, or duplicated, in whole or in part.

27

CHECKPOINt What English documents

helped form the basis for the first ten amendments to the U.S. constitution?

Civil Rights One afternoon, during a class break, Bill and Santos were talking. Bill complained that his parents had been monitoring the web sites he had visited on his computer. “I can’t believe they can do that. What about my civil right of privacy?” Santos replied, “I went through the same thing with my parents. When I complained about it, my dad said there was no mention of a right of privacy in the Constitution.”

© GETTY IMAGES/PHOTODISC

What's Your Verdict?

Was Santos’ father correct?

Through the Bill of Rights and other amendments that were to follow, the Constitution became a shield for the personal, natural rights of the individual. These rights are referred to as civil rights. The most vital of these rights are those protected by the First Amendment: freedom of speech, freedom of the press, and freedom to assemble. The First Amendment also protects the people’s rights to practice the religion of their choice. Religion has inspired men, women, and children to noble and caring conduct. However, misguided religious zeal also has caused great violence and destruction. The founders of the republic recognized this deep-seated religious impulse but wisely imposed a role of neutrality for the government. They created a wall of separation between church and state. The government tolerates all religions but supports none in any strictly religious efforts.

Due Process of Law Another significant amendment in the Bill of Rights is the Fifth Amendment. It states, “No person shall be . . . deprived of life, liberty, or property, without due process of law . . .” Unlike the Declaration of Independence, the Constitution does not mention “the pursuit of happiness.” However, respect by others for one’s life, liberty, and property surely helps one to be secure and content, if not consciously happy. 28

Chapter 2 > Constitutional Rights

The First Amendment protects your right to practice the religion of your choice. If the founding fathers believed that freedom of religion was important, why do you think they opted for separation between church and state?

The Constitution does not spell out what is meant by due process. The term has, however, come to be defined through various U.S. Supreme Court decisions. At the present time due process of law is taken to mean that, at a minimum, a defendant over whom jurisdiction has been established must be provided adequate notice and a proper hearing. Due process of law requires fundamental fairness in compliance with reasonable and just laws. If convicted, the criminal may be deprived of property by fine, of liberty by imprisonment, and even of life by lawful execution. Due process of law is a concept embodied throughout the Constitution. It is assured by guaranteeing ■

the right to be secure against unreasonable searches and seizures (Fourth Amendment) ■ the right not to be a witness against yourself in any criminal case (Fifth Amendment) ■ in criminal prosecutions, the right for the accused to a speedy and public trial by an impartial jury of the state and district where the crime was committed, and to be informed of the nature and cause of the accusation;

the right to be confronted with the witnesses against him; to have a set process for obtaining witnesses in his favor, and to have the benefit of a defense counsel (Sixth Amendment) ■ trial by jury in civil suits where the value in controversy exceeds $20 (Seventh Amendment) ■ that excessive bail will not be required nor excessive fines imposed, nor cruel and unusual punishment inflicted (Eighth Amendment)

The Abolition of Slavery Originally the practice of slavery was enabled under the Constitution. In particular, the Constitution provided that Congress could not prohibit, prior to 1808, the “migration or importation of such persons. . . .” Slaves were further dehumanized by a provision in Article I that counted them as only three-fifths of a person for the purpose of apportioning representatives to Congress. Under Article IV, a slave who escaped to another state had to be returned to “the party to whom such service or labor may be due.” Finally, as a consequence of the Civil War, these provisions were negated by the Thirteenth Amendment, which states in part, “Neither slavery nor involuntary servitude, except as a punishment for a crime whereof the party shall have been duly convicted, shall exist within the United States, or any place subject to their jurisdiction.”

The Right to Vote The Fifteenth Amendment provided the newly freed slaves the legal right to vote. It decreed, “The right of citizens of the United States to vote shall not be denied or abridged by the United States or by any State on account of race, color or previous condition of servitude.” Although the Thirteenth Amendment abolished slavery, and the Fifteenth guaranteed the former slaves the right to vote, racial discrimination continued in both the North and the South. Some southern states enacted a poll tax. This was a fixed payment per person required before the person could vote. Many former slaves could not afford to pay a tax in order to vote. In 1964, more than 100 years after the Civil War began, the Twenty-Fourth Amendment guaranteed that the right to vote in federal elections “shall not be denied or abridged . . . by reason of failure to pay any poll tax or other tax.” The Supreme Court subsequently applied the same rule to all state elections, declaring such

T

he Wyoming territorial legislature granted women the right to vote 50 years before Congress passed the Nineteenth Amendment. The Wyoming law was enacted on December 10, 1869, in an attempt to attract female settlers.

taxes unconstitutional under the Fourteenth Amendment. DENIAL BY GENDER The exclusion of “gender” in the Fifteenth Amendment was a deliberate denial of voting rights to women of all races. This injustice was removed in 1920 with the passage of the Nineteenth Amendment. It provided “The right of citizens of the United States to vote shall not be denied or abridged by the United States or by any State on account of sex.” DENIAL BY AGE Students protested during the Vietnam War, in part, because they objected to being eligible to fight in a war at the age of 18— but not vote until age 21. One result of these student protests was the hasty enactment of an amendment that gave all citizens age 18 or older the right to vote. The Twenty-Sixth Amendment was approved overwhelmingly by the Senate and House in March 1971. It was ratified by 38 states within three months.

T

he Emancipation Proclamation by President Abraham Lincoln is looked upon as a landmark in the struggle against slavery. Many historians, however, see it purely as a political ploy. They point out that it actually freed only a handful of individuals, if any at all. By its wording it did not apply in the border states where slaves were held but who were fighting on the union side. Nor did it apply in the areas of the Confederacy held by union forces. Of course, the Confederate states did not act in compliance with it. Do you think that Lincoln acted ethically by limiting his proclamation to a strictly political position? Why or why not?

2-1 Foundations of the U.S. Constitution

29

Peripheral Rights Some rights are not explicitly mentioned in the U.S. Constitution or Bill of Rights, but courts have recognized them as necessary to protecting the rights that are mentioned. The right of privacy is such a peripheral right—it is not specifically mentioned in the Constitution or Bill of Rights. In the 1960s the U.S. Supreme Court acknowledged the right of privacy as a separate right. The right of privacy has since been reinforced by several federal acts.

In What’s Your Verdict? even though Santos’ father was technically correct, the civil right of privacy does exist. However, concurrent with that right is the right of parents to control the health and welfare of their children. Therefore, such monitoring of web-site visits of underage children is legally permissible. CHECKPOINt How does the Constitution

address and protect citizens’ civil rights?

2-1 Assessment school.cengage.com/blaw/lawxtra

THINK ABOUT LEGAL CONCEPTS 1. The Declaration of Independence was adopted on (a) July 4, 1776 (b) July 4, 1789 (c) July 4, 1861 2. The “inalienable rights” mentioned in the Declaration of Independence are “life, liberty, and the pursuit of _ ?_.” 3. The American War of Independence lasted about (a) two years (b) four years (c) eight years (d) ten years 4. The 13 colonies originally created a common government under the “Articles of Incorporation.” True or False? 5. Opponents of the ratification of the original Constitution claimed the wording failed to protect human _?_.

6. _ ?_ was the thirteenth state to ratify the U.S. Constitution. 7. Which of the first 10 amendments guarantees freedom of the press? (a) First (b) Second (c) Third (d) freedom of the press is not guaranteed in the Bill of Rights 8. The original Bill of Rights contains the right of peaceful assembly. True or False? 9. Paupers, vagabonds, fugitives from justice, and slaves were not entitled to the privileges and immunities of free citizens under the Articles of Confederation. True or False? 10. The right of privacy was not acknowledged as a separate right until the 1960s. True or False?

THINK CRITICALLY ABOUT EVIDENCE Study the situation, answer the questions, and then prepare arguments to support your answers. 11. It is 1793 and you are a citizen and editor of a newspaper in one of the original states. Your newspaper regularly carries stories about corruption in state government. Finally, the state prosecuting attorney brings charges against you for criticism of government officials, a violation of state law, and closes your newspaper. You maintain that your freedom of the press under the U.S. Constitution has been violated. The prosecutor replies that the Bill of Rights does not protect you from state actions. Is the prosecutor correct? 12. You are a citizen of one of the 13 colonies. The time is mid-July 1776. You have just heard of the

30

Chapter 2 > Constitutional Rights

issuance of the Declaration of Independence. What reasons might cause you to withhold your support of the Declaration? 13. As a protest against overseas military action by the government, about 600 sign-carrying students from a local school gathered peacefully at a major road intersection. As curious drivers slowed to observe, traffic came to a standstill. Within minutes, the police moved in and forced the crowd to break up. Several individuals who dropped their signs and “went limp” on the pavement were arrested. Did the police action violate the protesters’ First Amendment constitutional right “peaceably to assemble”?

2-2 Division and Balance of Governmental Powers GOALS

KEY TERMS

■ Describe the system of checks and balances in

system of checks and balances impeachment cases political party amendment democracy

the U.S. government ■ Explain how the power to govern is divided between the federal and state governments

A System of Checks and Balances

they enjoyed under the Articles of Confederation. States with large populations feared they would be dominated by the less populous states (of which there were many more), if each state had an equal number of votes. The solution? The national legislature would consist of two bodies:

Thomas was critical of the U.S. Constitution because he said it had not been ratified by a democratic vote of the people. Shakira said in response that of course it had been.

What's Your Verdict?

1. a Senate, with two members from every state—regardless of population, and 2. a House of Representatives, with seats allocated to the states in proportion to their population.

Who is correct?

Warren E. Burger, former Chief Justice of the U.S. Supreme Court, has pointed out that at the time the Constitution was drafted and adopted, “There was not a country in the world that governed with separated and divided powers providing checks and balances on the exercise of authority by those who governed.” The 55 delegates who drafted the U.S. Constitution displayed great foresight in devising a unique system of checks and balances. This system is based upon the creation of a balance of power between governmental branches by having the powers given one branch check the powers given another.

Branches of Government The three branches of the U.S. government are the legislative, the executive, and the judicial. LEGISLATIVE BRANCH The legislative branch

makes the laws. At the federal level, the heart of the legislative branch is the Congress. When the Constitution was drafted, the convention delegates were concerned with equal representation in Congress. States with small populations worried they might lose some of the independence

republic sovereignty interstate commerce intrastate commerce

The existence of two bodies allows for various checks and balances. For instance, all bills for taxing or appropriating funds must originate in the House. However, a majority vote of both bodies is required for passage of any bill, including tax and appropriations bills. Even the checks the legislative branch can exercise on the abuse of power by the executive or judicial branches are divided between the two houses of Congress. For example, the House has the power to impeach or accuse any civil officer of the United States—including the President and Vice President—of treason, bribery, or other high crimes and misdemeanors. However, the Senate has the sole power to try all impeachment cases. Impeachment cases are criminal proceedings against a government official for misconduct in office. Conviction in such cases requires a twothirds vote of the Senate members present. Finally, the people may vote for their representatives in elections to the House (for a two-year term), the Senate (for a six-year term), and the executive office (for a four-year term). EXECUTIVE BRANCH The executive branch is

headed by the President and the Vice President. These officers are elected by a vote of the people. 2-2 Division and Balance of Governmental Powers

31

In the electoral process, citizens vote for the electors who are pledged to support candidates selected by political parties. Because the President and the Vice President are not elected directly by the people, the candidate who receives the greatest popular vote for president may not always win the office. This has occurred four times, in 1824, 1876, 1888, and 2000. A political party is a private organization of citizens who select and promote candidates for election to public office. Party members agree with these candidates on important governmental policies and legislation. Political parties are not mentioned in the Constitution. JUDICIAL BRANCH The third branch of

government is the judiciary. It is headed by the Supreme Court of the United States. Ultimately, the Supreme Court decides on the constitutionality of a statute passed by the legislative branch and signed by the President as head of the executive branch. The Supreme Court also may decide if a particular action or decision exceeds the powers granted to the executive branch or legislative branch under the Constitution. If so, the action or decision is void.

requires the legislatures of two-thirds of all the states to call a convention of all the states. The convention may propose one or more amendments. Under either method of proposal, the amendment becomes a valid part of the U.S. Constitution only if it is ratified by the legislatures of three-fourths of the states, or if it is ratified by conventions in three-fourths of the states.

The U.S. Form of Government In a pure democracy, every adult citizen may vote on all issues. This is almost impossible to make happen in a nation of more than 300 millionpeople. Instead, the United States is a republic or a representative democracy. Voters select their representatives to the legislative, executive, and judicial branches of government. Although representatives make the day-to-day decisions, the sovereign power ultimately resides in the people. This means that the highest final authority to decide what the law shall be rests with the citizens who exercise their right and duty to vote. In many elections only a minority of the total number of voters who are eligible actually exercise this right.

Changing the Constitution A major check is provided by the power of constitutional amendment meaning to change or alter. The Constitution may be amended in two ways. The first way has been used for all amendments adopted to date. The amendment is proposed by a two-thirds majority vote in both the Senate and the House. The second way

Access school.cengage.com/blaw/lawxtra and click on the link for Chapter 2. Check out the “failed amendments”—amendments that never received the level of support needed to ratify them—or amendments recently proposed by Congress. Choose one of the failed or proposed amendments. Write a paragraph describing the amendment, and then explain whether you agree or disagree with it.

school.cengage.com/blaw/lawxtra

32

Chapter 2 > Constitutional Rights

Democratic Representation in Hungary The history of democratic representation in Hungary is long and complicated. For nearly a thousand years, the only form of representation was an assembly of nobles called the Diet which met infrequently at different places around the country. Finally, due to the reforms introduced in the nineteenth century, a parliament was formed. Unfortunately, the parliament was relatively weak, and Hungary was dominated by outside powers. In 1867, the parliament gained considerable powers when the Austrian emperor Franz Joseph agreed that Hungary should be administered as a separate nation within the empire. The parliament continued into the twentieth century as a semi-representative body. After the collapse of Communism in 1989, Hungary gained its first parliament that truly represented all Hungarians.

In What’s Your Verdict? Thomas is correct. The Constitution was proposed, ratified, and has since been amended only by representatives of the people in Congress, in conventions, and in the state legislatures. There has never been a purely democratic vote on the Constitution.

to the United States by the Constitution, nor prohibited by it to the States, are reserved to the States respectively, or to the people.” Here it is evident that the Constitution is a shield against the consolidation of unlimited power by the federal government. Article VI recognizes the Constitution to be “the supreme law of the land.” It prevails over any possible contrary state constitution or law. At the same time, every state’s constitution and statutes are supreme on matters over which the federal government has not been given authority. Powers kept by the states include control over most business law and contract law, most criminal and tort law, real property and probate law, and domestic relations law.

CHECKPOINt How did the Constitution

create a system of checks and balances?

The Power to Govern Several thousand people had assembled in the park for the annual Fourth of July celebration. The state’s senior U.S. senator opened the ceremonies. He said, “I am proud and pleased to be a citizen of the United States of America and of this great state. But, before I am a citizen of either state or country, I, like all of you, am a human being. Our humanity precedes our citizenship. Government exists only by the will and consent of the people. The people control the government. Any control by the government of the people occurs because the people have granted permission for that control.”

What's Your Verdict?

Limiting States’ Powers

The Constitution and the Bill of Rights were written by representatives of the people elected by voters in the 13 original states. These documents place a priority on the basic rights of human beings. This priority exists whether people act as individuals or together. The Ninth Amendment recognizes this priority by stating, “The enumeration in the Constitution of certain rights, shall not be construed to deny or disparage others retained by the people.”

The Fourteenth Amendment opens with these sweeping provisions: “All persons born or naturalized in the United States and subject to the jurisdiction thereof, are citizens of the United States and of the state wherein they reside. No state shall make or enforce any law which shall abridge the privileges or immunities of citizens of the United States; nor shall any State deprive any person of life, liberty, or property, without due process of law; nor deny to any person within its jurisdiction the equal protection of the laws.” Recall that the Fifth Amendment had previously applied such limitations only to the powers of the federal government. The Fourteenth Amendment subjects the states to the same restraint. Due process, as outlined in the Fifth Amendment, is not the only constitutional protection that state governments are required to respect. Other relevant amendments in the Bill of Rights also have been applied to the states. As a result, today the shield of the Constitution is effective against abuse of power by both the federal and state governments. At both levels, statutes have been enacted to provide comparable protection against abuse of power by private individuals and corporations.

Sovereignty of the States

Powers of the Federal Government

The Tenth Amendment acknowledges the continued sovereignty of all of the states to govern their own citizens within their own borders. Sovereignty in this sense means supreme political authority free from external control. The Tenth Amendment declares, “The powers not delegated

The federal government has the duty to protect every state against invasion. It may raise and support armies, a navy, and an air force for national defense. The federal government alone may establish post offices, coin money, and tax imports and exports. Both federal and state legislatures may impose other taxes—on sales and

Does the senator’s analysis conform to the governmental structure created by the framers of the Constitution?

2-2 Division and Balance of Governmental Powers

33

SELECTED FEDERAL REGULATORY AGENCIES AND THEIR POWERS Federal Reserve System’s Board of Governors Determines economic policies involving the money supply, credit availability, and interest rates. Federal Trade Commission (FTC) Assists the antitrust division of the Justice Department. Enforces laws against anti-competitive business activities. It also is involved in protecting consumer rights, preventing monopolistic behavior, and eliminating unfair and deceptive trade practices. Securities and Exchange Commission (SEC) Enforces laws regulating the disclosure of information related to the buying and selling of stocks and bonds. It also regulates the stock exchanges themselves. National Labor Relations Board (NLRB) Charged with preserving employees’ rights to join labor unions and to participate in collective bargaining. It also is charged with eliminating employer, employee, and union unfair labor practices. Federal Communications Commission (FCC) Controls all interstate channels of communication and channels of communication between the United States and foreign nations. This includes satellite, telegraph, telephone, radio, and television forms of communication. Equal Employment Opportunity Commission (EEOC) Charged with the elimination of workplace discrimination based on race, religion, sex, color, national origin, age, or disability. Occupational Safety and Health Administration (OSHA) Governs health and safety in the workplace. Consumer Product Safety Commission (CPSC) Researches the safety of various consumer products, including toys. It also collects data on mishaps with products and has banned various products including toys, baby cribs, and fireworks. Food and Drug Administration (FDA) Enforces a number of pure food and drug acts. Tests and reviews any new drug before it can be marketed to the American public. The FDA also conducts food product inspections and regulates the availability and use of medical devices such as prosthetics and pacemakers. Environmental Protection Agency (EPA) Enforces antipollution acts (the Clean Air and Clean Water Acts, for example) and regulates the creation, marketing, and use of various hazardous chemicals. Also handles cleanups of toxic dumpsites around the country. Nuclear Regulatory Commission (NRC) Charged with insuring the safety of nuclear power plants in the United States. Department of Homeland Security (DHS) Controls an agency formed by the merger of 22 separate agencies into one with the mission of protecting the American homeland from terrorist attack.

on incomes. The federal government has exclusive power to regulate interstate commerce, or commerce that affects trade between states, and foreign commerce. Each state retains authority to regulate intrastate commerce, or trade within its own borders. As a result of the nationwide, or interstate, impact of the Great Depression in the 1930s, the federal government assumed more and more power in the regulation of business. Today this power is exercised through its many agencies. The figure above lists and describes some of the major federal agencies. When a federal agency properly makes regulations to control its 34

Chapter 2 > Constitutional Rights

commercial area of responsibility, those regulations have the force of federal law. The states often have similar agencies with jurisdiction only within their boundaries. Congress has the power to make detailed laws it considers appropriate for executing the powers given to it in the Constitution. The people, through their votes, ultimately control the entire governmental structure. Each adult retains power over choice of a place to live and to work, a career, friends, travel, holiday activities, and many other personal lifestyle factors. In What’s Your Verdict? the senator’s comments do accurately reflect the reality of the relationship

between citizens and the government today. However, even after the Bill of Rights was added to the Constitution, certain deficiencies existed in the Constitution and in the resulting life of the nation. Additional amendments have been ratified through the years to correct these problems.

CHECKPOINt How is the power to

govern divided between the federal and state governments?

2-2 Assessment school.cengage.com/blaw/lawxtra

THINK ABOUT LEGAL CONCEPTS 1. Under the U.S. Constitution, all spending bills must originate in the _ ?_. 2. Which of the following powers was not retained by the states? (a) power to define and apply tort (personal injury) law (b) power to define and apply criminal laws (c) power to define and apply probate laws (d) the states retained all of the above powers 3. Individual states in the United States can coin money. True or False? 4. According to the Constitution, the legislative powers of the federal government are vested in the __?__ of the United States.

5. The U.S. Senate is made up of _ ?_ members from every state. 6. The U.S. House of Representatives is made up of the same number of members from each state. True or False? 7. The U.S. Senate has the power to initiate impeachment proceedings against a U.S. President. True or False? 8. The _ ?_ branch of government is headed by the President.

THINK CRITICALLY ABOUT EVIDENCE Study the situation, answer the questions, and then prepare arguments to support your answers. 9. You are a member of the legislature of a large Midwestern state. Your state is running short of money to carry out some much needed programs. As a possible solution you suggest that the state government issue its own currency to people who work for it. The currency can be exchanged for dollar bills at a rate that is to be fixed by the state the first of every month. Is your idea constitutional? 10. Later in the legislative session mentioned in exercise 9, you become disenchanted with your fellow citizens when you learn that only 28 percent of those eligible to vote actually did so in the last election. Consequently, you pass a law requiring that everyone vote in every election.

What arguments can you make in support of such a measure? Against? 11. The Equal Rights Amendment (ERA) was formally proposed in 1972. It provided: “Equality of rights under the law shall not be denied or abridged by the United States or by any state on account of sex.” For more than ten years, supporters and opponents conducted campaigns in all states. When the time limit for approval elapsed in 1982, only 35 of the required state legislatures had voted in favor. Women in a sorority at a nearby college now wonder if another campaign should be launched to pass the ERA. How would you advise them?

2-2 Division and Balance of Governmental Powers

35

2-3 The Internet and Constitutional Rights GOALS

KEY TERMS

■ Understand the issues of jurisdiction that arise

cyberlaw

browser

with Internet usage ■ Identify the various constitutional issues involved in the use of electronic communication

Who Controls the Internet? Sharksinthepool.biz Inc. is a Nevada manufacturing company that sells an elbow-stiffening device for pool players. The device holds the elbow in the proper position for a smooth and accurate follow through while cueing a ball. The device has been very popular for several years and has only been sold on the Internet to customers throughout the United States as well as in Japan, Canada, Mexico, Europe, and South America. Recent research into the use of this device has found that even moderate use may cause a pinched-nerve syndrome somewhat like carpel tunnel but in the user’s elbow. You are the founder and president of the company. You are extremely concerned about the lawsuits that may be filed against your company.

What's Your Verdict?

Often the opinions in these cases do not fully decide the issues, but clarify them. The judges in the cases leave it to Congress, the state legislatures, and federal and state administrative agencies to actually create the lasting cyberlaw. An example can be found in the complex area of jurisdiction, where states have competing laws to determine where a case is to be tried. The courts often are faced with the choice of either duplicating the work of other courts in other jurisdictions or accepting the result determined in the other courts without adequate review.

The Birth of the Internet The U.S. Department of Defense laid the groundwork for the Internet in the late 1960s during the so-called Cold War. It did so by having its

Where may these suits be filed? What jurisdiction’s law will be used to decide these cases once filed?

Literally right in front of your noses an entire new area of law is being formed. Cyberlaw—law that is intended to govern the use of computers in ecommerce and, more generally, the Internet—is developing at a previously unheard of pace. Legal issues born in this advancing wave of technology are testing the courts and established law to the maximum. The cases in which these issues are developed involve the computer and subject areas such as jurisdiction, trademarks, copyrights, contracts, privacy, obscenity, defamation, security of data, and crime. 36

Chapter 2 > Constitutional Rights

© GETTY IMAGES/PHOTODISC

Foundations of Cyberlaw

One of the most difficult issues in cyberlaw is determining jurisdiction, or where a case will be tried. Based on what you know about the Internet, why do you think this is so difficult?

Advanced Research Projects Agency (ARPA) tie together computers on various academic campuses and research centers around the country in a network called ARPANET. The idea was to increase the pace of development of defense projects by empowering the sharing of defense-related information between researchers through the use of electronic mail, file transfers, and online chats. The success of the idea was immediate and, as a result, this defense network was expanded and even connected to overseas networks. Soon academics outside of the defense establishment began to take notice of the usefulness of the medium. As a consequence, in the late 1980s the National Science Foundation (NSF) built its own network. This more advanced NSF network—or Internet—took over the functions of the defense network and continued to grow and gain in popularity among the academics and students who had the expertise to use it. Eventually, commercial interests began to inquire about the Internet. Their requests to use it for business purposes, however, were turned down. This was primarily because the NSF Internet, like the defense network before it, was never intended to serve a commercial purpose. In fact, until 1991, all the users of the NSF network had to agree to a contract that explicitly disallowed the use of the Internet for business purposes.

plished through the use of a scanning program called a browser. A browser utilizes the attributes of hypertext to allow immediate access to any and all subjects and the background information necessary to comprehend them. To quote Berners-Lee, “there is power in arranging information in an unconstrained, web-like way.” To that end, he created hypertext markup language (HTML) and hypertext transfer protocol (HTTP). HTML dictates the format of standard Web documents, while HTTP provides the standard for exchanging files in the HTML format. Note that in this web-like arrangement of information, there are a multitude of connecting points and, therefore, a multitude of paths and storehouses for information. Shut down one path or storehouse and many more spring up in its place. The very decentralization of the Internet makes it nearly impossible to exercise legal control over it and what goes on within it.

Evolving Legal Issues in Cyberspace Cyberlaw is developing in many separate areas at once. Contracts, privacy, obscenity, defamation, security of data, trademarks, copyrights, and crime all have been the focus of many decisions intended

In 1990, after 20 years of scientific and academic use, the Internet was opened up to new users through the creation of hypertext. Hypertext is a nonlinear format for the publishing of information used to send e-mails, transfer mountains of data, and converse in chat rooms. These new users were seeking specialized information in subject areas in which they did not have a great deal of background. The access these new users desired was developed chiefly through the work of one person, Tim Berners-Lee. Working at the European Laboratory for Particle Physics, Berners-Lee developed the World Wide Web (WWW) Program mainly in his spare time. This program’s effect was to simplify the Internet to allow almost universal access to anyone with the use of a computer. Berners-Lee, who has not earned a profit from his development, is currently director of the World Wide Web Consortium, the coordinating body for Web development. His vision is that of open access to as wide a body of information as possible for every user. This access is accom-

© GETTY IMAGES/PHOTODISC

The World Wide Web

Information on the Internet is arranged in a web-like fashion. How does this affect government’s ability to exercise legal control over it?

2-3 The Internet and Constitutional Rights

37

to resolve the legal issues raised by computer usage and the Internet. These issues will be explored in the remainder of this lesson and in the Cyberlaw features in the chapters that follow. THE ONGOING ISSUE OF JURISDICTION Final answers to the legal questions in all of the areas mentioned above can only be arrived at after the court responsible for giving them is determined. Traditionally, the state courts have been given jurisdiction over cases between their citizens. Cases with relatively large amounts of money at stake and between citizens of different states, along with cases that involve questions of federal law, have been left to the federal courts to decide. (See Lessons 3-2 and 3-3 in the text for more information on this topic.) Regardless of whether or not a case is filed in state or federal court, however, the vital issue soon becomes whether or not the case is to be decided within the state where it was filed or in the state where the e-business is physically located. For example, would a corporation physically located in New Jersey and incorporated there have to fund a defense against a case filed against it in California? The answer to this question of jurisdiction is based on two vital legal considerations. First of all, the laws of the state in which the case is filed must allow suits against non-resident defendants. Most states do have statutes—referred to as “longarm statutes” for the long arm of the law—that, for the good of their citizens, permit such cases to be heard. However, the standards the state’s longarm statute sets for the involvement of the nonresident defendant in the state must be met or the suit will be dismissed for lack of jurisdiction. These standards vary from state to state. Some standards require a concrete presence in the state in the form of an office, a traveling salesperson, a mailing address, etc. Others simply require a reasonable presumption that the defendant is doing business in the state by its level of involvement there. The second legal consideration relating to jurisdiction is whether or not allowing the state or federal court (the latter using the state rules) to decide the case would be a violation of the due process requirements of the U.S. Constitution. As discussed earlier in this chapter, these requirements are embedded in the Fifth and Fourteenth Amendments to the Constitution. The Fifth Amendment, originally meant to apply to the actions of the federal government, prohibits the taking of life, liberty, or property without the

38

Chapter 2 > Constitutional Rights

due process of law. The Fourteenth Amendment extends this same prohibition to the states. According to the latest decisions regarding cyberlaw issues, the establishment of jurisdiction by a state over a nonresident defendant is only proper if the defendant has either a “substantial, continuous, and systemic presence” in the state or has “minimum contacts” with the state. Most e-businesses lack the former and, therefore, are judged by the minimum contacts rule. Soliciting business from a web site available in the state generally has been held to be a minimum contact. Accordingly, it is reasonable to presume that a lawsuit (over contracts, product liability, warranty, etc.) might be generated as a result of such activity. Simply providing an informative web site has been held not to be a minimum contact, however. In other words a car company merely providing pictures and characteristics of its cars on its web site but not providing a means by which one can be purchased through the web site itself would be free of the threat of suit based strictly on it as a “minimum contact.” There must be something more to indicate that the defendant intended to serve the market in the state. In What’s Your Verdict? Sharksinthepool.biz Inc. most likely would have to defend against the product liability suits wherever they were filed in the United States. Should they not defend and suffer a default judgment or defend and lose, the judgment may be entered and enforced against them in a Nevada court without substantial review by the Nevada court. INTERNATIONAL JURISDICTION International

cases are governed by international law, which is currently under serious revision due primarily to e-commerce. An established, pre-e-commerce principle is that a country may exercise jurisdiction

Imagine that you are a member of the governing body of the Internet. Help to set up a system of rules and a procedure for enforcement to prevent the appearance of web sites that you consider unethical. These may include sites that contain instructions on how to make bombs or how to assist or commit crimes. Research and appraise the rules that are currently in place for this purpose.

Internet-Related Constitutional Issues A respected Internet news service carried on its web site a picture of the U.S. Secretary of State provided by the Secretary’s publicity staff. The picture depicted the Secretary of State shaking hands in front of a foreign embassy next to a statue of a nude woman. The web site does not have the controls required to ensure that minors do not have access to it. In fact, it promotes itself as a source of information for school-age children.

© GETTY IMAGES/PHOTODISC

What's Your Verdict?

Would the news service be in violation of federal statutes against online p*rnography accessible by minors? Would the Secretary of State or the Secretary’s office be in violation of the statute? What action can an e-business take to remove the uncertainty involved in both international and domestic jurisdiction issues?

only over activities with a substantial effect on or in that country. However, new standards under consideration would impose a country-of-origination selection principle on business-to-business transactions and a country-of-residence-of-the-consumer principle on consumer transactions. The reviewability of decisions made overseas and then entered in the domestic courts of the United States for enforcement currently is covered mainly by treaties between individual countries. Most of the uncertainty involved in the jurisdiction issue can be removed if the e-business puts a notice on its web site and clarifying section in its contracts. This notice or clause should state, in effect, that all legal issues shall be resolved in a particular jurisdiction (hopefully its home state) with favorable laws. If the laws of a particular jurisdiction do not allow such limits on suits, then the e-business should not do business in that jurisdiction. CHECKPOINt What characteristic of the

Freedom of Speech The legal cornerstone of any defense against governmental action that would place limitations on freedom of speech are the rights protected by the First Amendment to the U.S. Constitution. This protection against federal governmental action is extended by the Fourteenth Amendment against similar state actions. Note that these basic means of expression— speech, press, religion, assembly, and petition—are jealously protected not only in their fundamental forms but also in the extension of those forms. Such extensions may include flag burnings, antigovernment publications, or what the vast majority would label obscene. The theme of this extended protection is that censoring these questionable forms of expression would ultimately have a chilling effect on more generally acceptable forms. However, even these fundamental rights are not absolute. Certainly the freedom of speech does not condone your shouting “fire” in a crowded theater or a newspaper maliciously publishing lies about a celebrity. In the case of obscenity, for example, the basic law was formulated by the U.S. Supreme Court in the mid-1950s.

Internet makes exercising legal control over it nearly impossible?

2-3 The Internet and Constitutional Rights

39

that issue have varied through the years. The Roth decision, quoted above, indicated that the key question was whether or not the questionable material had any redeeming social importance. If it did not, then it could be regulated or banned totally by the federal or state government.

© GETTY IMAGES/PHOTODISC

MILLER V. CALIFORNIA In the 1960s the standard

Freedom of speech is an important right protected by the First Amendment. Besides communicating orally, what other means of expression are protected under the umbrella of freedom of speech?

ROTH V. UNITED STATES In Roth the Court deter-

mined that obscenity did not have the protection of the First Amendment. To reach this conclusion the court noted that the protections given speech and the press were to ensure the open exchange of ideas that would allow necessary political and social change. Therefore, to quote the Court in Roth v. United States, All ideas having even the slightest redeeming social importance—unorthodox ideas, controversial ideas, even ideas hateful to the prevailing climate of opinion—have the full protection of the guaranties, unless excludable because they encroach upon [interfere with] the limited area of more important interests. But implicit in the history of the First Amendment is the rejection of obscenity as utterly without redeeming social importance. Obviously, it is vital to be able to determine what is or is not obscene in order to apply the court’s decision. The standards used to determine 40

Chapter 2 > Constitutional Rights

became whether or not the questionable material was “utterly without redeeming social value.” This standard was much more difficult to prove and resulted in much greater latitude for those publishing material arguably obscene. Finally, in a 1973 case, Miller v. California, the Supreme Court toughened the standards by deciding that the questionable materials were to be judged by the standards of the community affected. Unfortunately, the Miller standards proved inadequate when applied to the Internet environment. Slowly, however, case law has begun to fill in the necessary gaps. For example, the issue of “community standards” raises the question, “Which community?” The answer to the question often determines whether or not someone will spend years in jail. Currently, the case law holds the sender to the standards of the recipient’s community. This tends to put the responsibility on the party profiting from such an exchange to know those standards and refuse to send the material to areas in which the community standards would make such an act illegal. Local community standards also are applicable in determining whether or not the material has any literary, artistic, political, or socially redeeming value when taken as a whole. Certainly, what is of such value will vary greatly from an urban to a suburban setting. Because of the unpredictability of the interpretation of the laws it has been suggested that the “community” be considered to be the Internet community rather than any particular geographical region. Thus far, the courts have declined to adopt this perspective. Supreme Court decisions of the last 50 years have held that once material is determined to be obscene, it can be regulated by the local, state, and federal governments. The types of regulation and their extent, however, have been open to challenge. See “Landmark Decisions in Cyberlaw,” on the next page. COMMUNICATIONS DECENCY ACT In 1996,

Congress passed the Communications Decency Act (CDA). One purpose of CDA is to control the knowing transmittal of obscene material

LANDMARK DECISIONS IN CYBERLAW Burger King Corporation v. Rudzewick The court determined that a defendant company or other entity of one state that has created a “substantial connection” through “minimum contacts” purposely directed toward a foreign state(s) becomes subject to the laws of that other state(s). 1985 decision of the U.S. Supreme Court found at 471 U.S. 462 Roth v. United States In this eloquently worded decision the court stripped away the protections of the First Amendment from obscene material. 1957 decision of the U.S. Supreme Court found at 354 U.S. 476 Miller v. California This decision set down a three-part test to be used to determine whether or not material can be considered obscene and consequently regulated or banned by local, state, and federal governments. 1973 decision of the U.S. Supreme Court found at 413 U.S. 15 Ashcroft v. American Civil Liberties Union (Ashcroft II) In this decision the U.S. Supreme Court upheld a district court injunction against enforcing the Child Online Protection Act (COPA) due to the Court’s feeling that the statute unduly chilled free speech where less-restrictive means are available to achieve the statute’s intended ends. 2004 decision found at 524 U.S. 656

through the Internet. The statute provides heavy fines and imprisonment for up to five years for first-time violators with even stronger penalties for subsequent offenses. Upon passage, the CDA came under immediate legal attack from the American Civil Liberties Union and others. The result, due mainly to the legally ambiguous term “indecent” used in the statute, was the declaration that the statute was overbroad and chilling of protected speech. Consequently, the statutory portions relying on that term were declared unconstitutional. Currently, another section of the Act aimed at protecting Internet sites from lawsuits for carrying content created by others is under attack. A lawsuit filed in Illinois in late 2006 targeted a housing ad site that allegedly carried racially discriminatory content created by the parties advertising properties for sale. The section of the CDA under attack has in the past even protected employers whose employees have used computer systems at work to violate the CDA’s own provisions in threatening and harassing others. However, the carrying of discriminatory housing ads violates another federal statute, the Fair Housing Act, which prohibits such third-person-created ads in print media. Internet companies such as Google, Yahoo, AOL, eBay, and Amazon have filed briefs in the case in support of the immunity provided by CDA. CHILD ONLINE PROTECTION ACT The Child

Online Protection Act (COPA) of 1998 required commercial sites to use up-to-date methodology to restrict access to their sites containing material harmful to children. This Act was amended due to the alleged inability of site operators to afford

the necessary equipment and the overbroad definition of “harmful material” so that it would have arguably included legitimate art works and scientific information about sex. As access-control technology continues to become more accurate and cheaper, the Act’s potential for enforcement by the federal government has increased. If convicted of violating the Act, commercial web sites face fines of up to $50,000 per incident and web-site operators face jail terms of up to six months. See the figure on page 43 for a summary of the statutes relating to free speech issues and citations to these statutes. In What’s Your Verdict? the Child Online Protection Act of 1998 might have made criminal both the actions of the news site in posting it and the actions of the Secretary of State’s staff in circulating the picture.

The Right of Privacy The right of privacy was legitimized as a right by the U.S. Supreme Court in the late 1960s. Ever since then, it has been based on a person’s reasonable expectations as to whether his or her actions, communications, beliefs, and other personal attributes deserve protection from those who would improperly use them. For example, a recent case involved an employee’s privacy regarding personal e-mail sent at work. The court held that the employee had no true expectation of privacy on a company’s e-mail system. Therefore, the employee’s personal e-mails were not protected by the right of privacy. Other issues of privacy remain unresolved. For example, spamming—the practice of sending unrequested bulk e-mailings to hundreds of thousands 2-3 The Internet and Constitutional Rights

41

of potential customers via the Internet—is supported by the ability to generate address lists. This generation typically is performed by popular web sites compiling the “cookies” that are left behind every time such a site is visited by a web user. A cookie is a packet of information containing the Internet address and other personal information about a web site visitor, such as buying patterns. This information is then sold without the user’s knowledge or permission. Existing federal law rightfully protects against such information being gathered on the videos and cable programs you rent or purchase. But, no such law exists in the United States that covers gathering information about users while they are online. The European Union (EU) has passed a directive that requires its member nations to pass legislation protecting consumers—not companies—against such practices. The European Union Directive forbids member nations from permitting the transfer of personal data to companies and other entities in a nation, like the United States, that does not provide an adequate level of personal privacy protection. Should U.S. firms doing business in the European Union, especially in the areas of credit and insurance, transfer data back to the United States they could be in violation of the directive and subject to penalties and civil suits. To address this problem, the Department of Commerce negotiated with the European Union to produce a

list of precautions that a U.S. firm should take in order to comply with the EU directive. BigBro Inc. assured its employees that any e-mails they sent over the company’s computer system would be confidential and that no action, such as reprimand or termination, would ever be taken based on such communications. However, Gulli Bell, an employee, sent several e-mails to a fellow employee complaining about management decisions and stating that the company president was “twice as smart as an ox.” She was dismissed soon thereafter, and the e-mails were cited as the primary reason. The state where BigBro was located was an employment-at-will state, meaning that Bell could legally be fired without statement of cause. Bell brought suit against BigBro for breach of her right of privacy but lost.

In THIS CASE

CHECKPOINt What are the constitutional

issues related to use of the Internet?

© PHOTODISCS/GETTY IMAGES

Whenever you visit a web site, personal information about you may be collected and then sold by the web site operator. What is your opinion of this practice? Do you think it should be outlawed? Why or why not?

42

Chapter 2 > Constitutional Rights

STATUTES OF IMPORTANCE TO CYBERLAW Lanham Act of 1976 (Title 15 U.S.C. Section 1051 et. seq.) Provides for the registration and protection of trademarks and copyrights. Trademark Dilution Act of 1996 (Title 15 U.S.C. Section 1125) Attempts to protect the holders of famous trademarks from the effects of domain-page registrations that tend to confuse and dilute the trademark’s goodwill. Computer Fraud and Abuse Act (Title 18 U.S.C. Section 1030) An often-amended act that pioneered the treatment of computer-related crimes as unique offenses in the United States Code. Telecommunications Act of 1996, also known as The Communications Decency Act of 1996 (CDA) (Title 47 U.S.C. Section 223, Title V) Makes it criminal to knowingly transport obscene material through interactive computers or interstate or foreign commerce for sale or distribution. In addition Section 230 of the CDA gives protection from defamation suits to Internet service providers. Child Online Protection Act of 1998 (Title 47 U.S.C. Section 231) Makes it a crime for site operators to fail to use proper technology and methodology to prevent minors from accessing “harmful” material. Fraudulent Online Identity Sanctions Act of 2004 Increased potential prison sentences by up to seven years for someone who provided “material and misleading false contact information to a domain name registrar, domain name registry, or other domain name registration authority.”

2-3 Assessment school.cengage.com/blaw/lawxtra

THINK ABOUT LEGAL CONCEPTS 1. Typically, when citizens of the same state sue one another, their case will be handled by (a) the state courts (b) the federal courts 2. What is obscene is now determined by one uniform national standard. True or False? 3. An employee’s personal e-mail, composed and sent at work, is protected by the right of privacy. True or False?

4. The Act that would offer protection to an employer whose employee used the computer system at work to harass his ex-girlfriend is the (a) CDA (b) COPA (c) FTCA (d) none of the above 5. Statutes that allow a citizen of one state to file suit against a non-resident of the state are called (a) Jurisdictional Reach Acts (b) long-arm statutes (c) Interstate Commerce Acts

THINK CRITICALLY ABOUT EVIDENCE Study the situation, answer the questions, and then prepare arguments to support your answers. 6. Sy Bore has been receiving spam mail advertisem*nts from adult-oriented web sites that contain pictures. Some of the pictures are very similar to ones that have been termed obscene in his state, and the senders were punished at criminal law there. However, the senders of the pictures he is receiving in the spam mail are located in states other than Bore’s. Bore persuades the local prosecutor to bring charges against the senders based on the obscene

nature of the pictures. At trial, the senders defend themselves by showing the material is not obscene in their home states. Can they still be convicted? 7. Consider the effects of making the European Union’s privacy standards federal law here in the United States. What groups would support a movement to do so? What groups would oppose the move? What would be the effects on business?

2-3 The Internet and Constitutional Rights

43

Chapter 2 Assessment

school.cengage.com/blaw/lawxtra

CONCEPTS IN BRIEF 2-1 Foundations of the U.S. Constitution 1. The documents that helped form the U.S. government include the Declaration of Independence, the Articles of Confederation, and the U.S. Constitution, including the Bill of Rights. 2. The First Amendment protects free speech, free press, free exercise of any private religion, freedom to assemble peaceably, and the right to petition the government for redress if one is wronged. 3. Civil rights are personal, natural rights of individuals guaranteed by the Constitution. 4. Although the Constitution does not spell out what is meant by “due process,” U.S. Supreme Court decisions have come to define it as a defendant being provided, at a minimum, adequate notice and a proper hearing. Due process of law is assured by the First, Fourth, Fifth, Sixth, Seventh, and Eighth Amendments. 5. Voting rights were extended to freed slaves by the Fifteenth Amendment and to women

through the Nineteenth Amendment. The voting age for all citizens was lowered to eighteen by the Twenty-Sixth Amendment.

2-2

Division and Balance of Governmental Powers 6. A vigorous and useful balance of power exists among the three basic branches of government: legislative, executive, and judicial. 7. The Tenth Amendment allows states the freedom to govern their citizens. However, the Constitution limits states’ powers through the Fourteenth Amendment.

2-3 The Internet and Constitutional Rights 8. Cyberlaw is developing at a rapid pace as the underlying technology advances. 9. Freedom of speech and the right of privacy are the two main constitutional issues related to Internet use.

YOUR LEGAL VOCABULARY Match each statement with the term that it best defines. Some terms may not be used. 1. Personal, natural rights guaranteed by the Constitution 2. Internet scanning program 3. Governmental system wherein each citizen may vote directly to decide issues 4. Division and allocation of the powers of government between its various branches 5. Governmental systems wherein citizens elect representatives to decide issues 6. Supreme political authority free from external control 7. Charter for common government for the 13 colonies prior to the adoption of the Constitution 8. Document that provides a framework for the federal government, consisting of 7 articles and 27 amendments 9. Concept of fundamental fairness in compliance with reasonable and just laws 10. The first ten amendments to the U.S. Constitution 11. Law that is intended to govern the use of computers in e-commerce 12. Change or alteration

44

Chapter 2 > Constitutional Rights

a. b. c. d. e. f. g. h. i. j. k. l. m. n. o. p. q.

amendment Articles of Confederation Bill of Rights browser civil rights cyberlaw Declaration of Independence democracy due process of law impeachment cases interstate commerce intrastate commerce political party republic sovereignty system of checks and balances U.S. Constitution

REVIEW LEGAL CONCEPTS 13. The Constitutional Convention of 1787 was meant to make changes in the Articles of Confederation instead of create a new document. Why do you think the Articles were discarded? 14. Can you foresee a time when the U.S. might become a true democracy? If so, how could this occur? 15. While the system of checks and balances works to prevent the accumulation of too much power by any one branch of the government, it also causes problems in the functioning of government. What do you think some of these problems might be? 16. Could you get your friends and neighbors to sign the Declaration of Independence today?

During the Vietnam era, young people around the country tried to do just that. Most citizens who read the document but were not told that it was the Declaration, refused to sign and, in some cases, reacted violently to its content. Why is that? 17. Susan, a manager in a local company, became romantically involved with one of her subordinates at work. They sent love messages back and forth by using the company’s Internet connection. The company regularly monitored its employee’s on-the-job e-mails and had a policy against such personal involvements. As a consequence, Susan was fired. She brought suit for invasion of privacy. Will she win?

WRITE ABOUT LEGAL CONCEPTS 18. Write an essay on how the framers of the Constitution and the Bill of Rights could justify the maintenance of slavery and the denial of voting rights to women. 19. Write a paragraph stating your opinion about which is the most important right preserved by the Bill of Rights. 20. The Bill of Rights is presented in this book in the language the founding fathers used. Rewrite the Bill of Rights in your own words.

21. HOT DEBATE While the passages denouncing the people of England and the slave trade were cut from Jefferson’s draft of the Declaration, numerous direct accusations against the King of England were left in the document. Why was the King singled out for criticism and not the people? Was this a wise move on the part of the Continental Congress?

MAKE ACADEMIC CONNECTIONS 22. HISTORY Rhode Island was the last of the 13 colonies to ratify the Constitution. Using the Internet and the library, determine what caused them to pause for so long. Write a one-page paper summarizing your findings.

23. SOCIAL SURVEY Apply the standards of the Child Online Protection Act to works of art, advertisem*nts, and other such displays easily visible to the young in your community. What conclusions can you draw about the Act and its enforcement as a consequence?

THINK CRITICALLY ABOUT EVIDENCE Study the situation, answer the questions, and then prepare arguments to support your answers. 24. Jennifer let her cousin use her computer during a recent visit. Without Jennifer’s permission the cousin visited several web sites that sold p*rnography. Now Jennifer is receiving spam from similar sites all around the world. No matter how often she reports the objectionable nature of the material, it keeps appearing. Evidently, the site her cousin visited sold her web address to others in the same line of business. It makes

her very upset, but she cannot change her web address for some time. What practical and legal means can Jennifer use to solve this problem? 25. In 1919 the Eighteenth Amendment outlawed “the manufacture, sale, or transportation of intoxicating liquors. . . .” This law led to widespread illegal traffic in liquor by criminals. The controversial law also led to unspoken approval of the illegal trade by many citizens

2-1 Foundations of Chapter the U.S. 2Constitution Assessment

45

who continued to buy and consume liquor. The Eighteenth Amendment was repealed by the Twenty-First Amendment, but individual states were permitted to continue to enforce Prohibition within their borders. By 1966, all individual

states had abandoned Prohibition. According to experts, abuse of alcoholic beverages is the most serious drug problem in overall harmful effect on society. Should Prohibition be reinstated? If not, what, if anything, should be done?

ANALYZE REAL CASES 26. A Champaign County, Illinois, board of education granted permission to religious teachers to give religious instruction in public school buildings in grades four to nine. Subject to approval of and supervision by the superintendent of schools, the teachers were employed by a private religious group and included representatives of the Catholic, Jewish, and Protestant faiths. Only pupils whose parents requested religious instruction were required to attend the religious classes. McCollum, a resident with a pupil who was enrolled in one of the public schools, sued for a writ, or court order, to end the religious instruction as a violation of the First Amendment to the Constitution. The writ was denied, and the Illinois Supreme Court upheld the denial. The parent appealed to the U.S. Supreme Court. Should the writ have been granted to end the practice as a violation of the First Amendment? Why or why not? (McCollum v. Board of Education, 333 U.S. 203) 27. In 1964, a motel owner in Atlanta restricted its clientele to white persons, three-fourths of whom were interstate travelers. U.S. government attorneys charged that this policy violated the Civil Rights Act of 1964, which forbids such discrimination against blacks. The trial court ordered the motel to stop refusing blacks as guests. The motel owner appealed to the U.S. Supreme Court, claiming the Civil Rights Act was unconstitutional. Was it? Why or why not? Does the federal government have the power to regulate matters that might in any way affect interstate commerce? (Heart of Atlanta Motel, Inc. v. United States, 379 U.S. 241) 28. Cleveland police had received information that a person wanted for questioning about a recent bombing was hiding in a particular two-family dwelling. There was said to be a large quantity of illegal lottery materials hidden in the home. Upon arrival, three officers knocked on the door and demanded entrance. Mapp, who lived on the top floor with her daughter, telephoned her attorney and then refused to admit the officers

46

Chapter 2 > Constitutional Rights

without a search warrant. Three hours later, reinforced by additional officers, the police returned. When Mapp did not answer immediately, they tried to kick in the door, then broke its glass pane, reached in, unlocked it, and entered. Meanwhile Mapp’s attorney had arrived, but the officers would not let him see his client or enter the house. Mapp demanded to see a search warrant. When an officer held up a paper claiming it to be a warrant, Mapp grabbed the paper and placed it in her bosom. A struggle ensued during which the officer recovered the paper. Mapp was handcuffed for resisting the officer. The entire house was searched, but all that was found were certain allegedly “lewd and lascivious books and pictures.” Mapp was convicted of knowingly having them in her possession. At the trial, no search warrant was produced nor was the failure to produce one explained. Mapp appealed to the U.S. Supreme Court for a reversal of her conviction because it was based on a search that was illegal under the U.S. Constitution. Was the evidence for conviction obtained in violation of the U.S. Constitution? If so, was it admissible in the trial against the defendant Mapp? (Mapp v. Ohio, 367 U.S. 643) 29. Weber was injured while staying in a hotel in Italy. When he returned to the United States, he filed suit against the hotel in a New Jersey court. Weber based his claim that the New Jersey court had jurisdiction on the fact that the hotel’s web site advertising for rooms for rent was available in that state. Was Weber correct in maintaining that the New Jersey court had jurisdiction? (Weber v. Jolly, 977 F. Supp. 327) 30. When a company began sending literally millions of unsolicited e-mails per day to its subscribers, America Online (AOL) blocked the transmissions. The sender then sued claiming an abridgement of its First Amendment rights by the action and requested an injunction preventing AOL’s actions against its spamming. How should the court decide? (Cyber Promotions, Inc., v. America Online, Inc., 948 F. Supp. 436)

Sports & Entertainment Law Seinfeld’s “George” a Plagiarization? C OSTANZA V. S EINFELD 693 N.Y.S.2 D 897 BACKGROUND Remember George Costanza, the character in Seinfeld portrayed for laughs as an inept, petty failure in business, romance, and other ventures? The plaintiff in this lawsuit, Michael Costanza, was not amused.

THE LAW Defamation requires a false written or oral statement to a third party or parties that brings the victim into disrepute, contempt, or ridicule by others.

THE ISSUES (1) Was Michael Costanza defamed by the portrayal of George Costanza on Seinfeld? (2) Was he defamed by being called a flagrant opportunist for his lawsuit and publication? HOLDING Because Michael Costanza’s actual image was not associated with the character portrayed on television, the Court held that defamation was not a viable cause of action. In addition, giving the timing of the book and lawsuit, the label “flagrant opportunist” was considered a statement of opinion, not fact. Consequently it could not base a lawsuit for defamation. PRACTICE JUDGING 1. Do you consider Michael Costanza a public figure? Why or why not? What would be the consequences for the lawsuit if he were considered a public figure? 2. Ethically, do you feel that Michael Costanza was defamed? Why or why not?

© TODD BIGELOW/AURORA/GETTY IMAGES

FACTS Michael Costanza sued comedian Jerry Seinfeld, NBC, and others for invasion of privacy and defamation. The cause of action was based upon the use of the character over the successful TV show’s long run. George Costanza had Michael Costanza’s character and last name. Michael alleged that George, bald and obese, even looked like him. Michael also noted that, like himself, the show’s character was portrayed as knowing Seinfeld from college and being from Queens, New York. Michael alleged that the character’s personality, including the traits of being self-centered, unreliable, and generally incompetent, have been imputed to him by the similarities and have humiliated him. He also noted that the co-creator of Seinfeld called him a “flagrant opportunist” for publishing a book alleging his connections with and handling by the defendants during the highly publicized timeframe of the show’s last episode. He sought damages in the amount of $100 million because he believed the interplay with George Costanza was one of the main reasons for the show’s successful and very lucrative run. Jerry Seinfeld, NBC, and other defendants offered as defenses that Michael Costanza waived any right to sue by accepting a minor role on a Seinfeld episode aired in

the middle-1990s, that he made the similarities public in a book on the subject, and that the label “flagrant opportunist” was a statement of opinion and was, therefore, not actionable as being defamatory.

Sports & Entertainment Law

47

Chapter 3 Court Systems

Anthony Destin works as a junior designer at a large fashion house called Berentinos. After two years on the job, Anthony learns that his co-worker Sarah Blake earns 35 percent more than he does. Anthony and Sarah were hired at the same time and have exactly the same job. Anthony has five more years experience in the field than Sarah does. Anthony discussed his concern with Tom Sortee, one of his colleagues. Tom commented that such discrimination against males has long been the rule at Berentinos. Anthony loves his work but can’t help feeling he is being treated unfairly.

WHERE DO

YOU STAND?

1. What alternative courses of action, other than filing suit in court, might Anthony utilize to bring about a resolution of the issue? 2. Does Anthony have a duty to anyone, legally or morally, to bring a lawsuit instead of using alternative forms of dispute resolution?

48

Chapter 3 > Court Systems

© GETTY IMAGES/PHOTODISC

3-1 Forms of Dispute Resolution 3-2 The Federal Court System 3-3 State Court Systems

3-1 Forms of Dispute Resolution GOALS

KEY TERMS

■ Explain how disputes can be settled without

litigate mediator arbitrator court trial court

going to court ■ Name the different levels of courts and describe their jurisdictions and powers

Dispute Resolution

the clause is enforceable. If both parties agree to the contract then either party can require a dispute over the repairs to be resolved by arbitration.

What's Your Verdict?

Odeen and Dva Click ran the Clickety-Clack Garage in a suburb of Boston, Massachusetts. Part of their standard repair contract reads that any and all disputes arising out of their repair work were to be settled by arbitration rather than by a court of law.

How Do Courts Settle Disputes? A court can be defined as a governmental forum that administers justice under the law. Courts decide disputes between private individuals and try criminal cases. A court may award damages or order other appropriate relief in resolving private disputes and impose punishment (fine and/ or imprisonment) in the criminal cases. Courts follow impartial and thorough procedures to make decisions. Witnesses are in some cases compelled to give testimony. The accused party is allowed equal opportunity to argue her or his side of the case. Typically, two levels of courts are involved in deciding a dispute. These two levels are trial courts and appellate courts.

Is this clause enforceable?

© GETTY IMAGES/PHOTODISC

How Can Disputes Be Resolved Without Going to Court? Many people decide too quickly to litigate—or to allow a court to resolve their disputes. When one person injures another or fails to keep a binding agreement, often the best solution is for the parties to negotiate a settlement themselves. Together, they often can reach a mutually acceptable solution. As an alternative to face-to-face negotiations, the parties may invite an independent third party to act as a mediator. The mediator tries to develop a solution acceptable to both sides of the dispute. The actions of a mediator are advisory (not legally binding on the parties). In other cases, the parties may retain an arbitrator. An arbitrator usually holds an informal hearing to determine what happened. The arbitrator’s decision, unlike that of a mediator, is binding on both parties. The decision can be enforced by court order if necessary. Sometimes a provision for arbitration is included in the original agreement between the parties. In What’s Your Verdict?,

verdict original jurisdiction appellate court transcript appellate briefs

Disputing parties may retain a mediator or an arbitrator to develop a solution. What is the difference between a solution offered by a mediator and a solution offered by an arbitrator?

3-1 Forms of Dispute Resolution

49

TRIAL COURTS The court in which a dispute is first heard is called a trial court. The trial court

hears the witnesses testify and reviews the other pertinent evidence firsthand so as to determine the facts of a case. The trial court will then apply what it selects as the appropriate law to the facts to reach a verdict, or decision, in the case. Because it has the power to make these initial decisions of fact and law, a trial court is said to have original jurisdiction over the case. A trial court consists not only of a judge but also of lawyers, who are officers of the court. Other court personnel necessary for its operation include clerks, sheriffs or marshals, bailiffs, and jury members. Clerks enter cases on the court calendar, keep records of proceedings, and often compute court costs. Sheriffs or their deputies serve as bailiffs. Bailiffs summon witnesses, keep order in court, and take steps to carry out judgments in the state court systems. Marshals have these duties in the federal court system. Juries are citizens sworn by a court to decide issues of fact in court cases.

fact. Instead appellate jurisdiction is concerned solely with errors of law. Appellate courts examine the transcript, which is a verbatim record of what went on at trial. They also read appellate briefs, or written arguments on the issues of law, submitted by the opposing attorneys. Appellate courts also often listen to attorneys’ oral arguments in support of these briefs. During such oral argument the appellate judges question attorneys about the case. Finally, appellate courts decide whether the decision of the lower court should be affirmed (upheld), reversed (overturned), amended (changed), remanded (sent back to the trial court for corrective action or possibly a new trial), or a combination of these.

CHECKPOINt What are the two levels of

courts, and what is the function of each?

© GETTY IMAGES/PHOTODISC

APPELLATE COURTS An appellate court reviews decisions of lower courts when a party claims an error of law was made during the lower court’s proceeding. Because appellate court judges were not present to evaluate firsthand the truthfulness of the witnesses, their testimony, and other evidence, they do not make determinations of

Contact the clerks of your local courts— city, county, and state—to find out the types of cases over which each court has jurisdiction.

Which type of court, trial or appellate, is pictured here? Justify your answer.

50

Chapter 3 > Court Systems

3-1 Assessment school.cengage.com/blaw/lawxtra

THINK ABOUT LEGAL CONCEPTS 1. Which of the following means of dispute resolution may be enforced by a court order? (a) The parties invite a mediator to develop a solution. (b) The parties retain an arbitrator to prescribe a solution. (c) The parties call the police in to resolve the dispute. 2. There is no difference between mediation and arbitration. True or False? 3. A trial court has _?_ jurisdiction over a case. 4. Courts (a) award damages in criminal cases (b) hear witnesses at the appellate level (c) decide questions of fact at the trial level (d) enforce mediators’ decisions (e) all of the above 5. Appellate courts review decisions of lower courts when one or more of the parties appeal, claiming an error of law was made during the lower court’s proceeding. True or False? 6. Appellate court review of trial court decisions is normally confined to errors of _ ?_.

7. Which of the following are involved in appellate court proceedings? (a) bailiffs (b) juries (c) judges (d) witnesses 8. To _ ?_ means to allow a court to resolve a dispute. 9. When an appellate court sends a case back down to a trial court for a new trial, it is said to have _ ?_ the case. (a) affirmed (b) reversed (c) remanded (d) none of the above 10. A decision of the lower court that has been overturned by an appellate court is said to have been (a) affirmed (b) reversed (c) amended (d) remanded 11. A decision of the lower court that has been upheld by an appellate court is said to have been affirmed. True or False? 12. A decision of the lower court that has been changed by an appellate court is said to be (a) affirmed (b) reversed (c) amended (d) remanded

THINK CRITICALLY ABOUT EVIDENCE Study the situation, answer the questions, and then prepare arguments to support your answers. 13. Your boss at Swift Electronics is getting ready to enter into a series of contracts with a new group of suppliers. She is concerned with avoiding the delays and high costs of any litigation that might result from the new agreements. She asks you for your recommendations. You suggest putting a binding arbitration clause in the contracts. Why? 14. Your boss at Swift Electronics decides against the recommendations you made (see exercise 13). Six months later, a supplier sends the wrong projection device in response to your order. The supplier then refuses to correct the mistake. Your boss hires an attorney at $250 per hour and takes you to lunch to admit she made a mistake in not following your ideas. Ultimately, the case is set for trial. What type of jurisdiction will this court have? What will be done during the trial?

15. In exercise 14, who may be present during the trial? If there is an appeal, which of these parties will not have to appear again? 16. Don Long is convicted of second-degree murder for killing a woman he had been seeing over the last few months. The court sentenced him to 30 years in jail. One year after his trial, Don admits that he withheld evidence that would have cleared him. He states he did so to protect his twin brother, who actually committed the crime, but who was dying of cancer at the time. He further states that he did not want his brother to spend the last years of his life behind bars. His brother has since died. The evidence is conclusive as to Don’s innocence. Should the court set aside his conviction due to the new evidence? What policies would support the court in not doing so?

3-1 Forms of Dispute Resolution

51

3-2 The Federal Court System GOALS

KEY TERMS

■ Identify the source of power of the federal courts

general jurisdiction specialized jurisdiction

■ Name the major federal courts and describe

writ of certiorari

their jurisdictions and powers

Origin of the Federal Court System What's Your Verdict?

Tom and Martina were discussing how the various federal courts were empowered to decide cases. Tom said that federal courts received their power from the U.S. Constitution. Martina maintained that Congress had created and empowered them. Who is correct?

Through Article III of the Constitution, the people conferred the power to judge certain criminal and civil matters on a system of federal courts: Section 1. The judicial Power of the United States shall be vested in one Supreme Court, and in such inferior Courts as the Congress may from time to time ordain and establish. The Articles of Confederation did not allow for a Supreme Court, and some citizens did not think a Supreme Court would be needed under the Constitution. As a result, after George Washington’s inauguration as the nation’s first president, it took nearly six months for Congress to utilize the power it was granted under Article III and pass the Federal Judiciary Act. This act “ordained and established” the U.S. Supreme Court (USSC) and thirteen district courts. In 1891, approximately a century after passing the initial Federal Judiciary Act, Congress passed a Judiciary Act that established the federal Courts of Appeal. Certain specialized courts, such as those concerned primarily with tax or bankruptcy matters, also were created as the need for them arose. In What’s Your Verdict? both Tom and Martina are correct. Ultimately, the federal courts receive their power to adjudicate cases from the Constitution. 52

Chapter 3 > Court Systems

However, the Constitution left to Congress the power to ordain and establish these courts if and when it deemed them necessary. CHECKPOINt What is the source of power

of the federal courts?

Jurisdiction of the Federal Courts What's Your Verdict?

Susan Bean, a citizen of Illinois, sued Wallis Turk, a citizen of the state of Colorado, for the breach of a construction contract on Bean’s new Chicago residence. More than $600,000 was at stake. Bean filed the suit in Illinois state court. Turk filed a motion to remove the case to the federal courts. Will the case be heard in federal or state court?

Currently there are three levels of federal courts with general jurisdiction. These are federal district courts, federal courts of appeals, and the U.S. Supreme Court. A court with general jurisdiction can hear almost any kind of case. A court with specialized jurisdiction hears only one specific type of case. The chart on page 53 illustrates the three levels of the federal court system.

Federal District Courts The federal (or U.S.) district court is the lowest level of federal court with general jurisdiction. This is the trial court (the first court to hear the dispute) of the federal system. It has the power to determine the facts and to make initial determinations of the law to use in deciding the case.

In general, district courts FEDERAL COURT SYSTEM have original jurisdiction over (a) federal questions, or cases that arise under the Constitution, U.S. law, and UNITED STATES SUPREME COURT U.S. treaties; and (b) lawsuits between citizens of different (Cases Involving Federal Law) states, between a U.S. citizen and a foreign nation, or between a U.S. citizen and 13 UNITED STATES COURTS OF APPEALS (12 Circuit Courts) a citizen of a foreign nation. (1 Court of Appeals for the Federal Circuit) These parties are said to have diversity of citizenship. More than $75,000 must be in dispute for a federal court to STATE SPECIALIZED MANY hear a diversity of citizenship UNITED STATES SUPREME FEDERAL FEDERAL DISTRICT COURTS lawsuit. Otherwise a state COURTS COURTS* AGENCIES court will decide the case. Bean v. Turk in What’s *Among the federal courts with specialized jurisdiction are the Tax Court, Court of International Trade, and the Claims Court. The Bankruptcy Court acts as an adjunct Your Verdict? will be tried of the District Courts. in the federal courts. As the case has more than $75,000 in question and is between citizens of different according to the Constitution, is over “cases affectstates, it falls within the jurisdictional limits of the ing ambassadors, other public ministers and consuls federal courts. These courts, when at least one of and those in which a state shall be party.” The most the parties requests their exercising it, have priority important function of the USSC, however, is the jurisdiction over state courts in such diversity cases. exercise of its appellate jurisdiction. This jurisdiction is exercised over cases on appeal from the U.S. Federal Courts of Appeals Courts of Appeals or from the highest courts of the The federal courts of appeals have appellate jurisvarious states. If the Supreme Court believes that diction over the district courts, certain specialized a case contains a constitutional issue sufficiently federal courts, and many federal administrative important to be decided by it, the Supreme Court agencies. Such power is exercised when the result will issue a writ of certiorari to the last court that of a case in a lower court is appealed by one or heard the case. This “writ” or order compels the more of the parties to the case. lower court to turn over the record of the case to Appellate courts do not accept any new evithe Supreme Court for review. dence or call witnesses. Instead, they review the trial transcripts, appellate briefs, and oral arguments of the attorneys to reach a decision. No appellate court, not even the USSC, can change the factual determinations of a jury. There are 13 federal courts of appeal. Twelve of these are circuit courts, each of which is Access school.cengage.com/blaw/lawxtra and responsible for an assigned geographic area. The click on the link for Chapter 3. Click on one of thirteenth is dedicated to the “federal circuit.” As the topics on the home page of the official web such, it handles patent cases appealed out of the site of the U.S. Supreme Court. Write a one-paradistrict courts and appeals from federal courts graph summary of your findings. with specialized jurisdictions.

United States Supreme Court

school.cengage.com/blaw/lawxtra

The U.S. Supreme Court (USSC) has both original and appellate jurisdiction. Its original jurisdiction, 3-2 The Federal Court System

53

Jurisdiction over state supreme court cases is limited to those in which a federal question has been brought out at the trial court level. The decisions of the USSC that interpret or apply the U.S. Constitution are final and can only be overturned by the USSC itself or by a constitutional amendment. CHECKPOINt Name the three levels of

federal courts and describe the jurisdiction of each.

The jury in Martin’s case convicted him of grand larceny partially based on evidence found in the trunk of the car he was driving. The state appellate courts upheld his conviction. The USSC, however, took jurisdiction over the case by issuing a writ of certiorari to the state supreme court. The USSC on appeal then held that the search of his trunk that discovered the items was improper. The USSC overturned his conviction, as the jury could have been improperly influenced by such evidence.

In THIS CASE

3-2 Assessment school.cengage.com/blaw/lawxtra

THINK ABOUT LEGAL CONCEPTS 1. A Supreme Court did not exist under the Articles of Confederation. True or False? 2. There are _ ?_ levels of courts with general jurisdiction in the federal system. 3. The federal trial courts are known as (a) district courts (b) federal county courts (c) common law courts (d) all of the above

4. Cases involving citizens of different states in which less than $75,000 is at stake must be tried in the state courts. True or False? 5. An appeal to the federal court system of a ruling of a federal agency initially would be taken to (a) a federal district court (b) a federal court of appeals (c) the USSC

THINK CRITICALLY ABOUT EVIDENCE Study the situation, answer the questions, and then prepare arguments to support your answers. 6. Ms. Tant of New York City recently sued Mr. Bloom, also of New York City. She claimed that he had run into and injured her while he was jogging. She asked for $50,000 in damages. When she filed her suit in federal district court, Mr. Bloom’s attorney immediately moved for the case to be dismissed from the federal court for two reasons. What were they? 7. Ms. Tant’s case (see exercise 6) was thrown out of federal court. She later filed it in the New York state court solely as a case involving Mr. Bloom’s negligent jogging. When she lost, she appealed all the way to the highest New York state court but still lost. She then sought to appeal to the U.S. Supreme Court. Will the U.S. Supreme Court hear her case? Why or why not?

54

Chapter 3 > Court Systems

8. Paul Stone sued his employer for assault and battery due to the actions of several of his female co-workers. While gathered around the coffee machine each morning, they would whistle at him, make lewd comments, and touch and pinch him. When the case was dismissed from the state circuit court, Stone appealed. The intermediate court of appeals sustained the result in the lower court, and Stone appealed to the state supreme court. When the state supreme court also sustained, Stone sought to appeal to the U.S. Supreme Court. Are there any federal issues in this case that would allow the U.S. Supreme Court to take jurisdiction? What might prevent the nation’s highest court from so doing?

3-3 State Court Systems GOALS

KEY TERMS

■ Compare the structure of a typical state court

court of record associate circuit courts county courts

system with the structure of the federal courts ■ Explain the jurisdictions of the specialized courts in a typical state system

Sheila had a beautiful apartment with a view of the harbor. After she had lived there several years, her landlord gave her and the other tenants of the apartment building 30 days notice to vacate. Sheila sued to prevent the mass evictions. After losing in the state trial court, she vowed to appeal the issue directly to the U.S. Supreme Court.

© GETTY IMAGES/PHOTODISC

Can she do so?

In most states trial courts—those courts with general original jurisdiction over both criminal and civil matters—are known as circuit courts.

probate courts

Other states refer to them as superior courts, district courts, or courts of common pleas. Regardless of their name, these trial courts are the courts of record in the state system. A court of record keeps an exact account of what goes on at trial. The accuracy of this “record” is vital, as any appeal filed depends on it. The record may include a transcript of what was said, the evidence that was submitted, statements, the determinations of the court officials, and the judgment of the court. State trial courts also review the decisions of courts of more specialized jurisdiction, such as the small claims courts, which are not courts of record. Where necessary, state trial courts retry such cases to make a proper record for the purpose of potential appeals. Because a state trial court has original jurisdiction over cases before it, it makes determinations

What's Your Verdict?

State Trial Courts

juvenile courts

municipal courts

A Typical State Court System

The typical state legal system resembles the federal system. The state legislature makes the laws. The state executive branch enforces those laws in the courts of the state judicial branch. The courts of a typical state’s judicial branch are organized into three tiers. In the bottom tier is a geographically based set of trial courts with either general or specialized jurisdictions. An appellate tier of courts is next. This appellate tier is then capped by a state supreme court as the ultimate level of appeal. Both the trial and appellate courts are controlled and supervised by this supreme court. In What’s Your Verdict? Sheila could not take her appeal directly to the U.S. Supreme Court. Generally, there is no guaranteed right to be heard on appeal by even the state supreme court, with the exception of capital murder convictions.

small claims courts

If the parties to a case do not request a jury, a judge will preside over the case. Do you think it is always in the best interest of the parties to request a jury? Why or why not?

3-3 State Court Systems

55

A TYPICAL STATE COURT SYSTEM

SUPREME COURT

INTERMEDIATE APPEALS COURT (In Populous States)

TRIAL COURT (Of Original, General Jurisdiction)

(Courts of Limited Jurisdiction)

FAMILY COURT

JUVENILE COURT

MUNICIPAL COURT

JUSTICE’S COURT (The Court of a Justice of the Peace)

PROBATE COURT

CRIMINAL COURT

SMALL CLAIMS COURT

of the facts in the case often by using a jury. If a jury is not requested by the parties in the case, the judge will determine the facts. Once the facts have been determined, the judge will select and apply the law to the facts to reach a verdict.

the correct law themselves to reach a new result. If, however, they conclude that the use of the incorrect law interfered with the proper determination of the facts of the case, they may send the case back down for a new trial.

State Courts of Appeals

State Supreme Courts

An appeal from a court of record is typically reviewed by a panel of judges in a state court of appeals. The panel of judges from the state court of appeals usually consists of no more than three judges. The state court appellate procedure is very similar to that of the federal courts of appeals. In particular, the state appellate panel evaluates the record of the case and the appellate briefs submitted by the attorneys for the parties. It also may hear oral arguments by those attorneys in the case. No new evidence can be introduced at this level. The judges at the appellate level check to be sure that the lower court used the correct law to conduct and resolve the case. If the correct law was used, the result in the case will be upheld. If court of appeals judges conclude that the trial court used the incorrect law, they may apply

Generally, in whatever legal issue you confront, you are entitled to a trial and to one appeal, if filed in a timely manner and in the proper form. In states with the intermediate level of courts of appeals, only cases that involve the most vital and complex legal issues are taken to the justices of the state supreme court. (Justice is the title given to judges who sit on state supreme courts and the federal Supreme Court.) Typically, at the state supreme court level, a panel of three or more justices reviews the cases on appeal in the same manner as the lower appellate courts. State supreme courts issue the final decision on matters of law appealed to them. If a U.S. Constitutional or other federal question of law is involved, however, a further appeal can go to the U.S. Supreme Court. In several states, in addition to its appellate jurisdiction, the state

56

Chapter 3 > Court Systems

supreme court has original jurisdiction over most state impeachment cases. CHECKPOINt What does a typical state

Although the penalties for violating ordinances can be as severe, ordinances are not considered criminal laws. Only state and federal governments can make an act criminal.

court system have in common with the federal court system?

Small Claims Courts

State Courts with Specialized Jurisdictions What's Your Verdict?

Chase, age 15, violated his state’s anti-hacking law. He was caught and referred to a juvenile court.

Will he be treated differently under the juvenile court’s jurisdiction than under a regular trial court’s jurisdiction?

Below the main circuit court level in most states are courts with specialized jurisdictions. These courts include the associate circuit, municipal, small claims, juvenile, and probate courts.

Minor individual suits would not often be heard if not for the small claims courts. These courts handle disputes in which small amounts, generally $2,500 or less, are involved. Attorneys generally are not required in small claims courts. The judge hears the case without a jury or formal rules of evidence. Decisions of small claims courts also can be appealed to a state trial court.

Juvenile Courts Younger members of society—those over 13 and under 18 years of age in most states—are referred to as juveniles. The map on page 58 shows the minimum age at which each state tries an individual as an adult. Society typically believes that juveniles should not be held as responsible as adults for their criminal acts. To carry out this policy, special juvenile courts have been set up. In these courts the juvenile is entitled to his or her full constitutional rights, including the right

Associate Circuit Courts Many states have a layer of courts below their main courts of general original jurisdiction. These lower courts are referred to as associate circuit courts or county courts. Such courts hear minor criminal cases, state traffic offenses, and lawsuits in which relatively small amounts are involved (usually no more than $25,000). Generally, these courts are not courts of record. However, they take a significant burden off the higher courts, even though appeals from their decisions can be taken to a state trial court for a trial on the record.

City or Municipal Courts Cities typically have courts that administer their ordinances. These municipal courts are usually divided into traffic and criminal divisions. As city ordinances often overlap with or duplicate state laws, less serious violations occurring within city limits are brought before such municipal courts for their first trial. The result can then be appealed to the state trial court level if necessary.

A

t age 17, Horace Samuels was brought before the juvenile court charged with child molestation. He was convicted and, as a juvenile, was sentenced to probation in a foster home. While in the home, he was again caught and tried for the same offense. Again, he was tried as a juvenile. Now he is 18 and about to live on his own in an area in which many children of the same age that he has been charged with molesting also live. Horace’s probation officer realizes that Horace has not been rehabilitated and is a potential danger to the neighborhood’s young. The law requires that the juvenile’s record remain closed so that the individual may begin a new life free and clear of past mistakes. What should the probation officer do?

3-3 State Court Systems

57

AGE TRIED AS AN ADULT

WA MT

VT ME

ND MN

ID WY

WI

SD NE

NV

UT

CO

KS

CA AZ

OK

NM

IA

MI

HI

AK

PA

OH IL IN

MO

KY

VA

NH MA RI NJ CT DE MD

NC

TN

SC

AR MS AL

TX

NY

W V

OR

GA

LA FL

15 14 13 12 10

Washington, D.C., and 22 states do not have a set minimum age for a child to be tried as an adult. Twenty-eight of the states do have a set minimum age—the range is 10 to 15 years of age.

to be represented by an attorney. If the juvenile is found guilty of the charges brought, the court has wide powers in determining what should be done. However, the emphasis for juveniles generally is on rehabilitation, not punishment. Possibilities open to the court include release into the supervision of parents, guardians, or governmental officials; placement in foster homes; and detention in correctional facilities. These courts ensure that most of the criminal cases involving juveniles do not become public knowledge. The courtroom is closed while an

A

juvenile offender’s rehabilitation is a priority with society to prevent the loss of the productive resources that youth could someday provide. Juveniles are capitalized by society through education and supported by various health systems and programs. Instead of sending them off to jail and pushing them into the cycle of crime and punishment, society seeks to protect its investment by assisting them to get back on a productive path.

58

Chapter 3 > Court Systems

informal hearing into the charges is conducted. Any records made on juvenile cases are not open to the public. If rehabilitation fails or is shown to be impossible, the young offender can be tried and punished as an adult. This occurs only in cases involving a very serious offense. For example, murder and certain other crimes may bring about trial and punishment through the criminal law system. Appeals from actions of the juvenile courts are directed to the circuit courts. In What’s Your Verdict? Chase would be treated far differently under the juvenile court’s jurisdiction. He would be entitled to a juvenile court hearing where his age, level of maturity, and potential would be considered in determining how he is to be treated.

Probate Courts When individuals die, their property and other interests must be divided according to their wishes and the appropriate laws. The courts that administer wills and estates to accomplish this task are called probate courts.

CHECKPOINt Name the typical state

courts that have specialized jurisdiction.

3-3 Assessment school.cengage.com/blaw/lawxtra

THINK ABOUT LEGAL CONCEPTS 1. Which of the following is generally not included in the “record” of a case? (a) verbatim transcript (b) physical and documentary evidence (c) media coverage (d) all of the above are generally included 2. Attorneys generally are not required in a small claims court. True or False? 3. A state trial court has the power to hear some appeals. True or False?

4. A dispute between neighbors involving property damage of $750 would probably be heard in (a) federal district court (b) probate court (c) small claims court (d) associate circuit court 5. A state supreme court can make its own determinations of the facts in a particular case. True or False?

THINK CRITICALLY ABOUT EVIDENCE Study the situations, answer the questions, and then prepare arguments to support your answers. 6. Gwendolyn Hunt was driving while intoxicated when she hit and killed Felicia Meyers, a sevenyear-old, in a school crosswalk. Gwendolyn was 16 years old at the time. However, she was on probation from a reduced sentence for driving 62 in a 35 mph residential speed zone. Should Gwendolyn be tried as an adult for vehicular manslaughter? 7. You are driving along a residential city street when you are stopped by a city police officer for speeding. You are given a citation and a summons to appear before a particular court. What court would that most likely be and why? 8. You are driving along the interstate highway headed for a ballgame in a nearby city. A state highway patrol car suddenly appears in your rearview mirror, lights flashing. You pull over and are cited for doing 80 in a 65 mph zone. Your receive a summons to a particular court. What court would that most likely be and why? 9. After a long illness, your uncle dies. He leaves a sizeable estate but no will is found for several

weeks. Finally, one of his ex-wives appears with a document she claims to be a valid will. It shows her and her children by him receiving most of his property. Your parents and the deceased’s other children contest the will. Should the federal or state courts handle this case? Why? Which court in which system is most likely to hear it? 10. Jean Marie of Bangor, Maine, ordered a new laptop computer from a company in California. The computer was advertised as having cuttingedge technology and the fastest computing speed in its class. When Jean received the laptop, he found it ran much slower than expected. Upon further examination, he discovered that the computer chips utilized were out of date and their speed less than 25 percent of the current ones. The laptop cost $499 plus shipping. The company refused to refund Jean’s money. Should he sue? What court would be available to him? Will he receive justice in the matter?

3-3 State Court Systems

59

Chapter 3 Assessment

school.cengage.com/blaw/lawxtra

CONCEPTS IN BRIEF 3-1 Forms of Dispute Resolution 1. Mediation and arbitration offer dispute resolution alternatives to litigation. Using these alternative methods of dispute resolution helps to avoid the costs, delays, and difficulties of a court trial. 2. The levels of courts are trial courts, which have original jurisdiction, and appellate courts, which review decisions of lower courts. 3-2 The Federal Court System 3. The federal courts receive their power from Article III, Section 1, of the U.S. Constitution. 4. Federal courts can have either general or specialized jurisdiction. Courts with general jurisdiction include federal district courts, federal courts of appeals, and the U.S. Supreme Court. Courts with specialized jurisdiction hear only one specific type of case. 3-3 State Court Systems 5. The typical state court system is similar to the federal court system. Both the federal and most

6.

7.

8.

9.

10.

state court systems have trial courts, appellate courts, and a supreme court. Associate circuit courts, or county courts, hear minor criminal cases, state traffic offenses, and lawsuits involving small amounts of money. Municipal or city courts usually feature two divisions, one to handle traffic and another to handle violations of all other ordinances. Penalties for ordinance violations (which are not considered crimes) are less severe than federal and state criminal law penalties. Small claims courts feature relaxed rules of evidence and do not require representation by an attorney. The dollar amount that can be adjudicated usually is limited to around $2,500. In every state, there are juvenile courts with specialized jurisdiction over persons under a certain age. These courts emphasize rehabilitation over punishment. In certain instances, juveniles can be bound over for trial as adults. Probate courts settle the estates of decedents who die with or without a will.

YOUR LEGAL VOCABULARY Match each statement with the term that it best defines. Some terms will not be used. 1. Verbatim record of what went on at trial 2. Court that reviews trial court decisions to determine if a significant error of law was made during trial 3. Court in which an accurate, detailed report of what went on at trial is made 4. Independent third party who develops a binding, courtenforceable dispute resolution 5. Order to a lower court to produce the record of a case for the Supreme Court to review 6. Power to hear a case in full for the first time 7. Court that hears cases involving individuals generally between the ages of 13 and 18 8. To place a dispute before a court of law for resolution 9. Court that hears cases involving violations of ordinances 10. Power to hear only cases of a specific type

60

Chapter 3 > Court Systems

a. b. c. d. e. f. g. h. i. j. k. l. m. n. o. p. q. r. s.

appellate brief appellate court arbitrator associate circuit court (or county court) court court of record general jurisdiction juvenile court litigate mediator municipal court original jurisdiction probate court small claims court specialized jurisdiction transcript trial court verdict writ of certiorari

REVIEW LEGAL CONCEPTS 11. What is the fundamental difference between arbitration and mediation? 12. Why are appeals heard by higher courts? 13. Why should the federal courts handle cases involving private matters between citizens of different states (for example, cases involving

diversity of citizenship with more than $75,000 at stake)? 14. Why shouldn’t the USSC be able to substitute its own factual determinations for those of a jury in a lower court?

WRITE ABOUT LEGAL CONCEPTS 15. Wherever you go you are charged with knowing all the laws, ordinances, rules, and regulations that apply to you. “Ignorance of the law is no excuse.” Because of the confusion this causes, there has been a movement in the last 50 years to make state laws more uniform. From sales law to various parts of the criminal code, the effect has been positive. Should state court systems be made more uniform? What are the advantages and disadvantages of such an idea? 16. Judges cost an extraordinary amount of money. To attract competent personnel to the judiciary, salaries must be comparable to what

an exceptional lawyer would earn in private practice. A way to provide the necessary salary levels is to save money elsewhere in the system. To that end, many states use extensively trained non-lawyers to preside over municipal courts and hold other minor judiciary positions. Do you think this is a good idea? Why or why not? 17. HOT DEBATE Write a paragraph discussing the following question: If Anthony decides to file suit in state court against his employer, what federal court might ultimately claim jurisdiction? Under what circ*mstances?

MAKE ACADEMIC CONNECTIONS 18. BIOLOGY The jurisdictional shield of the juvenile courts ends at age 18. Research the maturation process of hom*o sapiens and determine whether or not, according to the

scientific evidence, 18 is a good cutoff age. What implications does the result of your research have for driving age, voting age, age to enter the military, and age to drink in your state.

THINK CRITICALLY ABOUT EVIDENCE Study the situation, answer the questions, and then prepare arguments to support your answers. 19. Gomez wanted to sue Shapiro for failure to complete work under their contract but could not afford the expenses and time delays associated with litigation. What alternatives are available to resolve this matter? 20. If you were called to testify as a witness in a friend’s criminal trial, would you have the moral strength to tell the complete truth even if your friend might be convicted as a result? 21. June lost her case in trial court. She thought that the plaintiff, Sid, had lied during the trial. On appeal, she requested that she be allowed to appear and explain why she thought Sid had lied. Will her request be granted?

22. Suppose someone has injured you. Do you have a duty to sue them for the injury even when the costs of bringing suit may be more than you can recover? 23. On May 7, Bart Masters turned 18. The evening of that day, he was caught stealing a woman’s purse. “Don’t worry,” he told his friend Julia, “I just turned 18 today, they won’t try me as an adult. I’m too close to being a juvenile.” Is he correct? 24. Priscilla is being sued in small claims court by her landlord for past due rent. “Just wait,” she exclaims. “Once I get in front of that jury, they won’t award him the money. After all, he shut off the water several times and didn’t even have heat in the building for over a month last winter.” What is wrong with Priscilla’s estimate of the situation?

3-1 Forms of Chapter Dispute 3 Assessment Resolution

61

ANALYZE REAL CASES 25. In April of 2004, the U.S. Supreme Court refused to hear the appeal of a case involving Virginia Military Institute (VMI). The case centered on the issue of whether or not, at a public supported college such as VMI, a prayer traditionally offered before the evening meal should be prohibited as a violation of the constitutional mandate requiring the separation of church and state. In an unusual step, two of the Justices of the Court criticized the other Justices for avoiding the responsibility of rendering a clear decision with attendant guidelines on the matter. Failing to take the case left in place a lower court’s decision against having the prayer. The two Justices pointed out that, although there were previous decisions against such prayer in the public schools where attendance was enforced, colleges were voluntary institutions. Should the court have taken the case? How do you feel the court should rule on the mandatory recital of the pledge of allegiance, with the phrase “Under God” therein, in public schools or colleges? (Bunting v. Mellen, USSC 03-863, 181 F. Supp. 2d 619) 26. Taylor and Fitz Coal Company had a dispute over the amount of money due under a mineral lease.They submitted the dispute to arbitration according to the provisions of the lease. The arbitrators awarded the lessor, Taylor, $37,214.67. Taylor did not like the amount awarded and filed suit in court. Will the court conduct a trial to determine whether the amount is fair? (Taylor v. Fitz Coal Company, 618 S.W.2d 432, Ky.) 27. A 17-year-old boy first appeared in juvenile court to answer charges of vehicular homicide. The incident had occurred when he was 16. By the time the case actually went to trial the boy was 18. Should he be tried as an adult? Why or why not? (Commission v. A Juvenile, 545 N.E.2d 1164) 28. When a tip was received that marijuana was being grown in his greenhouse, Michael Riley’s property was observed from a helicopter by an investigating officer. The helicopter flyover was deemed necessary because the items within the greenhouse could not be observed from a nearby road. Riley was subsequently arrested and tried for offenses stemming from his cultivation efforts. The defense challenged the use of the

62

Chapter 3 > Court Systems

information acquired by the helicopter flyover saying that it was an illegal search under the Fourth Amendment to the U.S. Constitution. The Florida Supreme Court agreed with the defense. Could the State of Florida appeal the decision to the U.S. Supreme Court? Why or why not? (Florida v. Riley, 102 L.Ed. 2d 835) 29. Colleen Donnelly filed charges with the Equal Employment Opportunity Commission (EEOC) against her employer, Yellow Freight, for sex discrimination. After she filed the charges, Colleen received notice from the EEOC that she had 90 days to bring suit. This 90-day period was a procedure for claims under the federal Civil Rights Act of 1964. Within that period Donnelly did indeed file suit but in an Illinois state court under an Illinois statute that also prohibits such discrimination. After the 90-day period had expired, however, she tried to transfer the suit to a federal court. Yellow Freight defended claiming that she could not make such a transfer as the 90-day period had expired for federal actions. Should she be allowed to go forward with her suit? Why or why not? (Donnelly v. Yellow Freight System, Inc., 874 F.2d 402) 30. A government-owned P-51 fighter plane landed at a Los Angeles airport and waited on a runway for a tow truck, on instructions from the tower. Shortly thereafter, a plane owned by Douglas Aircraft Company began approaching the airfield to land. The Douglas aircraft struck the P-51, which was parked on the runway. The United States brought suit against Douglas, claiming that the Douglas pilot was negligent. During the trial, evidence was introduced indicating that the Douglas pilot was careless in not seeing the parked P-51, but that the airport was covered with a haze and the P-51 was painted in camouflage colors. Also, the Douglas pilot had “zigzagged” his plane while taxiing in order to improve his forward vision. The trial was conducted before a jury. The government claimed that the issue of whether the Douglas pilot was negligent was an issue of law for the judge to decide. Douglas claimed that it was an issue of fact for the jury to decide. Which one is correct? (United States v. Douglas Aircraft Company, 169 F.2d 755, 9th Cir.)

Case For Legal Thinking TEXAS V. JOHNSON USSC, 109 S U P R E M E C O U R T R E P O R T E R 2533

BACKGROUND In 1984, during the Republican National Convention in Dallas, Texas, a political demonstration was held. The point of the demonstration was to protest the policies of the President of the United States, Ronald Reagan. Mr. Reagan was to be re-nominated by the convention for a second term. The demonstration also targeted several Dallas-based corporations for their government-related activities. FACTS As the demonstration was being conducted, a protestor took an American flag from a pole outside one of the buildings. The protestor passed the flag to another protestor, Johnson, who subsequently doused it with kerosene and set it afire. As the flag of the United States of America burned, the protestors chanted, “America, the red, white, and blue, we spit on you.” The protestor who set the flag afire was ultimately charged with a crime and tried in a Texas trial court. LOWER COURT The trial court found Johnson guilty of violating a state statute which prohibited the desecration of state or national flags. The statute defined desecration as the physical mistreatment of such objects so as to seriously offend individuals likely to observe or discover the act.

© GETTY IMAGES/PHOTODISC

APPEAL Johnson appealed his conviction on the basis of the First Amendment’s guarantee of freedom of speech. A Texas Court of Appeals affirmed the convic-

tion even after noting that such a statute, which might have a potentially chilling effect on speech, deserved close scrutiny in its application. However, said the court, the conviction could be upheld in this instance as being necessary to guarantee the peaceful nature of the assembly due to the highly inflammatory nature of a flag burning. The case was then appealed to the Texas Court of Criminal Appeals. The Court of Criminal Appeals reversed the decisions of the inferior courts and sent the case back to the trial court with instructions to dismiss the charges. The Court, in so doing, indicated that the statute was too broad in its reference to protecting against a breach of the peace. The U.S. Supreme Court then took jurisdiction by a writ of certiorari.

ISSUE Was the conviction of Johnson justifiable by the state of Texas as an attempt to protect against breaches of the peace, or did the flag burning have overriding elements of communication that the state statute failed to protect? DECISION The United States Supreme Court, in an opinion written by Justice Brennan, held that the First Amendment freedom of speech had indeed been violated by the application of the statute. PRACTICE JUDGING 1. Should the court affirm or reverse as a consequence? 2. The record did not contain any evidence indicating that a breach of the peace had indeed occurred. How do you think this affected the decision? 3. There is an exception for “fighting words” provided for governments who want to enforce statutes that might chill public speech. Could this exception have been applied to this situation in order to uphold the conviction?

Case For Legal Thinking

63

Chapter 4 Criminal Law and Procedure

While sitting in his pickup waiting to cross South Street in Springfield, Missouri, Thomas, an infantryman on leave from the war in Iraq, was hit and killed by Hercule, a 45-year-old intoxicated driver. At the time, Hercule was driving at high speed trying to evade pursuing police cars. Hercule was arrested at the scene. At trial, his attorneys were able to have the blood-test evidence that showed a 0.20 level of intoxication (double the legal limit) thrown out. Hercule was convicted only of second-degree manslaughter, served a week of “shock” jail time, and was set free on two years probation. Outraged at what he termed the failure of the legal system, Mark, Thomas’s younger brother, took his parent’s car and ran Hercule over in the street as he left the jail. Hercule, a married father of four, was crippled for life as a result.

WHERE DO

YOU STAND?

1. Was Mark morally justified in his actions? 2. What action should the law take against Mark?

64

Chapter 4 > Criminal Law and Procedure

© GETTY IMAGES/PHOTODISC

4-1 Criminal Law 4-2 Criminal Procedure

4-1 Criminal Law GOALS

KEY TERMS

■ Understand the three elements that make up a

crime vicarious criminal liability

criminal act ■ Classify crimes according to the severity of their potential sentences ■ Identify the types of crimes that affect business

Crimes and Criminal Behavior O’Brien, a vice president at the Del Norte Credit Union, cleverly juggled the company books over a period of years. During that time, he took at least $135,000 belonging to the credit union. When the theft was discovered by outside auditors, O’Brien repaid the money with interest.

What's Your Verdict?

Has he committed a crime despite the repayment?

The most fundamental characteristic of a crime is that it is a punishable offense against society. When a crime occurs, society—acting through police and prosecutors—attempts to identify, arrest, prosecute, and punish the criminal. These efforts are designed to protect society, not the victim of the crime. As a consequence, it is up to society’s chosen representatives to determine what action to take against the person who committed the crime. The victim can sue identified criminals for civil damages. However, victims seldom do so because few criminals have the ability to pay judgments.

N

ot more than two centuries ago society routinely imposed a single punishment for a felony. That punishment was death. What do you think was happening at that time in history that required such a level of punishment?

felony misdemeanor white-collar crimes

Crimes should be carefully defined by statutes, or laws enacted by state or federal legislatures. A well-worded criminal statute clearly and specifically defines the behavior it prohibits. It does not interfere with protected individual rights in so doing.

Elements of Criminal Acts Before you can be convicted of a crime, the prosecution must prove three elements regarding your behavior at the trial. These elements are 1. whether you had a duty to do or not to do a certain thing 2. whether you performed an act or omission in violation of that duty 3. whether or not you had criminal intent DUTY Generally, under our system everyone has a legally enforceable duty to conform his or her conduct to the law’s requirements. To establish a specific legal duty in a trial, the prosecutor cites to the judge a statute that prescribes the duty owed in the case at hand. VIOLATION OF THE DUTY The breach of duty—the

specific conduct of the defendant that violates the statute—is the criminal act. For example, all states have statutes that make battery a crime. These statutes often define criminal battery as “the intentional causing of bodily harm to another person.” A breach of this duty could be proved in a trial by the testimony of a witness who saw the defendant punch the victim. CRIMINAL INTENT The third element, criminal

intent, also must be proven in most cases. Criminal intent generally means that the defendant intended to commit the specific act or omission defined as criminal in the controlling statute. 4-1 Criminal Law

65

Note that intent and motive are separate in the eyes of the law. Intent refers strictly to the state of mind that must be present concurrently with the alleged criminal act or failure to act. Motive refers to what specifically drives a person to act or refrain from doing so. In What’s Your Verdict? O’Brien owed a duty, defined by state statute, to not take the credit union’s money. Violation of this duty, the criminal conduct of taking another’s property or money by a person to whom it has been entrusted, is embezzlement. This act could be proved with the testimony of the auditors. O’Brien’s criminal intent, to wrongfully deprive others of the money, can be inferred from his conduct. So O’Brien did commit a crime. His return of the money does not alter this fact. Criminal intent creates two issues related to corporations. First, can a corporation, which is an organization, form criminal intent the way humans can? The answer is yes. If the corporation’s employees have criminal intent, their employer may be judged to have criminal intent. If the employees were doing their assigned duties and the criminal act benefits the organization, most courts will hold that the organization had criminal intent. The second issue relates to corporate presidents, treasurers, and other officers. When a corporate employee commits a crime, can officers be held criminally responsible? Again, the answer often is yes. In some situations, the officer will be held criminally liable under the doctrine of vicarious criminal liability. Vicarious means substituted. The criminal

ADA ACCESSIBILITY GUIDELINES he Americans with Disabilities Act (ADA) is a comprehensive federal civil rights statute that outlaws discrimination on the basis of disability. On July 23, 2004, after years of planning and evaluation, the Architectural and Transportation Barriers Compliance Board (Access Board) issued several amendments to the Americans with Disabilities Act Accessibility Guidelines (ADAAG) to cover newly constructed and altered recreation facilities. These rules were intended to provide access for the disabled to facilities such as amusem*nt park

T

66

Chapter 4 > Criminal Law and Procedure

intent of the employee is used as a substitute for the requirement of criminal intent for an officer. Criminal intent also is related to age. Under early common law, children under age 7 were considered incapable of forming the criminal intent necessary for crimes. Those over age 14 were presumed capable of recognizing the difference between right and wrong and so were as accountable as adults for their acts. For children ages 7 through 14, such ability had to be proved. Today, statutes in most states fix the age of criminal liability at 18, but the figure ranges from 16 to 19. State statutes provide that minors as young as 7 may be tried and punished as adults if they are accused of serious crimes such as murder. Generally, however, what is a crime for an adult is juvenile delinquency for a minor. The lack of the mental capacity to know the difference between right and wrong can be a defense to being held responsible for the commission of criminal acts. Accordingly, insane persons are not held responsible for their criminal acts. However, people who voluntarily become intoxicated or drugged are held responsible. Some minor crimes do not require the element of criminal intent. Traffic offenses are an example. A driver may be exceeding the speed limit without noticing, yet still be in violation of the law. In addition, in certain circ*mstances extreme carelessness may be considered the same as criminal intent. Suppose you drive 80 miles per hour through a residential neighborhood while drunk and kill a

rides, marinas, golf courses, physical fitness facilities, recreational camps, etc. THINK CRITICALLY Evaluate these new rules based on their stated cost to the owners of such facilities. (Access lawxtra.swlearning.com for a link to a detailed listing of estimated costs per type of facility.) If you were the president of a company in the recreational field, what impact might these rules have on your decision to update or build new facilities?

Crimes can be classified in many different ways. One method of classification is to identify against whom or what they are committed. The table below lists and gives examples of this classification method. Crimes also can be classified as either felonies or misdemeanors according to the severity of their potential sentences.

Felony

© GETTY IMAGES/PHOTODISC

A felony is a crime punishable by confinement for more than a year in a state prison or by a fine of more than $1,000, or both—or even death. Murder, kidnapping, arson, rape, robbery, burglary, embezzlement, forgery, theft of large

Classification of Crimes by Party, Interest, or Property Injured Imagine that the person in this photo killed a pedestrian while driving under the influence of alcohol. Do you think the law should treat the intent of this person as criminal? Why or why not?

pedestrian. You may not have intended to strike the pedestrian with your car. However, your conduct was so careless that in many jurisdictions courts will treat it the same as criminal intent. You could be convicted of the crime of vehicular homicide. CHECKPOINt What three elements must

be proven at trial before someone can be convicted of a crime?

Classifications of Crimes Murdock was a witness at a civil trial for damages. Before testifying, he took an oath “to tell the truth, the whole truth, and nothing but the truth.” Nevertheless, while being questioned by one of the attorneys, Murdock deliberately lied, hoping to help the defendant.

What's Your Verdict?

What crime has Murdock committed?

Crimes against . . .

Type of Crime

a person

assault and battery kidnapping rape murder

property

theft robbery embezzlement

government and administration of justice

treason tax evasion perjury

public peace and order

rioting disorderly conduct illegal speeding

realty

burglary arson criminal trespass

consumers

fraudulent sale of securities violation of pure food and drug laws

decency

bigamy obscenity prostitution

4-1 Criminal Law

67

sums, and perjury are examples of felonies. People who lie under oath (as Murdock did in What’s Your Verdict?) commit the felony of perjury.

Misdemeanor A misdemeanor is a less serious crime. It usually is punishable by confinement in a county or city jail for one year or less, by a fine of $1,000 or less, or both. Crimes such as disorderly conduct and speeding usually are misdemeanors. Some states classify lesser misdemeanors as infractions. Persons convicted of infractions can only be fined. Because there is no risk of being jailed, the defendant charged with an infraction is not entitled to a jury trial. Parking violations and littering are examples of infractions.

Business-Related Crimes Officers of the major competing DRAM memory chip manufacturers met at a trade convention. All of the officers agreed to use the same wholesale prices in bids to supply computer retailers with the chips. They also agreed to follow the lead of the biggest company in making future price changes. Each officer agreed to promote sales by concentrating only within an assigned geographical region.

What's Your Verdict?

Have the officers and their companies committed any crime?

CHECKPOINt Name the two categories

Like people, businesses are subject to general criminal law. Offenses committed in the business world typically are referred to as white-collar crimes. These crimes do not involve force or violence and do not cause physical injury to people or physical damage to property. Common examples of whitecollar crimes are not paying income taxes, stock fraud, using false weighing machines, conspiring to fix prices, making false fire insurance and auto insurance claims, engaging in false advertising, committing bribery, engaging in political corruption, and embezzling. Because physical violence is not involved, courts tend to be more lenient with white-collar criminals. Punishments usually include fines or short prison sentences. In What’s Your Verdict? the corporate officers and their corporations were guilty of violating criminal portions of the antitrust laws. Antitrust laws state that competing companies may not cooperate Littering in this beach town carries a fine of $1,000. Research to find out in fixing prices or in dividing sales the fine for littering in your area. regions. Antitrust laws require that business firms compete with one another. Some of the more common business-related crimes follow. © GETTY IMAGES/PHOTODISC

of crimes classified by the severity of their potential sentences.

LARCENY Larceny, commonly known as theft, is the wrongful taking of money or personal property belonging to someone else, with intent

68

Chapter 4 > Criminal Law and Procedure

to deprive the owner of it. Robbery is a variation of larceny. It is the taking of property from another’s person or immediate presence, against the victim’s will, by force or by causing fear. Burglary is another variation of larceny. It is entering a building without permission when intending to commit a crime. Other types of larceny include shoplifting, pick pocketing, and purse snatching. Larceny may be either a felony or a misdemeanor. The classification is determined by the value of the property stolen and other circ*mstances. Robbery and burglary are always felonies. A thief who sells the stolen goods is guilty of the separate crime of selling stolen property. RECEIVING STOLEN PROPERTY Knowingly receiv-

© GETTY IMAGES/PHOTODISC

ing stolen property consists of either receiving or buying property known to be stolen, with intent to deprive the rightful owner of the property. One who receives stolen property is known as a fence. FALSE PRETENSES One who obtains money or other property by lying about a past or existing fact is guilty of false pretenses. This crime differs from larceny because the victim parts with the property voluntarily. False pretenses is a type of fraud. FORGERY Forgery is falsely making or materially

altering a writing to defraud another. The most common forgeries are found on checks when one signs another’s name without permission to do so. Forgery also includes altering a check, such as changing “$7” to “$70” and “Seven” to “Seventy.” Forgery usually is a felony. Of course, if others authorize you to sign their names, there is no forgery. BRIBERY Bribery is unlawfully offering or giving anything of value to influence performance of an official in the carrying out of his or her public or legal duties. Soliciting or accepting the bribe is also criminal. In many states, bribing nongovernmental parties is also a form of bribery called commercial bribery. Thus, paying a private company’s purchasing agent to obtain a sale may be bribery. It is usually bribery when a professional gambler pays an athlete to lose a game intentionally. The federal Foreign Corrupt Practices Act of 1977 prohibits most instances of bribery in foreign countries by U.S. companies. Unlike the United States, many other nations do not make criminal or prosecute such actions by their companies.

What is the evidence that this man is committing a burglary? Is there any evidence that a robbery is in process? If so, what is it?

COMPUTER CRIME The computer revolution has created a range of problems for criminal law. For example, larceny is “the wrongful taking of the personal property of others.” This traditional definition of the crime made it difficult to prosecute those who steal computer data. Many courts concluded that there was not a “taking” if an intruder merely copied the information in the computer. Even if an intruder copied and erased computer information, some courts concluded that there was no taking of “personal property”

Search the Internet for state and federal laws that prohibit computer-related crimes. An example would be to research electronic identity theft in all its forms from credit card fraud to invasion of accounts.

4-1 Criminal Law

69

but only the loss of electrical impulses, which no one really owns. In response, the federal and state governments have created new criminal laws, such as the federal Computer Fraud and Abuse Act (Title 18 U.S.C. Section 1030) specifically targeting computer-related crimes. EXTORTION Extortion (commonly known as

blackmail ) is obtaining money or other property from a person by wrongful use of force, fear, or Intercomp is a U.S. company that routinely buys surplus computers in this country and markets them overseas. Intercomp agreed to reimburse its agent for gifts given to a foreign government official. The official decided which companies were to be placed on a list of those authorized to do business in the country. Intercomp’s non-U.S. competitors all gave similar gifts to the official. However, for doing so, Intercomp was prosecuted in the United States under its Foreign Corrupt Practices Act, convicted, and fined $500,000.

In THIS CASE

the power of office. For example, the extortionist (blackmailer) may threaten to inflict bodily injury. At other times, the threat may be to expose a secret crime or embarrassing fact if payment is not made. CONSPIRACY An agreement between two or

more persons to commit a crime is called conspiracy. Usually the agreement is secret. The conspiracy is a crime separate from the crime the parties planned to commit. Depending on the circ*mstances, conspiracy may be either a felony or a misdemeanor. Business executives of competing corporations sometimes conspire to fix prices or to divide markets. ARSON Arson is the willful and illegal burning or exploding of a building. Arson occurs when someone intentionally starts a fire or causes an explosion that damages or destroys a building or other property without the owner’s consent. In some states, arson also occurs if you burn your own building to defraud an insurer. CHECKPOINt Name the types of

business-related crimes.

4-1 Assessment school.cengage.com/blaw/lawxtra

THINK ABOUT LEGAL CONCEPTS 1. Legally, a crime is considered an offense against (a) the victim (b) society (c) the court (d) none of the above 2. Ben is convicted and sentenced to two to five years in jail. The crime he has committed is a (a) misdemeanor (b) felony

3. In some situations, an officer of a corporation will be held criminally liable for the acts of an employee under the doctrine of _?_. 4. Falsely making or materially altering a writing to defraud another is known as the crime of false pretenses. True or False?

THINK CRITICALLY ABOUT EVIDENCE Study the situation, answer the questions, and then prepare arguments to support your answers. 5. Phillips developed a scheme to generate funds by sending bogus bills for small amounts for District Sanitation Services to residents of affluent areas. Enough people paid these bills to make the practice quite profitable. What crime has Phillips committed? 6. A corporation was cited and charged with illegal pollution for dumping chemical wastes

70

Chapter 4 > Criminal Law and Procedure

into a river. The dumping happened when an employee mistakenly opened the wrong valve. The company pleaded not guilty because the dumping was not intentional. Neither the company nor the employee knew of the ban on dumping this particular chemical. Is either argument a good defense? Why or why not?

4-2 Criminal Procedure GOALS

KEY TERMS

■ Know the rights people have when arrested

probable cause defense procedural defense substantive defense self-defense

and their potential criminal liability for the actions of others ■ Name and describe the two types of defenses to criminal charges ■ Understand appropriate punishments for crimes

Rights and Responsibilities During a routine traffic stop of a small truck for speeding, an officer became suspicious of the cargo the truck contained due to a smell coming from inside. When his request to search the truck was refused, the officer radioed for the assistance of a drug dog. Unfortunately, the dog was unavailable. Finally, he ordered the driver to open the cargo area. When the driver did so, the officer found more than a ton of marijuana. At trial, the defense attorney maintained that it was an illegal search and seizure and that the marijuana should not be allowed to be used as evidence.

What's Your Verdict?

Do you agree? Why or why not?

One of the major objectives of the framers of the Constitution of the United States was to protect individuals from the powers of the federal government. The constitutional limitations created as a consequence have been extended over the years so as to also apply to the state and local governments. The authors of the Constitution believed it was better for society to give individuals too much liberty than to allow the government too much power. Thus, in this country, people suspected or accused of criminal conduct have rights that are not available in many other countries. For an example of this, see the Global Issues feature on this page.

Rights of the Accused The constitutional right to due process requires fundamental fairness in governmental actions. It

immunity contempt of court punishment plea bargaining

requires fair procedures during an investigation and in court. As discussed in Chapter 2, criminal defendants may not be compelled to testify against themselves. Evidence presented against them cannot be developed by unreasonable searches and seizures by the authorities. For example, in What’s Your Verdict? the police officer’s sense of smell was not as keen as the police dog’s. Therefore, the search was deemed unreasonable as it lacked probable cause (a reasonable ground for belief ) and the marijuana could not be used as evidence. Perhaps the most important right is the right of the accused criminal to be represented by a lawyer. The state will provide a lawyer for a person who cannot afford to hire one.

Criminal Procedure in France Criminal procedure in France—and most other European countries—is significantly different from criminal procedure in the United States and Great Britain. A few examples from the French system are • persons accused of crimes are not presumed innocent until proven guilty • victims and their families can become parties to investigations • judges take an active role in trials, including examining and cross-examining witnesses • persons accused of crimes have no protection from self-incrimination and can be compelled to testify • testimony based on hearsay and opinion is allowed.

4-2 Criminal Procedure

71

Defenses to Criminal Charges Access school.cengage.com/blaw/lawxtra and click on the link for Chapter 4. Compare Amnesty International’s reports on rights abuses for the United States and France. Write a one-page report on your findings.

school.cengage.com/blaw/lawxtra

Will and Zack were arrested for stealing a car. Both signed confessions at the police station. At their trial, they claimed that their right to due process had been violated. They said they had not been advised of their right to remain silent and to have a lawyer present when questioned.

What's Your Verdict?

If true, are those good defenses?

To convict a person of a crime, the evidence must establish guilt with proof “beyond a reasonable doubt.” This means that there must be no sufficient basis placed in evidence at the trial that would logically indicate that the defendant did not commit the crime. Defendants have a constitutional right to a trial by jury. There will be a jury if either the state prosecutor or the defendant requests one. In criminal jury trials, the defendant usually is found guilty only if all the jurors vote to convict.

To insure the fair application of laws, a criminal defendant may often be able to avoid conviction in court by the use of one or more defenses. In criminal law, a defense is a legal position taken by an accused to defeat the charges against him or her. In court the state must prove that the defendant is guilty beyond a reasonable doubt. Even when the prosecution has conclusive evidence to

Responsibility for the Criminal Conduct of Others

© GETTY IMAGES/PHOTODISC

A person who knowingly aids another in the commission of a crime also is guilty of criminal wrongdoing. For example, one who acts as a lookout to warn a burglar of the approach of the police is an accomplice in the burglary. Similarly, one who plans the crime, or otherwise intentionally helps, is guilty of the same crime. In most jurisdictions, if someone is killed during the commission of a felony, all the people who participated are guilty of the crime of felony murder. As discussed previously in this chapter, corporations can be held vicariously liable for the conduct of their employees. Also, officers of corporations may be criminally liable for their actions as managers. CHECKPOINt What constitutional rights

would you have if you were accused of a crime? Why would it be important to be represented by an attorney if you were accused of a crime?

72

Chapter 4 > Criminal Law and Procedure

show such guilt, all too often that evidence cannot be introduced due to procedural defenses. In addition, even if there is sufficient admissible evidence to show guilt, defendants also may use substantive defenses to excuse their conduct.

Procedural Defenses Procedural defenses are based on problems with the way evidence is obtained or the way an accused person is arrested, questioned, tried, or punished. For example, a defendant who had confessed to a crime might say that she signed the confession only because she was threatened by the police. This would be a procedural defense. Ignorance of the law is not a defense. The legal system assumes everyone knows the law. Thus, if you park in a no-parking area because you did not see the sign, you have no defense. In What’s Your Verdict? if Will and Zack could prove they were not so advised, such a violation of their procedural rights under the Constitution would be a valid defense to any attempt by the prosecution to use their confessions as evidence against them. Because of the likelihood of such claims by defendants, however, most police departments videotape the rendering of the Miranda warning. They also obtain signed acknowledgements from suspects that the warning has been given.

Substantive Defenses Substantive defenses disprove, justify, or excuse the

alleged crime. Most substantive defenses discredit the facts that the state sought to establish. In other words they discredit the very substance of the prosecution’s case against the defendant. For example, an eyewitness may have placed the defendant at the scene of the crime. The defendant may establish a substantive defense by showing that he was in the hospital at the time of the alleged crime. Selfdefense, criminal insanity, and immunity are other examples of substantive defenses. Self-defense is the use of the force that appears to be reasonably necessary to the victim to prevent death, serious bodily harm, rape, or kidnapping. This defense also extends to members of one’s family and household and to others whom one has a legal duty to protect. You may not use deadly force if non-deadly force appears reasonably sufficient. Only non-deadly force may be used to protect or recover property. You may not set deadly traps to protect unoccupied buildings. In addition, a civilian may not shoot a thief who is escaping with stolen property. Criminal insanity generally exists when, because of a verifiable mental disease or defect, the accused does not know the difference between right and wrong. If the accused is criminally insane, there is no criminal intent and therefore no crime. At a trial, the defendant must prove the criminal insanity. Immunity is freedom from prosecution even when one has committed the crime charged. Sometimes one criminal may be granted immunity

A state law makes “hit-andrun” driving a crime. The law requires drivers of motor vehicles involved in an accident to stay at the scene, give their names and addresses, and show their driver’s licenses. Barlow, who was arrested for violating this law, claimed that the law was unconstitutional. He said that the law violated his Fifth Amendment right against self-incrimination. In concluding the law was constitutional, the court said that the right against self-incrimination applies only to statements that would implicate a person in a crime. Merely identifying oneself as a party to an accident does not indicate guilt.

© GETTY IMAGES/PHOTODISC

In THIS CASE

Why must police officers be very careful in the way that they arrest and question accused individuals?

4-2 Criminal Procedure

73

A

law passed in Oklahoma in the late 1980s, known as the “Make My Day” gun law, is credited with cutting burglaries in that state in half since its passage. The law was enacted after 66-year-old Dr. Frank Sommer was charged with a criminal act for shooting a burglar in his own home. The state senator who sponsored the law said that its purpose is to protect the victim of crime who defends his home and his family against unlawful intrusion from any criminal prosecution or civil action.

CHECKPOINt Name the two defense

© GETTY IMAGES/PHOTODISC

in exchange for an agreement to testify about the criminal conduct of several other criminals. In other instances, there is no agreement. Instead, the government grants immunity to a reluctant witness to remove the privilege against selfincrimination. A witness who refuses to testify after the grant of immunity is in contempt of court. Contempt of court is action that hinders the administration of justice. It is a crime punishable by imprisonment. Why do you think society regards contempt of court a crime punishable by imprisonment?

categories and give an example of each.

Punishments for Crimes To conserve water, a city ordinance prohibited the watering of gardens, flower beds, and yards after the declaration of a drought emergency. Gill was on vacation when the declaration was issued. As soon as she returned from the trip, she began to water her lawn. Gill was caught and cited for violating the ordinance.

What's Your Verdict?

What is an appropriate penalty for this type of offense?

74

Chapter 4 > Criminal Law and Procedure

“Let the punishment fit the crime” is more easily said than done. Any penalty provided by law and imposed by a court is called a punishment. The purpose is not to remedy the wrong but rather to discipline the wrongdoer. If reasonably swift and certain, punishment should also deter others from similar behavior. Punishment also may remove criminals from society and, in some cases, criminals may be rehabilitated during their sentences. Punishments for crimes range from fines to imprisonment, and in some cases, death. In What’s Your Verdict? Gill was guilty of an infraction, which did not require criminal intent. Nevertheless, her conduct was illegal so she would probably be fined. Criminal statutes ordinarily set maximum limits for punishment, but allow a judge discretion within those limits. An accused person may agree to plead guilty to a less serious crime in exchange for having a more serious charge dropped. This is called plea bargaining. The accused voluntarily gives up

the right to a public trial to avoid the risk of a greater penalty if convicted. Plea bargains must be approved by the judge of the court with the criminal jurisdiction over the case.

CHECKPOINt What is the purpose of

punishment for a crime?

4-2 Assessment school.cengage.com/blaw/lawxtra

THINK ABOUT LEGAL CONCEPTS 1. The authors of the Constitution believed it was better for society to give individuals too much liberty than to allow the government too much power. True or False? 2. An individual who helped in a car jacking in which someone was killed by another car jacker can be charged with felony murder. True or False? 3. Which of the following is not a substantive defense? (a) self-defense (b) immunity

(c) criminal insanity (d) All of the above are substantive defenses. 4. Ignorance of the law is a procedural defense. True or False? 5. Immunity is never granted to someone who has committed the crime. True or False? 6. A deal in which the prosecutor offers an accused an opportunity to plead guilty to a lesser charge in return for the accused testifying against other involved parties is called a _?_.

THINK CRITICALLY ABOUT EVIDENCE Study the situation, answer the questions, and then prepare arguments to support your answers. 7. Sharon spent the weekend with her friend Amelia. Amelia proposed a plan for shoplifting CDs from a local music store. Sharon was to go to the store clerk, say she felt ill, and then pretend to faint. This distraction would allow Amelia, at the other end of the store, to place CDs in her shopping bag without risk of being seen. At first Sharon said she could not do something like that because it is against the law. Amelia argued that Sharon would not be breaking the law, only Amelia would. Is Amelia right? If a person can think of a way to profit by violating the law without risk, what reasons are there for not breaking the law? 8. Art is charged with kidnapping. After listening to all the evidence, 11 of the 12 jurors found him guilty. The twelfth juror voted he was not guilty. Has Art been convicted of the crime?

9. Rosa shot a burglar in her home when he was about to enter her bedroom with a knife in his hand. What would be a good defense for her? 10. John left his home state of Missouri early one afternoon to drive to New Mexico. Eventually the sun set, but John continued on. Hours later, while driving 70 mph on an interstate highway east of Amarillo, he was pulled over by the Texas highway patrol. The last speed limit sign John had seen had been in Oklahoma and had read 70 mph. As she issued the ticket, the officer noted that the speed limit in Texas on interstates is 70 during the day but drops to 65 at night. John protested he was unaware of the change. Was he still guilty of speeding?

4-2 Criminal Procedure

75

Chapter 4 Assessment

school.cengage.com/blaw/lawxtra

CONCEPTS IN BRIEF 4-1 Criminal Law 1. A crime is a punishable offense against society. In order to convict, the prosecution must establish a duty, an act or omission in violation of the duty, and, in most cases, criminal intent. 2. Crimes are generally categorized as felonies or misdemeanors. Some states classify minor misdemeanors as infractions. 3. Some crimes in which a business may be the victim are robbery, burglary, shoplifting, employee theft, passing bad checks, vandalism, receiving stolen property, and embezzlement. 4. Some crimes in which a business person or firm may be the perpetrator are income tax evasion, price fixing, false advertising, and bribery. 5. Generally, any adult capable of knowing the difference between right and wrong is considered sane under the law.

4-2 Criminal Procedure 6. Anyone accused of committing a crime has certain due process rights. These rights include freedom from arrest without probable cause; the right to be represented by a lawyer; the right to cross-examine witnesses; the right to not testify against oneself; and the right to a speedy, public, fair trial. 7. Procedural defenses relate to the way evidence is obtained or the way the accused person is arrested, questioned, tried, or punished. Substantive defenses disprove, justify, or excuse the alleged crime. Common substantive defenses are self-defense, criminal insanity, and immunity. 8. Crimes are punishable by fine, imprisonment, or both. Some states execute certain criminals.

YOUR LEGAL VOCABULARY Match each statement with the term that it best defines. Some terms may not be used. 1. Legal position that disproves, justifies, or otherwise excuses an alleged criminal act a. contempt of court 2. Accused person admitting guilt to a less serious crime in b. crime exchange for having a more serious charge dropped c. defense 3. The use of the force that appears to be reasonably necessary to d. felony the victim to prevent death, serious bodily harm, rape, or kide. immunity napping f. misdemeanor 4. Punishable offense against society g. plea bargaining 5. Action that hinders the administration of justice h. probable cause 6. Crime punishable by either a fine of more than $1,000 or by i. procedural defense confinement for more than one year in a state prison or both or by death j. punishment 7. Substituted criminal liability k. self-defense 8. Freedom from prosecution even when one has committed the l. substantive defense crime charged m. vicarious criminal liability 9. Any penalty provided by law and imposed by a court n. white-collar crime 10. Crime punishable by confinement in a county or city jail for one year or less, by fine of $1,000 or less, or both 11. A criminal defense based on how the evidence was obtained or how the accused was arrested, questioned, or tried

76

Chapter 4 > Criminal Law and Procedure

REVIEW LEGAL CONCEPTS 12. Why must criminal statutes carefully and clearly define the prohibited behavior? 13. Why are certain acts crimes in one state but not in another? Why are there varying levels of punishment for the same crime in different states?

14. Explain the significance of the main steps in a criminal prosecution. Is the cause of justice served when a defense attorney gets his client off because of a “loophole” in the law? Consider both the short-term and the long-term consequences.

WRITE ABOUT LEGAL CONCEPTS 15. An old adage of the law states, “As society becomes less secure, criminal punishments become more severe.” Write a paragraph relating this to the United States’ experience with the death penalty and other punishments in the last few decades.

16. HOT DEBATE Write a paragraph debating whether or not the protections afforded those accused of criminal activity by the Constitution are mainly used by individuals who are truly guilty to avoid their deserved punishment.

MAKE ACADEMIC CONNECTIONS 17. SOCIAL STUDIES As a result of a U.S. Supreme Court decision in the early 1960s, improperly or illegally obtained evidence cannot be used to convict an accused in the court system. This is referred to as the “exclusionary rule,” as the evidence is excluded from consideration by the court. The rule was put into place to stop police from using the wrong tactics to

obtain incriminating evidence. Other developed countries handle such actions by their police force in a different manner. Research how England and two other European countries discipline improper or illegal attempts to gather evidence by their police and whether or not the evidence developed thereby can be used against the accused at trial.

THINK CRITICALLY ABOUT EVIDENCE Study the situation, answer the questions, and then prepare arguments to support your answers. The entire action took only 20 seconds. Bif fin18. Elton had three children. As they grew up and ished long before Gail returned. Has Bif commitbecame eligible to drive, Elton bought each a ted a crime? If so, what crime? car. To avoid paying high premiums for their car insurance, Elton titled each car in his name. 20. Paula’s boyfriend moved to her home town, He then stated to his insurance company that Oklahoma City, from Houston, Texas. Two weeks he and his wife were the primary drivers on after he arrived, he asked her to phone his the vehicles. When the youngest child totalled bank in Houston and inquire about his balance. “his” car by hitting a large deer, the insurance She did so as a favor to him and found out the company paid to replace the car. What could balance was more than $40,000. As soon as Elton be charged with should his deception be she told him, he left and drove to Houston to uncovered? remove it from his account. In Houston, he was arrested by the FBI for fraud and several related 19. Bif was in Gail’s office waiting to go to lunch crimes. Paula was then charged with the federal with her. Gail owned a business in competition crime of making a phone call across state lines with Bif’s business. When Gail excused herself for the furtherance of a fraudulent scheme. Her to go to the restroom, Bif looked at her comboyfriend only knew the scheme had been sucputer screen and saw part of a customer list. Bif cessful by Paula’s report of the large balance had a blank diskette which he inserted into the in his old account. What would be a possible computer. He quickly copied the file onto his defense for Paula? diskette. Then he put his diskette in his pocket.

Chapter 4-1 Criminal 4 Assessment Law

77

ANALYZE REAL CASES 21. The Royal Scotsman Inn built a motel that did not comply with the building code. Therefore, Scotsman was refused an occupancy permit. The chair of the county council approached a representative of the motel and offered to have “everything taken care of” in exchange for the payment of $12,000. Scotsman was faced with the possibility of a large loss of revenue. Therefore, Scotsman agreed to pay the money. The Federal Bureau of Investigation arrested the council chairperson after tape recording the discussion and seeing the exchange of the money. What crime did the council chairperson commit? (United States v. Price, 507 F.2d 1349) 22. Basic Construction Company was engaged in the road-paving business. Two of its lower-level managers rigged bids by giving competitors the prices that Basic would bid for work. That is a criminal violation of the Sherman Antitrust Act. Will Basic be criminally liable for the conduct of its manager? (United States v. Basic Construction Company, 711 F.2d 570) 23. Citrin was an employee of International Airport Centers (IAC). As such he was responsible for identifying properties that IAC might want to buy. To help in his work, IAC loaned Citrin a laptop computer on which to store the data he collected. Citrin eventually decided that he would branch off on his own and start a competing firm. Before quitting IAC and turning in his laptop, Citrin downloaded a computer program that wiped out the hard drive which contained the data IAC had paid him to collect as well as evidence of his misconduct. What statute was Citrin convicted of violating? (Int’l Airport Ctrs. L.L.C. v. Citrin, No. 05-1522, 7th Cir., 2006) 24. Pack was the president of Acme Markets, Inc., a large national retail food chain. Both Pack and Acme were charged with violating criminal provisions of the federal Food, Drug, and Cosmetics Act. They were charged with allowing interstate shipments of food contaminated by rodents in an Acme warehouse. Pack defended himself by stating that although he was aware of the problem, he had delegated

78

Chapter 4 > Criminal Law and Procedure

responsibility for the sanitary conditions of food storage to responsible subordinates. Can Pack be criminally liable in these circ*mstances? (United States v. Pack, 95 S. Ct. 1903) 25. Feinberg owned a cigar store in a poor neighborhood of Philadelphia. He sold cans of Sterno, which contains about 4 percent alcohol, to people in the neighborhood who mixed it with water and drank it to become intoxicated. After conducting this business for some time, Feinberg purchased a quantity of Institutional Sterno. It contained 54 percent alcohol. The cans were marked “Danger—Poison,” and had a picture of a skull and crossbones; Feinberg did not warn customers of the difference. As a result, 33 people died from alcohol poisoning. Did Feinberg commit a crime? (Commonwealth v. Feinberg, 234 A.2d 913, Pa.) 26. To help attract convention business to the city, a group of hotels, restaurants, and various other businesses in Portland, Oregon, formed an association. The association was funded by contributions. To provide incentives for contributions to the association, its members agreed to stop doing or curtail doing business with those who did not contribute. As a part of this effort, the Portland Hilton Hotel’s purchasing agent threatened a Hilton supplier with the loss of the hotel’s business unless such a contribution was forthcoming. Such activities and the agreement behind them are criminal violations of the federal antitrust laws. As a result, the federal government charged Hilton accordingly and a guilty verdict was returned at the trial. Hilton appealed, as it had been shown in court that the manager and assistant manager of the hotel had, on at least two occasions, told the purchasing agent not to participate in the boycott. He was instead to follow corporate policy and purchase supplies only on the basis of price, quality, and service. Should the decision be reversed on appeal given the employee’s actions were clearly against corporate policy and directions by corporate executives? (United States v. Hilton Hotels Corporation, 467 F.2d 1000)

Sports & Entertainment Law Criminal Intrusion or Protected Freedom G ALELLA V. O NASSIS

353 F. S U P P . 196, 487 F.2 D 986

BACKGROUND The involvement of the paparazzi in the death of Princess Diana of England brought cries that they were guilty of criminal manslaughter. The incident brought to mind the actions of the man who is said to have given the paparazzi their original bad reputation, Ronald E. Galella. Paparazzi (Italian for “buzzing insects”) became known for their intrusive behavior as a result of documentation in the court records of the many incidents related to the case of Galella v. Onassis.

© BILL RAY/TIME LIFE PICTURES/GETTY IMAGES

FACTS Jacqueline Onassis was the remarried widow of President John F. Kennedy and mother of the two Kennedy children, John and Caroline. Photos of Mrs. Onassis and the children, especially those exposing emotions or questionable behavior, were drawing a premium from publishers in the early 1970s. As a consequence, Galella began a campaign of intrusive behavior likely to provoke marketable snapshots. In her deposition, a tenyear-old Caroline Kennedy said, “Unlike the many other photographers, Mr. Galella often rushes at me, snaps flash bulbs in my face, trails me closely and uses other

techniques that I find dangerous and threatening…. I fear I will be hurt.” On September 24, 1969, Galella allegedly jumped out of some bushes in Central Park in front of John’s oncoming bicycle. The nine-year-old boy swerved to miss the photographer and crashed his bike, injuring himself and causing the Secret Service Agent accompanying the Onassis party to say, “Look what you almost did, you almost killed John.” Ultimately, because of his alleged criminal and tortious behavior, an injunction was issued by a federal district court preventing Galella from coming within 50 yards of Mrs. Onassis. In this action, Mr. Galella wants the injunction to be dismissed.

THE LAW The First Amendment through its interpretation by the U.S. Supreme Court sanctifies and renders almost inviolate the right of the press to report on the actions of public figures.

THE ISSUE Does this right provide immunity from all criminal and civil actions levied against the press for their actions in pursuing a story? HOLDING No, it does not. Said the court, “… the First Amendment does not immunize all conduct designed to gather information about or photographs of a public figure. There is no general constitutional right to assault, harass, or unceasingly shadow or distress public figures.” The injunction was upheld that the “plaintiff, his agents, servants, employees and all persons in active concert and participation with him from, inter alia, approaching within 100 yards of the home of defendant and her children, 100 yards of the schools attended by the children; at all other places and times 75 yards from the children and 50 yards from defendant; from performing surveillance of defendant or her children; from commercially appropriating defendant’s photograph for advertising or trade purposes without defendant’s consent; from communicating or attempting to communicate with defendant or her children.” PRACTICE JUDGING 1. Do you think the court could have achieved its objective by simply enjoining Galella from selling any photos he took of the subjects? Why or why not? 2. Do you think the injunction as issued above was effective? Why or why not?

Sports & Entertainment Law

79

Business Law - PDFCOFFEE.COM (2024)
Top Articles
Latest Posts
Article information

Author: Domingo Moore

Last Updated:

Views: 5499

Rating: 4.2 / 5 (73 voted)

Reviews: 88% of readers found this page helpful

Author information

Name: Domingo Moore

Birthday: 1997-05-20

Address: 6485 Kohler Route, Antonioton, VT 77375-0299

Phone: +3213869077934

Job: Sales Analyst

Hobby: Kayaking, Roller skating, Cabaret, Rugby, Homebrewing, Creative writing, amateur radio

Introduction: My name is Domingo Moore, I am a attractive, gorgeous, funny, jolly, spotless, nice, fantastic person who loves writing and wants to share my knowledge and understanding with you.